You are on page 1of 120

Din n MATHSCOPE

CHUYN S HC
Ch bin: Phm Tin Kha
Ph trch Latex: Phan c Minh, Nguyn Anh Huy

Thng 10/2013

Mc lc

Li ni u . . . . . . . . . . . . . . . . . . . . . . . . . . . . . . . . . . . . . . .

Bc nhy Viete

M u . . . . . . . . . . . . . . . . . . . . . . . . . . . . . . . . . . . . . . . . . .

Li gii nguyn thy ca bi ton v cc vn lin quan . . . . . . . . . . . . . .

Gi cho mt s bi ton . . . . . . . . . . . . . . . . . . . . . . . . . . . . . . . .

14

Cc bi ton th sc . . . . . . . . . . . . . . . . . . . . . . . . . . . . . . . . . . .

15

Vn dng phng php LTE vo gii cc bi ton s hc

17

Mt s khi nim . . . . . . . . . . . . . . . . . . . . . . . . . . . . . . . . . . . . .

17

Hai b . . . . . . . . . . . . . . . . . . . . . . . . . . . . . . . . . . . . . . . . .

18

Lifting The Exponent Lemma (LTE) . . . . . . . . . . . . . . . . . . . . . . . . . .

18

Mt s v d

. . . . . . . . . . . . . . . . . . . . . . . . . . . . . . . . . . . . . . .

21

Bi tp vn dng . . . . . . . . . . . . . . . . . . . . . . . . . . . . . . . . . . . . .

27

Cc bi ton s hc hon v vng quanh

29

Phng php i xng ha . . . . . . . . . . . . . . . . . . . . . . . . . . . . . . . .

29

Phng php dng bt ng thc . . . . . . . . . . . . . . . . . . . . . . . . . . . .

32

Bi tp t luyn . . . . . . . . . . . . . . . . . . . . . . . . . . . . . . . . . . . . .

36

Dy s s hc

39

Dy s nguyn v tnh cht s hc . . . . . . . . . . . . . . . . . . . . . . . . . . .

39

Dy s nguyn v tnh chnh phng . . . . . . . . . . . . . . . . . . . . . . . . . .

47

Mt s hm s hc v ng dng

61

Hm tng cc c s v s cc c s
Kin thc cn nh . . . . . . .
V d p dng . . . . . . . . . .
Bi tp c hng dn, gi . .
Bi tp t gii . . . . . . . . . .

.
.
.
.
.

.
.
.
.
.

.
.
.
.
.

.
.
.
.
.

.
.
.
.
.

61
61
62
65
66

Mt s hm s khc . . . . . . . . . . . . . . . . . . . . . . . . . . . . . . . . . . .
Hm phn nguyn . . . . . . . . . . . . . . . . . . . . . . . . . . . . . . . . . .
Hm tng cc ch s . . . . . . . . . . . . . . . . . . . . . . . . . . . . . . . .

68
68
69

.
.
.
.
.

.
.
.
.
.

.
.
.
.
.

.
.
.
.
.

.
.
.
.
.

.
.
.
.
.

.
.
.
.
.

.
.
.
.
.

.
.
.
.
.

.
.
.
.
.

.
.
.
.
.

.
.
.
.
.

.
.
.
.
.

.
.
.
.
.

.
.
.
.
.

.
.
.
.
.

.
.
.
.
.

.
.
.
.
.

.
.
.
.
.

.
.
.
.
.

.
.
.
.
.

Hm Euler . . . . . . . . . . . . . . . . . . . . . . . . . . . . . . . . . . . . . .

69

Bi tp tng hp . . . . . . . . . . . . . . . . . . . . . . . . . . . . . . . . . . . . .

70

Thng d bnh phng

73

Tnh cht c bn ca thng d bnh phng v k hiu Legendre . . . . . . . . . . .

73

Bi tp v d . . . . . . . . . . . . . . . . . . . . . . . . . . . . . . . . . . . . . . .

76

K hiu Jakobil . . . . . . . . . . . . . . . . . . . . . . . . . . . . . . . . . . . . . .

79

Bi tp v d . . . . . . . . . . . . . . . . . . . . . . . . . . . . . . . . . . . . . . .

80

Khai thc mt b . . . . . . . . . . . . . . . . . . . . . . . . . . . . . . . . . . .

81

Bi tp ngh . . . . . . . . . . . . . . . . . . . . . . . . . . . . . . . . . . . . . .

83

Cp v cn nguyn thy

85

Cp ca mt s nguyn dng . . . . . . . . . . . . . . . . . . . . . . . . . . . . . .

85

Cn nguyn thy . . . . . . . . . . . . . . . . . . . . . . . . . . . . . . . . . . . . .

97
101

Hm phn nguyn v phn l

nh ngha, tnh cht v bi tp


nh ngha . . . . . . . .
Cc tnh cht quen thuc .
Bi tp c bn . . . . . .

c bn
. . . . .
. . . . .
. . . . .

.
.
.
.

.
.
.
.

.
.
.
.

.
.
.
.

101
101
101
103

. . . . . . . . . . . . . . . . . . . .

105

Hm c cha phn nguyn . . . . . . . . . . . . . . . . . . . . . . . . . . . . . . . .

109

Hm phn nguyn trong vic tnh


nh ngha . . . . . . . . .
Tnh cht . . . . . . . . . .
Bi tp v d . . . . . . . .

.
.
.
.

115
115
115
116

Bi tp tng hp . . . . . . . . . . . . . . . . . . . . . . . . . . . . . . . . . . . . .

118

ng dng nh l Hermite v nh l Legendre


tng cc
. . . . .
. . . . .
. . . . .

.
.
.
.

.
.
.
.

.
.
.
.

.
.
.
.

.
.
.
.

ch s . .
. . . . . . .
. . . . . . .
. . . . . . .

.
.
.
.

.
.
.
.

.
.
.
.

.
.
.
.

.
.
.
.

.
.
.
.

.
.
.
.

.
.
.
.

.
.
.
.

.
.
.
.

.
.
.
.

.
.
.
.

.
.
.
.

.
.
.
.

.
.
.
.

.
.
.
.

.
.
.
.

.
.
.
.

.
.
.
.

.
.
.
.

.
.
.
.

.
.
.
.

.
.
.
.

.
.
.
.

.
.
.
.

.
.
.
.

.
.
.
.

.
.
.
.

.
.
.
.

.
.
.
.

Li ni u
S hc l b hong ca Ton hc
C th ni S hc l lnh vc xut hin sm nht trong lch s Ton hc. Khi con ngi bt
u lm vic vi nhng con s th khi y, S hc ra i.Tri qua hng nghn nm pht trin,
S hc vn gi c v p thun khit ca n. V p y c th hin qua cch pht biu
n gin ca bi ton, n ni mt hc sinh lp 6 cng c th hiu c. Th nhng, v p
y thng tim n nhng th thch su thm bn trong thch thc tr tu loi ngi. . .
Hy ni v nh l Fermat ln, mt nh l qu ni ting trong th gii Ton hc. Trong
mt tuyn tp vn hc vi ta Tho c vi Qu c truyn ngn Con Qu v Simon Flagg
ca Arthur Poges. Trong truyn ngn ny con Qu c ngh Simon Flagg t cho n mt cu
hi. Nu con Qu tr li c trong vng 24 gi, n s ly i linh hn ca Simon, cn nu n
u hng n s tr cho Simon 100.000 la. Simon t cho con Qu cu hi: nh l cui
cng ca Fermat c ng khng? Nghe xong, con Qu bin mt v bay vt i khp v tr
tip thu tt c tri thc ton hc tng c sng to ra. Ngy hm sau con Qu quay tr li
v th nhn: Simon, ngi thng, con Qu bun ru ni v nhn Si mon vi con mt y
thn phc. Ngay c ta, ta cng khng c kin thc ton hc trong mt thi gian ngn
nh th c th gii p c mt bi ton kh nh vy. Cng nghin cu su n cng rc ri
hn. . . Ch! Ngi c bit-Con Qu tm s- ngay c nhng nh ton hc gii nht trn cc
hnh tinh khc, h cn uyn bc hn nhng nh ton hc ca cc ngi nhiu, cng khng gii
ni cu khng? Th y, mt g trn sao Th nhn ging nh mt cy nm trn c kheo,
g c th gii nhm cc phng trnh vi phn o hm ring, m cng phi u hng thi.
Chnh v c cch pht biu n gin nhng cn nhng suy lun su sc v tinh t nn nhng
bi ton S hc trong cc k thi Olympic thng c dng phn loi hc sinh. Tuy rng
trn th trng c rt nhiu cun sch vit v S hc, nhng nhu cu sch v lnh vc ny
cha bao gi vi i. c bit, ngy cng nhiu cng phng php mi xut hin, cn mt u
sch khng nhng pht trin nhng phng php c m cn c th gii thiu nhng phng
php mi hoc nhng ci nhn mi v nhng vn c.
Chuyn S hc ca Din n Mathscope ra i chnh l p ng nhu cu ca ng
o hc sinh, sinh vin v gio vin trn khp c nc. Chuyn c thc hin bi cc thnh
vin ca Din n Mathscope, bao gm cc ch : Cp v cn nguyn thu, Cc bi ton s
hc hon v vng quanh, Dy s s hc, Hm s hc, B nng s m LTE, Phn nguyn,

6
Thng d chnh phng, Phng php bc nhy Viete. Hi vng y s l mt ti liu hu ch
cho cc bn c gn xa trong vic n luyn cho cc k thi Olympic.
hon thnh chuyn ny, ban bin tp xin gi li cm n su sc n tc gi ca cc
bi vit, cc thnh vin tham gia tho lun, ng gp trn Din n. c bit, xin gi li
cm n su sc n Ban qun tr ca Din n Mathscope lun to iu kin tt nht cho
ban bin tp hon thnh chuyn ny. Cun sch ny chnh l thnh qu qu trnh lao
ng nghim tc ca cc thnh vin trong ban bin tp, nhng hn ht y vn l mt sn
phm ng qu ca cng ng Mathscope ni ring v cng ng Ton hc Vit Nam ni chung.
Tuy c kim tra k cng nhng chuyn khng trnh khi nhng sai st. Mi gp v
chuyn xin c gi ln Din n Mathscope (mathscope.org) hoc gi v hp mail
phamtienkha@gmail.com. Xin chn thnh cm n.

Thnh ph H Ch Minh, ngy Ph n Vit Nam 20-10-2013


Thay mt Ban bin tp
Phm Tin Kha

Chuyn 1:
Bc nhy Viete
Phm Huy Hong

M u
Trong cc k thi hc sinh gii, cc bi ton v phng trnh Diophante bc hai khng cn
xa l. Phng trnh Pell l mt trong cc v d ni bt nht v phng trnh Diophante bc
hai, tuy nhin do lng bi ton v phng trnh Pell kh nhiu, nn trong k thi IMO 1988
xut hin mt dng bi phng trnh Diophante bc hai rt mi m thi by gi:
Cho a, b l hai s nguyn dng tha mn ab + 1|a2 + b2 . Chng minh rng
a2 + b2
ab + 1
l s chnh phng.
Bi ton ny c coi l kh nht trong cc k thi IMO trc nm 1988 v cng l bi ton
kh nht trong k thi ny. Tc gi Authur Engel tng bnh lun v bi ton ny (nguyn
vn):
" Nobody of the six members of the Australian problem committee could solve
it. Two of the members were George Szekeres and his wife, both famous problem
solvers and problem creators. Since it was a number theoretic problem it was sent
to the four most renowned Australian number theorists. They were asked to work
on it for six hours. None of them could solve it in this time. The problem committee
submitted it to the jury of the XXIX IMO marked with a double asterisk, which
meant a superhard problem, possibly too hard to pose. After a long discussion, the
jury finally had the courage to choose it as the last problem of the competition.
Eleven students gave perfect solutions."
Dch ra ting Vit nm na l:
"Khng c ai trong s su thnh vin ca hi ng gim kho ca c gii c
bi ton ny. Hai thnh vin ni bt trong , u l nhng ngi ni ting gii v
sng to cc bi ton, l George Szekeres v v ng. y l bi ton s hc nn n
c gi cho bn nh s hc ln nht ca c by gi. H c yu cu gii bi
ton trong su gi v khng c ai gii c sau . Hi ng thm nh np cho
1

i hc Khoa hc T Nhin

8
ban gim kho IMO XXIX bi ton ny vi hai du hoa th, ni ln l n rt
kh, hoc l qu kh ra trong k thi. Sau mt hi bn bc, hi ng IMO XXIX
quyt nh chn bi ton ny lm bi cui ca k thi. C mi mt hc sinh cho li
gii hon chnh ca bi ton."
Trong s 11 hc sinh c Gio s Ng Bo Chu ca chng ta.

Li gii nguyn thy ca bi ton v cc vn lin quan


Chng ta bt u vi bi ton gc:
Bi ton 1. Cho a, b l cc s nguyn dng tha mn ab + 1 | a2 + b2 .
Chng minh rng
a2 + b 2
ab + 1
l s chnh phng
Li gii. t k =
trnh

a2 +b2
.
ab+1

C nh k v xt tt c cc cp (a, b) nguyn dng tha mn phng


k=

a2 + b 2
,
ab + 1

c ngha l xt tp

S=

a2 + b2
(a, b) N N |k =
ab + 1

V S l tp cc cp s nguyn dng nn lun tn ti mt cp (a0 , b0 ) trong S m a0 + b0 t


gi tr nh nht v a0 > b0 .
Xt phng trnh
x2 + b20
= k x2 kx.b0 + b20 k = 0
xb0 + 1
l mt phng trnh bc hai n x. Ta bit rng phng trnh trn c mt nghim l a0 . Nh
vy theo nh l Viete th tn ti nghim a1 tha mn phng trnh bc hai vi n x trn v
a1 = kb0 a0 =

b20 k
.
a0

T y ta c a1 cng l s nguyn. Ta chng minh a1 khng m. Tht vy, nu a1 < 0 th


a21 kb0 a1 + b20 k > a21 + k + b20 k > 0,
mu thun. Do ta c a1 > 0. T y ta c:
Nu a1 > 0 th (a1 , b0 ) l mt cp thuc S. Theo nh ngha ca (a0 , b0 ) ta c:
a0 + b 0 6 a1 + b 0 a0 6 a1 .
Do cng theo Viete th:
a20 6 a0 a1 = b20 k < b20 a0 < b0 ,
tri vi gi thit ban u.
Do a1 = 0, v vy suy ra k = b20 l mt s chnh phng, ta c iu cn chng minh.

9
T bi ton ny ta c th thy c cc bc gii bi ton dng phng php ny nh sau:
1. Nhn dng bi ton thuc lp phng trnh Diophante bc hai (tr ln).
2. C nh mt gi tr nguyn m bi cho, ri gi s tn ti mt cp nghim tha mn
mt vi iu kin m khng lm mt tnh tng qut ca bi ton.
3. Da vo nh l Viete tm cc mi quan h v s mu thun, t tm c kt lun
ca bi ton.
im mu cht ca cc bi ton ny l nguyn l cc hn: Trong tp hp cc s nguyn dng
th lun tn ti s nguyn dng nh nht. Mnh trn khng nhng hu dng trong cc lp
bi ton ny m cn trong nhiu bi ton t hp, t hp s hc v s hc.
T nhng bi ton tip theo, ti s trnh by vn tt cc bc lm v cch lm thay v trnh
by y nh bi ton trn, cc bn c th t pht huy tnh t lm vic ca mnh. Phn
gi s c cui bi vit.
Bi ton 2. Tm tt c cc s nguyn dng n sao cho phng trnh sau c nghim nguyn
dng :
x2 + y 2 = n(x + 1)(y + 1).
Chng minh. Chng ta s lm theo cc bc nh bi ton trn:
1. C nh n, gi s tn ti cp (x0 , y0 ) m tng x0 + y0 min v x0 > y0 .
2. Xt phng trnh bc 2 n X nh sau:
X 2 X.n(y + 1) + y02 ny0 n = 0.
Phng trnh c nghim l x0 nn c nghim x1 .
3. p dng nh l Viete:
x0 + x1 = n(y0 + 1), x0 x1 = y02 ny0 n.
4. Tng t bi trc, cc bn chng minh x1 > 0 v t s chng minh x1 = 0 bng
cch chng minh x1 > 0 th s dn n mu thun.
5. T i n kt lun bi ton: x1 = 0 v y02 = n(y0 + 1) suy ra y0 + 1 | y02 , l iu khng
th xy ra khi y0 nguyn dng.
Do khng tn ti s nguyn dng n tha mn phng trnh u tin.
T bi ton ny, ta dn n c bi ton th v sau:
Bi ton 3. Gi s a, b nguyn dng tha mn:
b + 1 | a2 + 1, a + 1 | b2 + 1.
Chng minh rng a, b u l cc s l.

10
Chng minh. Nhn vo bi ton trn, t gi thit ta khng nhn thy mi tng quan gia a, b
v tnh chn l ca hai s . V vy, nh bn nng v kinh nghim, cch lm tt nht thm
d kin l s dng phng php phn chng.
Gi s a, b u l cc s chn. T gi s ny, bn c hy chng minh hai mu cht sau:
1. a + 1 v b + 1 nguyn t cng nhau.
2. a + 1 | a2 + b2 , b + 1 | a2 + b2 .
T suy ra tn ti n nguyn dng sao cho a2 + b2 = n(a + 1)(b + 1) v theo bi ton trn
ta c iu mu thun. V vy a, b l.
r
Bi ton trn cng l mt b quan trng ca mt bi ton trong IMO Shortlist 2009.
Bi ton 4. Tm tt c cc s nguyn dng n sao cho tn ti dy s nguyn dng a1 , a2 , ..., an
tha mn
a2 + 1
1
ak+1 = k
ak1 + 1
vi mi k tha mn 2 6 k 6 n 1.
(Phn gi s c cui bi vit).
Khi gii c hai bi ton trn th a s cc bi ton vi hai bin x, y s thnh chuyn "n
gin". Xin mi bn c th sc vi bi ton sau, v iu n sau mi l iu th v:
Bi ton 5. Tm tt c cc s n nguyn dng sao cho phng trnh sau c nghim nguyn
dng:
(x + y)2 = n(4xy + 1).
Chng minh. Chng ta tun t theo cc bc trn. p n l vi n l s chnh phng th
phng trnh lun c nghim nguyn dng.
r
ng ch l bi ton n gin nh vy m li l mt b cc k quan trng cho mt bi
ton kh sau:
Bi ton 6 (Taiwan MO 1998). Cho m, n l hai s l vi m > n > 1 tha mn
m2 n2 + 1 | n2 1.
Chng minh rng m2 n2 + 1 l s chnh phng.
Chng minh. Nhn vo bi ton ny v bi ton trn, chng ta khng th thy ngay s lin h.
Gi cho bi ton ny l lm th no chuyn v bi ton trc.
T gi thit ta c m2 n2 + 1 | n2 1, cho tin v gn hn, ta c m2 n2 + 1 | m2 . T y
ta c th t m2 = k.(m2 n2 + 1) vi k nguyn dng. n y th chc khng kh nhn ra
mi lin h: T gi thit m, n l, tn ti hai s nguyn dng a, b sao cho m = a + b, n = a b.
Do phng trnh trn tr thnh:
(a + b)2 = k(4ab + 1),
chng ta quay v bi ton trn. Vy k l s chnh phng, do 4ab + 1 cng l s chnh
phng hay m2 n2 + 1 l s chnh phng.
r

11
Nhn bi ton trn, nh mt ngi lm ton, chng ta khng khi thc mc l: liu c tn ti
hai s m, n nh vy khng tha mn m2 n2 + 1|n2 1 ri suy ra m2 n2 + 1 l s chnh
phng? Bng li suy ngh chng ta nn tm th mt nghim ca bi ton:
Bi ton 7. Tm mt cp nghim (m, n) l nguyn dng tha mn iu kin bi ton trn.
Chng minh. Th mt vi gi tr, bi ton khng h d nh cc bn tng: Chng ta khng
th "m" nghim ri suy ra c. Chng ta nn bt u vi cch lm t nhin nht: t
m2 = k(m2 n2 + 1). V bi ton ch yu cu mt nghim, ta bt u vi k = 1 l s chnh
phng u tin:
m2 = m2 n2 + 1 n = 1,
khng tha mn v m > n > 1.
Tip tc vi k = 4 l s chnh phng tip theo:
m2 = 4(m2 n2 + 1) 4n2 3m2 = 4.
T phng trnh trn suy ra 2 | m hay t m = 2t. T phng trnh tng ng vi
n2 3t2 = 1,
tr v phng trnh Pell quen thuc v phng trnh ny chc chn c nghim v 3 khng
phi l s chnh phng. Tm nghim ca phng trnh ny khng h kh, cc bn c th t
tm bng cch s dng cng thc nghim tng qut ca phng trnh Pell hoc h phng
trnh.
r
Vy tt c cc bi ton trn u c th tm c nghim tha mn. Bi ton va xong ch l
mt nghim n gin. Cu hi l c th tm c nghim tng qut khng? Cu tr li l c.
Trong k thi chn hc sinh gii Ton quc gia nm 2012, bi ton cng s dng phng php
bc nhy Viete gii c bi ton. Xin trch dn bi, bi gii trong ti liu "Nhn xt
v nh gi thi VMO 2012" ca Thy Trn Nam Dng:
Bi ton 8. Xt cc s t nhin l a, b m a l c s ca b2 + 2 v b l c s ca a2 + 2.
Chng minh rng a v b l cc s hng ca dy s t nhin (vn ) xc nh bi
v1 = v2 = 1; vn = 4vn1 vn2 , n > 2.
Chng minh. Gi s (a, b) l cp s t nhin l m a l c s ca b2 + 2 v b l c s ca
a2 + 2. Trc ht ta chng minh (a, b) = 1. Tht vy, t d = (a, b) th do d | a | b2 + 2 nn
d | 2. M a, b l nn d l, suy ra d = 1.
Xt s N = a2 + b2 + 2 th do a2 + 2 chia ht cho b nn N chia ht cho b. Tng t, N chia ht
cho a. V (a, b) = 1 nn t y suy ra N chia ht cho ab. Vy tn ti s nguyn dng k sao
cho
a2 + b2 + 2 = kab
(1)
Tip theo, ta chng minh k = 4. Tht vy, t A = {a + b|(a, b) N N , a2 + b2 + 2 = kab}.
Theo gi s trn th A 6= . Do tnh sp th t tt ca N, A c phn t nh nht. Gi s

12
a0 , b0 l cp s tha mn iu kin (1) vi a0 + b0 nh nht.
Khng mt tnh tng qut, c th gi s a0 > b0 . Xt phng trnh a2 kb0 a + b20 + 2 = 0 c
b2 +2
nghim a0 . Theo nh l Viete th phng trnh trn cn c 1 nghim na l a1 = kb0 a0 = 02 .
Theo cng thc nghim th r rng a1 nguyn dng. Nh vy (a1 , b0 ) cng l mt nghim ca
(1). Do tnh nh nht ca a0 + b0 , ta c a0 + b0 6 a1 + b0 , tc l a0 6 kb0 a0 suy ra ab00 6 k2 .
Ta c a20 + b20 + 2 = ka0 b0 suy ra
a0 b 0
2
+
+
=k
(2)
b0 a0 a0 b0
Do ab00 6 k2 nn t (2) ta c k 6 k2 + 2 + 1 hay k 6 6.
Mt khc, p dng bt ng thc AM-GM ta c a20 + b20 > 2a0 b0 nn k > 2.
Nu k 6= 4 th (a0 , b0 ) 6= (1, 1), do a0 b0 > 2. Dng (2) nh gi ta c k 6 k2 + 1 + 1 nn
k 6 4. Vy cc gi tr k = 5, 6 b loi. Nu k = 3 th do a20 + b20 + 2 = 3a0 b0 nn suy ra a20 + b20 + 2
chia ht cho 3, suy ra mt trong hai s a0 , b0 chia ht cho 3, s cn li khng chia ht cho 3.
Nu b0 = 1 th a0 chia ht cho 3, khi v tri khng chia ht cho 9 cn v phi chia ht cho
9, mu thun. Vy b0 > 1. T suy ra a0 b0 > 6. Li s dng (2) nh gi, ta suy ra
k6

2
8
k
+1+ k < .
2
6
3

M k N nn k 6 2, mu thun.
Nh vy ta chng minh c nu a, b l cc s t nhin l tha mn iu kin bi th
a2 + b2 + 2 = 4ab

(3)

Ta s chng minh trong trng hp nh vy th tn ti s nguyn dng n sao cho (a, b) =


(vn , vn+1 ) vi vn l dy s c nh ngha bi.
Trc ht, ta c nhn xt : Nu a, b l nghim ca (3) th (4a b, a) v (4b a, b) cng l
nghim ca (3). T , do (v1 , v2 ) l nghim ca (3) nn (4v2 v1 , v2 ) cng l nghim ca (3),
tc l (v2 , v3 ) cng l nghim ca (3). T y bng quy np suy ra (vn , vn+1 ) l nghim ca
(3).
Gi s tn ti cp s (a, b) tha mn (3) nhng khng tn ti n sao cho (a, b) = (vn , vn+1 ).
Trong cc cp s nh th, chn (a, b) c tng a + b nh nht. Khng mt tnh tng qut, gi
s a > b (ch a khng th bng b v nu a = b suy ra a = b = 1, khi (a, b) = (v1 , v2 ).
2
Theo nhn xt trn th 4b a, b cng l nghim ca (3). Nhng do 4b a = b a+2 < a (V a > b
nn ab b2 = (a + b)(a b) > 3)), nn 4b a + b < a + b. Theo nh ngha ca (a, b) trn,
2
phi tn ti n sao cho (4b a, b) = (vn , vn+1 ). S dng ng thc 4b a = b a+2 v b > 1, ta
suy ra 4b a 6 b. Nh vy 4b a = vn , b = vn+1 . Nhng t y a = 4vn+1 vn = vn+2 , tc
l (a, b) = (vn+1 , vn+2 ) mu thun. Vy iu gi s l sai, tc l phi tn ti s t nhin n sao
cho (a, b) = (vn , vn+1 ) v nh vy a, b l s hng ca dy (vn ). Bi ton c gii quyt hon
ton.
r
Nhn xt:
Bi ton ny c 2 chnh:

13
1. Chng minh rng nu k l s nguyn dng sao cho tn ti a, b nguyn dng tha
mn iu kin a2 + b2 + 2 = kab th k = 4. Phn ny kh quen thuc vi cc bn
bit v phng php phng trnh Markov hay bc nhy Viete.
2. M t tt c cc nghim ca phng trnh a2 + b2 + 2 = 4ab thng qua cp s hng
lin tip ca dy vn , ci ny gi l phng php gien.
Phng trnh (3) cn c th gii thng qua phng trnh Pell z 2 3b2 = 2. Tuy nhin
cch gii ny s kh cng knh v y khng phi l phng trnh Pell loi 1.
Nh vy qua nhn xt ca bi ton trn, thy Trn Nam Dng tng kt li cc bc lm
chnh: l s dng bc nhy Viete tm c k v s dng phng php Gien, phng
php li v hn tm c im c bit ca nghim, l nu (a, b) l nghim th (4b a, b)
cng l nghim, dn n tm c nghim tng qut ca phng trnh trn.
Ngoi ra thy cng cp ti phng trnh Markov, l mt phng trnh rt ni ting, c
th ni l "thy t" ca phng php bc nhy Viete:
Bi ton 9 (Phng trnh Markov). Tm tt c cc s nguyn dng k sao cho phng trnh
sau c nghim nguyn dng:
x2 + y 2 + z 2 = kxyz.
Chng minh. Chng ta va gii quyt xong cc lp bi ton vi hai n, vy bi ton ba n th
li gii c khc khng? V khc nh th no? Liu ta c th tm c cch gii tng qut cho
bi ton vi ba n nh vi hai n khng?
Cch gii bi ton ny ph thuc vo cch gii bi ton i vi hai s, trc ht ta cn b
sau:
B 1. k = 3 l s nguyn dng duy nht phng trnh sau lun c nghim nguyn
dng (x, y):
x2 + y 2 + 1 = kxy.
B trn l mt bi ton s dng bc nhy Viete quen thuc, mi bn c t gii.
Tr li bi ton:

1. Thy k = 1 th phng trnh c nghim (3, 3, 3) v k = 3 th phng trnh c nghim


(1, 1, 1).
2. Xt k 6= 1, 3. Gi s phng trnh c nghim (x0 , y0 , z0 ). Khng mt tnh tng qut, gi
s x0 6 y0 6 z0 v x0 + y0 + z0 nh nht trong tt c cc tng x + y + z vi x, y, z l
nghim ca phng trnh.
Nu y0 < z0 , xt phng trnh bc hai:
Z 2 k.x0 y0 .Z + x20 + y02 = 0.
Phng trnh trn c mt nghim l z0 . Theo nh l Viete th c nghim th hai
z1 tha mn:
x2 + y02
.
z1 = kx0 y0 z0 = 0
z0

14
T suy ra z1 nguyn dng v (x0 , y0 , z1 ) l nghim th hai. Do theo gi thit
cc hn ta c:
x0 + y0 + z0 6 x0 + y0 + z1 z0 6 z1 ,
dn n
x20 + y02 kx0 y0 = z1 z0 z1 z0 = (z1 1)(z0 1) 1 > y02 1,
v suy ra
1 > x0 (ky0 x0 ) > x0 (kx0 x0 ) > x0 ,
m x0 nguyn dng suy ra x0 = 1. Ta tr v bi ton y 2 + z 2 + 1 = kyz, chnh l
b suy ra k = 3, tri gi thit.
Nu y0 = z0 . Ta c
2y02 py02 + x20 = 0 x20 = y02 (px0 2) > x20 (px0 2),
v t dn n 3 > px0 , m px0 > 2 nn px0 = 3 suy ra p {1, 3}, tri gi thit
Vy k {1, 3} th phng trnh lun c nghim nguyn dng.

Trn y l mt s bi ton m ti mun gii thiu vi bn c, l cc bi ton kh quen


thuc v c lp li trong nhiu k thi. Ti hi vng qua bi vit ny bn c c th nm bt
phng php gii mt lp cc bi ton v phng trnh bc hai Diophante nhiu n.
Sau y l gi cho cc bi ton v mt s cc bi ton th sc.

Gi cho mt s bi ton
2. Chng minh x1 > 0:
x21 x1 n(y0 + 1) n(y0 + 1) + y02 = 0 x1 =

x20 + y02
1 > 1,
n(y0 + 1)

m x1 nguyn nn x1 > 0.
3. Vi a, b chn: c b + 1 | b2 1, b + 1 | a2 + 1 nn b + 1 | a2 + b2 . Tng t a + 1 | a2 + b2 .
Gi d = (a + 1, b + 1, hy chng minh d | 2 m d l do a + 1, b + 1 l nn d = 1, t suy
ra a2 + b2 = k(a + 1)(b + 1).
4. n = 1, 2, 3, 4. D dng ch ra dy vi n = 1, 2, 3. Dy c di 4: 4,33,27,1384.
Phn chng tn ti dy di 5: a1 , a2 , a3 , a4 , a5 . T y chng minh hai mnh sau:
(a) a2 , a3 chn (s dng phn chng)
(b) a2 + 1 | a23 + 1, a3 + 1 | a22 + 1.
5. Chng minh x1 > 0: t phng trnh ta c
4x1 y0 =
m x1 nguyn nn x1 > 0.

(x1 + y0 )2
1
1 > 1 x1 >
n
4y0

15

Cc bi ton th sc
Bi ton 10. Chng minh rng nu a, b l cc s nguyn dng sao cho k =
nguyn th k = 8.
Bi ton 11.

a2 +b2 +6
ab

l s

1. Tm n sao cho phng trnh sau c nghm nguyn dng:


(x + y + z)2 = nxyz

2. Tm n sao cho phng trnh sau c nghm nguyn dng:


(x + y + z + t)2 = nxyzt
Bi ton 12 (M rng phng trnh Markov). Cho a, b, c l ba s nguyn tha mn
a2 + b2 + c2 = kabc.
Chng minh rng hoc (a, b, c) = 1 hoc (a, b, c) = 3.
Bi ton 13. Chng minh rng phng trnh
x2 + y 2 + z 2 = n(xyz + 1)
c nghim nguyn dng khi v ch khi n c biu din di dng tng ca hai s chnh
phng.
Bi ton 14 (Adapted from Vietnam TST 1992). Tm tt c cc cp s nguyn dng (a, b)
tha mn
a2 + b2 = k(ab 1).
Bi ton 15 (Turkey TST 1994). Tm tt c cc cp (a, b) m ab | a2 + b2 + 3.
Bi ton 16. Cho a, b, c l ba s nguyn dng tha mn
0 < a2 + b2 abc 6 c,
chng minh rng a2 + b2 abc l s chnh phng.
Bi ton 17. Chng minh rng tn ti v hn cc cp (m, n) nguyn dng tha mn
m+1 n+1
+
= 4.
n
m
Bi ton 18 (IMO Shortlist 2003). Tm tt c cc cp a, b tha mn
a2
2ab2 b3 + 1
Bi ton 19. Chng minh rng tt c nghim nguyn dng ca phng trnh x2 +y 2 +1 = 3xy
l (x, y) = (F2k1 , F2k+1 ) vi Fn l s Fibonacci.

16
Bi ton 20 (IMO 2007, IMO shortlist). Cho a, b nguyn dng. Chng minh rng nu
4ab 1 | (4a2 1)2 , th a = b.
Gi . Dng phn chng, gi s a > b vi mi a, b tha mn. T gi thit, hy chng minh:
1. 4ab 1 | (a b)2
2. a0 b0 > (a0 + b0 )(4a0 b0 1) vi a0 , b0 l nghim nh nht theo ngha a0 + b0 min, t
suy ra mu thun
r
Bi ton 21 (**, Kiran Kedlaya). Chng minh rng (xy + 1)(yz + 1)(zx + 1) l s chnh
phng khi v ch khi xy + 1, yz + 1, zx + 1 l s chnh phng.
Gi .
1. Nu xy + 1, yz + 1, zx + 1 l s chnh phng th hin nhin ta c tch ba s
chnh phng.
2. Nu (xy + 1)(yz + 1)(zx + 1) l s chnh phng. Hy chng minh tn ti t tha mn h
sau:
(x + y z t)2 = 4(xy + 1)(zt + 1)
(x + z y t)2 = 4(xz + 1)(yt + 1)
(x + t y z)2 = 4(xt + 1)(yz + 1)
(t chnh l nghim ca phng trnh t2 + x2 + y 2 + z 2 2(xy + yz + zt + tx + zx + ty)
4xyzt 4 = 0). Xt nghim t nh nht.
S dng phn chng: gi s xy + 1 khng chnh phng. T y, hy chng minh:
(a) t >

1
max{x,y,z}

> 1 nn t > 0.

(b) Xt hai trng hp t = 0 v t > 0, dn n mu thun.


r

Ti liu tham kho


1. ng Hng Thng, Nguyn Vn Ngc, V Kim Thy, Bi ging s hc. NXB Gio dc,
1996.
2. Kiran S. Kedlaya, When Is (xy + 1)(yz + 1)(zx + 1) a Square. Mathematics Magazine,
Vol. 17, No.1, Feb., 1998.
3. Authur Engel, Problem Solving Strategies. Spinger Verlag, 1998.
4. Trn Nam Dng, Li gii v bnh lun VMO 2012. Din n Mathscope, 2012.
5. Site: http://www.artofproblemsolving.com/Forum

Chuyn 2:
Vn dng phng php LTE
vo gii cc bi ton s hc
Phm Quang Ton

B v s m ng (Lifting The Exponent Lemma) l mt b rt hu dng


trong vic gii cc bi ton s hc v rt c bit n trong lch s Olympiad.
Thc cht l n c m rng ra t b Hensel. Ta thng vit tt tn ca b
l LTE, tn Ting Vit th c th gi l b v s m ng. Bi vit ny xin
c gii thiu vi bn c v b v nhng ng dng c sc ca n vo cc bi
ton l thuyt s.
Bi vit ch yu da vo ti liu ca thnh vin Amir Hossein bn trang mathlinks.ro
(v mt l thuyt th mnh gi nguyn bn bi vit ca Amir Hossein sang bi vit
ny) v c km theo mt s v d c ly t cc k thi Olympic ton trn th gii.

Mt s khi nim
.
.
y, thay v k hiu a..b ngha l a chia ht cho b, ta s k hiu b|a. V a 6 ..b s c thay bng
b - a.
nh ngha 1. Cho p l s nguyn t, a l s nguyn v l s t nhin. Ta c p l ly tha
ng (exact power) ca a v l s m ng (exact exponent) ca p trong khai trin ca
nu p |a v p+1 - a. Khi ta vit p k a hay vp (a) = .
V d. Ta c v5 (5400) = 3 hay 53 k 5400 v 5400 = 53 32 22 .
Sau y l mt s tnh cht. Chng minh tnh cht ny khng kh, xin dnh cho bn c.
Tnh cht 1. Cho a, b, c l cc s nguyn. Ta c
1. vp (ab) = vp (a) + vp (b)
2. vp (an ) = n vp (a)
3. min{vp (a), vp (b)} 6 vp (a + b)
Du ng thc xy ra khi vp (a) 6= vp (b).
4. vp (gcd(|a|, |b|, |c|)) = min{vp (a), vp (b), vp (c)}
5. vp (lcm(|a|, |b|, |c|)) = max{vp (a), vp (b), vp (c)}
Ch . vp (0) = vi mi s nguyn t p.
1

Lp 9C THCS ng Thai Mai, Tp Vinh

17

18

Hai b
u tin, xin gii thiu vi bn c hai b . V hai b ny s gip ta tm cch chng
minh c cc nh l khc ca LTE.
B 1. Cho x, y l hai s nguyn v cho n l s nguyn dng. Cho s nguyn t p bt k
sao cho p|x y v p - x, p - y. Ta c
vp (xn y n ) = vp (x y).
Chng minh. Ta c p|x y nn
xn1 + xn2 y + + xy n2 + y n1 nxn1 6 0

(mod p)

M xn y n = (x y) (xn1 + xn2 y + + xy n2 + y n1 ) nn ta suy ra iu phi chng


minh.
r
B 2. Cho x, y l hai s nguyn v n l s nguyn dng l. Cho s nguyn t p bt k tha
mn p|x + y v p - x, p - y. Khi
vp (xn + y n ) = xp (x + y).
Chng minh. p dng b 1 ta c vp (xn (y)n ) = vp (x (y)) nn vp (xn + y n ) = vp (x + y).
(v n l). B c chng minh.
r

Lifting The Exponent Lemma (LTE)


nh l 1. Cho x v y l cc s nguyn (khng nht thit phi nguyn dng), n l mt s
nguyn dng v p l mt s nguyn t l tha mn p|x y v p - x, p - y. Ta c
vp (an bn ) = vp (a b) + vp (n)
Chng minh. Ta s i chng minh quy np theo vp (n). Trc ht, ta s i chng minh khng
nh sau:
vp (xp y p ) = vp (x y) + 1
chng minh iu th ta cn ch ra rng
p|xp1 + xp2 y + + xy p2 + y p1

(1)

p2 - xp1 + xp2 y + + xy p2 + y p1

(2)

Vi (1), nh p dng x y (mod p) ta suy ra


xp1 + + y p1 pxp1 0

(mod p)

19
Vi (2), ta t y = x + kp vi k N . Khi vi 1 6 i 6 p 1 (i N) th
y i xp1i (x + kp)i xp1i


i(i 1)
i
i1
p1i
2 i2
x + i(kp)x +
x
(kp) x +
2

xp1i xi + i(kp)xi1
xp1 + ikpxp2

(mod p2 ).

Do ,
xp1 + xp2 y + + y p1 xp1 + (xp1 + kpxp2 ) + (xp1 + 2kpxp2 ) + + (xp1 + (p 1)kpxp2 )
p1
pxp1 +
kp2 xp2
2
pxp1 6 0 (mod p2 )
Nh vy vp (xp y p ) = vP (x y) + 1.
Quay li bi ton, t n = pk h vi b, k N, b > 1 gcd(b, p) = 1. Khi th
k

vp (an bn ) = vp ((ap )h (bp )h )


 k

k
k1
k1
= vp ap bp = vp ((ap )p (bp )p )
= vp (ap
..
.

k1

bp

k1

k2

) + 1 = vp ((ap

)p (bp

k2

)p ))

= vp (x y) + k = vp (x y) + vp (n)
r

nh l c chng minh.

nh l 2. Cho hai s nguyn x, y, n l s nguyn dng l, v p l c nguyn t l sao cho


p|x + y v p - x, p - y. Khi
vp (xn + y n ) = vp (x + y) + vp (n).
Chng minh. p dng nh l 1 ta c
vp (xn (y)n ) = vp (x (y)) + vp (n)
hay
vp (xn + y n ) = vp (x + y) + vp (n)
r
nh l 3. (cho trng hp p = 2) Cho x, y l hai s nguyn l tha mn 4|x y. Khi
v2 (xn y n ) = v2 (x y) + v2 (n).
Chng minh. Theo b 1 th nu p nguyn t, gcd(p, n) = 1, p|x y v p - x, p - y th
vp (xn y n ) = vp (x y)

20
Do ta ch cn xt ti trng hp n l ly tha ca 2, tc cn chng minh
n

v2 (x2 y 2 ) = v2 (x y) + n
Tht vy, ta c
n

x2 y 2 = (x2

n1

+ y2
k

n1

)(x2

n2

n2

+ y2

) (x2 + y 2 )(x + y)(x y)

V x y 1 (mod 4) nn x2 y 2 1 (mod 4). Do


n1

v2 (x2
n

n1

+ y2

n2

n2

) = v2 (x2

+ y2

) = = v2 (x + y) = 1

Nh vy v2 (x2 + y 2 ) = n + vp (x y), ta c iu phi chng minh.

nh l 4. (cho trng hp p = 2) Cho hai s nguyn l x, y, n l s nguyn dng chn v


2|x y. Khi
v2 (xn y n ) = v2 (x y) + v2 (x + y) + v2 (n) 1.
Chng minh. Ta c 4|x2 y 2 nn t n = 2k h vi k, h N , gcd(h, 2) = 1. Khi ta c
k

v2 (xn y n ) = v2 (xh2 y h2 )
k1

= v2 ((x2 )2
..
.

k1

(y 2 )2

= v2 (x2 y 2 ) + k 1
= v2 (x y) + v2 (x + y) + v2 (n) 1
r
Ta c h qu sau:
H qu. Cho a, n l hai s nguyn dng:
i) p l hai s nguyn t l sao cho vp (a 1) = N , khi vi mi s t nhin ta c
v( an 1) = + vp (n) = .
ii) n chn sao cho v2 (a2 1) = N , khi vi mi s nguyn dng th v2 (an 1) =
+ v2 (n) = + 1.
Ch .
a) Nu trong cc bi ton i hi vn dng phng php LTE, ta nn ti cc iu kin
t ra ca n, x, y, la chn nh l ph hp a vo li gii bi ton.
b) Nu d liu bi ton cho a|b vi a, b N th vi mi p l c nguyn t ca b, ta lun c
vp (b) > vp (a). Ngc li, nu vp (b) > vp (a) th a|b. Nh vy
a|b vp (b) > vp (a)
y l mt tnh cht rt thng c dng trong cc bi ton s dng phng php
LTE.

21

Mt s v d
Sau y mnh xin a ra mt s v d v cc ng dng ca phng php ny.
V d . Tm s nguyn dng n nh nht tha mn 22013 |1999n 1.
Li gii. p dng nh l 4 ta c
v2 (1999n 1) = v2 (n) + v2 (2000) + v2 (1998) = v2 (n) + 5
tha mn 22013 |1999n 1 th v2 (n) + 5 > 2013 hay v2 (n) > 2008.
Vy s nguyn dng n nh nht tha mn ra l 22008 .
V d . (IMO Shortlist 1991) Tm s nguyn dng k ln nht tha mn 1991k l c ca
1992

19901991

1990

+ 19921991

Li gii. t a = 1991 th a l s nguyn t l. Do theo nh l 2 th






a+1
a1
2
a1
a1
va (a 1)a + (a + 1)a
= va (a 1)a )a + (a + 1)a


2
= va (a 1)a + a + 1 + va (aa1 )


2
= a 1 + va (a 1)a + a + 1




2
2
Cng theo nh l 2 th va (a 1)a + 1 = va (a)+va (a2 ) = 3 nn va (a 1)a + a + 1 = 1.


aa+1
aa1
= a. Ta thu c max k = a = 1991 .
Vy, va (a 1)
+ (a + 1)
V d 1. (Italy TST 2003) Tm b s nguyn nguyn (a, b, p) sao cho a, b l s nguyn dng,
p l s nguyn t tha mn 2a + pb = 19a .
Li gii. V a nguyn dng nn 17|19a 2a . Vy p = 17. p dng nh l 1 ta c
v17 (19a 2a ) = v17 (17) + v17 (a)
b = 1 + v17 (a) 6 1 + a
1. Nu b < 1 + a hay 1 6 b 6 a. D dng chng minh quy np rng 19a 2a > 17a vi
a > 1. M 17a > 17b . Vy a = b = 1 trng hp ny.
2. Nu b = 1 + a th d dng chng minh quy np 19a 2a < 17a+1 = 17b , mu thun.
Vy (a, b, p) = (1, 1, 17) l p n duy nht bi ton.
V d . (IMO 1990) Tm s nguyn dng n sao cho n2 |2n + 1.
Li gii. Vi n = 1 tha mn. Vi n > 2, nhn thy n l.
Gi p l c nguyn t l nh nht ca n. Khi ta suy ra 22n 1 (mod p). Gi k l s nguyn
dng nh nht tha mn 2k 1 (mod p). Khi k|2n. Theo nh l Fermat nh th 2p1 1
(mod p) nn k|p 1. Nh vy ta suy ra gcd(n, k) = 1 nn k|2. Vi k = 1 th p|1, mu thun.
Vy k = 2. Do p = 3 hay 3|n.
t v3 (n) = k (k N ). p dng nh l 2 th ta c
v3 (2n + 1) = v3 (3) + v3 (n) = 1 + k

22
Li c v n2 |2n + 1 nn v3 (2n + 1) > v3 (n2 ) k + 1 > 2k. Vy k = 1. t n = 3m vi m N
v gcd(m, 3) = 1.
Gi p1 l c nguyn t nh nht ca m. Khi ta c 26m 1 (mod p1 ). Gi k1 l s nguyn
dng nh nht tha mn 2k 1 (mod p1 ). Tng t th ta d dng suy ra k|6. V p1 > 5 nn
k = 3 hoc k = 6.
Vi k = 3 th p1 |7 nn p1 = 7. Vi k = 6 th p1 |63 m p1 > 5 nn p1 = 7. Tuy nhin
2n + 1 = 23m + 1 = 8m + 1 2 (mod 7) m 7|n2 , mu thun.
Vy c nguyn t duy nht ca n l 3 m 3 k n nn n = 3.
S nguyn dng n tha mn bi l n {1; 3}.
V d 2. (European Mathematical Cup 2012, Senior Division) Tm s nguyn dng a, b, n v
s nguyn t p tha mn
a2013 + b2013 = pn
Li gii. t a = px y, b = pz t vi x, y, z, t N; t, y > 1 v gcd(y, p) = 1, gcd(t, p) = 1.
Khng lm mt tnh tng qut, gi s rng x > z. D nhn thy rng n > 2013x > 2013z. Khi
phng trnh ban u tng ng vi
t2013 + p2013(xz) y 2013 = pn2013z
Nu x > z th p - V T . Do p - pn2013z suy ra n = 2013z. Vy ta c phng trnh
t2013 + p2013(xz) y 2013 = 1,
mu thun v V T > 2 (do , ty > 1). Vy x = z. Phng trnh tr thnh
t2013 + y 2013 = pn2013z = pk (k = n 2013z N )

(3)

Nu p|2013 th theo nh l Fermat nh ta suy ra t2013 + y 2013 2 (mod p), mu thun v p|pk .
Vy gcd(p, 2013) = 1.
D thy theo (3) th p|t + y. Do bng vic p dng nh l 2 ta c

vp t2013 + y 2013 = vp (t + y)
Ta li c t + y|t2013 + y 2013 v (3) nn ta suy ra
pk = t + y = t2013 + y 2013
t2012 (t 1) + y 2012 (y 1) = 0
V t, y > 1 nn t phng trnh ta suy ra t = y = 1. Do p = 2, t suy ra a = b = 2h , n =
2013h + 1 vi h N.
Nhn xt. Ta c th tng qut bi ton ln thnh: Gii phng trnh nghim nguyn dng
an + b n = p k
vi p nguyn t.
V d 3. (Romanian IMO TST 2005) Gii phng trnh nghim nguyn dng
3x = 2x y + 1
Li gii. Ta xt hai trng hp:

23
1. Nu x l th p dng nh l 1 ta c v2 (3x 1) = v2 (3 1) = 1 hay v2 (2x y) = 1. Do
x = 1. T phng trnh ta suy ra y = 1.
2. Nu x chn th p dng nh l 1 ta c
v2 (3x 1) = v2 (3 1) + v2 (3 + 1) + v2 (x) 1 = 2 + v2 (x)
v2 (2x y) = 2 + v2 (x) x + v2 (y) = v2 (x) + 2

(1)

t x = 2m k vi m, n N . Ta d dng chng minh bng quy np rng 2m k > m + 2


vi m N, m > 3. Do x > v2 (x) + 2 vi v2 (x) > 3 hay vi x > 2v2 (x) = 8. Nh vy
x > 8 th (1) khng xy ra. Vy x 6 8, x chn nn x {2; 4; 6}. T y ta tm c
(x, y) = (2; 2), (4; 5).
Vy phng trnh c nghim nguyn dng (x, y) = (1; 1), (2; 2), (4; 5).
Nhn xt. Qua bi ton trn, ta lu mt s tng c dng trong phng php ny: Vi
p l mt c nguyn t ca a = pm k vi m, k N th:
i) a > pvp (a) .
ii) pm k > m + vi m > . T y suy ra a > vp (a) + vi vp (a) > hay a > p .
Cc bi trn ch yu l cc bi khng kh vn dng b LTE v ta xc nh c cc
yu t p, a, b mt cch d dng. Tuy nhin, vn c mt s bi ton i hi ta phi i tm ra
cc yu t p, a, b ...
V d 4. (IMO 1999) Tm tt c cc cp (n, p) nguyn dng sao cho p l s nguyn t v
(p 1)n + 1 chia ht cho np1 .
Li gii. D thy vi n = 1 th p l s nguyn t bt k u tha mn ra. Vi n > 2, ta c
cc trng hp:
Trng hp 1. Nu p = 2 th n|2. Do n = 2.
Trng hp 2. Nu p l. Ly q l c nguyn t nh nht ca n, khi (p 1)n 1
(mod q) hay (p 1)2n 1 (mod q) v gcd(p 1, q) = 1. Ta ly o l s nguyn dng nh nht
tha mn (p 1)o 1 (mod q). Khi th ta suy ra o|2n. p dng nh l Fermat nh ta c
(p 1)q1 1 (mod q). Do o|q 1.
Nh vy, o|2n v o|q 1. Nu gcd(o, n) > 1 hay o, n chia ht cho s nguyn t r, khi ta suy
ra r|n v r 6 o. M o|q 1 nn o < q, do r < q. M r v q u l c nguyn t ca n, mu
thun vi iu kin nh nht ca q. Vy gcd(n, o) = 1. Do 2|o. Vy (p 1)2 1 (mod q)
hay q|p(p 2).
1. Nu q|p 2 th ta c (p 1)n + 1 1n + 1 2 (mod q). Vy q = 2. Ta c (p 1)n + 1
chia ht cho 2 nn p = 2, mu thun v p l.
2. Nu q|p. D nhn thy n phi l (v nu n chn th (p 1)n + 1 0 (mod 4), mu thun
v p l). Ta p dng nh l 2 ta c
vq ((p 1)n + 1) = vq (n) + vq (p) > vq (n) (p 1)

(4)

24
t p = q a b vi a, b N . D dng chng minh bng quy np q a b > a + 2 (ch v
q|p nn q > 3), du bng xy ra khi a = b = 1, q = 3. Do p > vq (p) + 2. Kt hp vi
(3) ta suy ra
p 2 > vq (p) > vq (n)(p 2)
Vy q = p = 3 v v3 (n) = 1. t n = 3k vi k N , gcd(k, 3) = 1, gcd(k, 2) = 1. Nh
vy t bi ta s c 9k 2 |8k + 1.
Hin nhin 9|8k + 1. Ta ch cn i tm k sao cho k 2 |8k + 1. Vi k = 1 th n = 3, tha mn.
Vi k > 2, hon ton tng t, ly r l c nguyn t nh nht ca k v s l s nguyn
dng nh nht sao cho 8s 1 (mod r). Ta suy ra s|2 nn s = 2. Khi r|82 1 hay
r|7, iu ny mu thun v 8k + 1 2 (mod 7).
Vy, cp s (n, p) tha mn bi l (1, p), (2, 2), (3, 3).
V d 5. (Brazil XII Olympic Revenge 2013) Tm cc b ba s (p, n, k) nguyn dng tha
mn p l s nguyn t Fermat v
pn + n = (n + 1)k
(5)
x

S nguyn t Fermat l s nguyn t c dng 22 + 1 vi x t nhin.


Li gii. t = 2x . Nu n = 1 th (5) p = 2k 1 = 2 + 1. Do k = 2, = 1 nn p = 3.
Nu n > 2. Ta gi r l mt c nguyn t ca n. T phng trnh ta suy ra pn 1 (mod n)
hay pn 1 (mod r). Do gcd(p, r) = 1. t k l s nguyn dng nh nht tha mn pk 1
(mod r). Ta cng c theo nh l Fermat nh th pr1 1 (mod r). Vy ta suy ra k|r 1 v
k|n. V gcd(r 1, n) = 1 nn k = 1. Ta c r|p 1 hay r|2 . Vy r = 2 hay 2|n. Ta c


(5) pn 1 = (n + 1) (n + 1)k1 1

T phng trnh dn n v2 (pn 1) = v2 (n + 1)k1 1 .
Nu k 1 l th


v2 (n + 1)k1 1 = v2 (n) < v2 p2 1 + v2 (n) 1 = v2 (pn 1),
mu thun. Vy k 1 chn. p dng nh l 4 ta c

v2 (pn 1) = v2 (n + 1)k1 1
v2 (p2 1) + v2 (n) 1 = v2 (n) + v2 (n + 2) + v2 (k 1) 1
v2 (p 1) + v2 (p + 1) = v2 (n + 2) + v2 (k 1)
Nu v2 (k 1) > v2 (p 1) th p 1|k. Do (n + 1)k n + 1 (mod p) theo nh l Fermat
nh. Tuy nhin theo (5) th n (n + 1)k (mod p) nn n n + 1 (mod p), mu thun. Vy
v2 (k 1) < v2 (p 1). Khi theo phng trnh ta c
1 6 v2 (p + 1) = v2 (2 + 2) < v2 (n + 2)
Do v2 (n + 2) > 2. Ta suy ra n 2 (mod 4).
x

1. Nu p > 5 th 22 + 1 > 5 nn x > 2. Do p 2 (mod 5). p dng n 2 (mod 4) th


ta suy ra pn 4 (mod 5). Do 4 + n (n + 1)k (mod 5). V n + 4 6 n + 1 (mod 5)
nn k 6 1 (mod 4). V k l nn k 3 (mod 4). Vy 4 + n (n + 1)3 (mod 5).

25
Nu n 0 (mod 5) th 4 + n (n + 1)3 3 (mod 5), mu thun.
Nu n 1 (mod 5) th 4 + n (n + 1)3 2 (mod 5), mu thun.
Nu n 2 (mod 5) th 4 + n (n + 1)3 4 (mod 5), mu thun.
Nu n 3 (mod 5) th 4 + n (n + 1)3 3 (mod 5), mu thun.
Nu n 4 (mod 5) th 4 + n (n + 1)3 3 (mod 5), mu thun.
Vy vi mi n N th n + 4 6 (n + 1)3 (mod 5). Ta loi trng hp p > 5.
2. Nu p = 5 th = 2. Khi th 3 = v2 (n + 2) + v2 (k 1). V v2 (n + 2) > 2 nn ta suy
ra v2 (n + 2) = 2, v2 (k 1) = 1. Ta cng c 5n + n = (n + 1)k .
Vi n = 2 th k = 3.
Vi n > 3. Gi q l c nguyn t l ca n th q|5(n,q1) 1 = 52 1 = 24. Vy q|3
nn q = 3. Do n 0 (mod 6). Kt hp vi n 2 (mod 4) ta suy ra 5n 1
(mod 13) nn n 1 (n + 1)k (mod 13). p dng nh l 1 ta c
n

n
k1
= v3 (k 1) + v3 (n)
v3 (5 1) = v3 (n + 1)
1 1 + v3
2
Vy 3|k 1. Ta cng c k 3 (mod 4) nn k 7 (mod 12). Theo nh l Fermat
nh ta suy ra (n + 1)k (n + 1)7 (n + 1) (mod 13). Nh vy n 1 n 1
(mod 13) dn n n 0 (mod 13), v l. (v vi 13|n th 5n 1 (mod 13), mu
thun do 5n 5 (mod 13)).
Vy (p, n, k) = (3, 1, 2), (5, 2, 3).
V d . Tm b ba s nguyn dng (a, b, c) sao cho ab + 1 = (a + 1)c .
Li gii. Gi p l mt c nguyn t l ca a. Khi th theo nh l 1 ta c
vp ((a + 1)c 1) = vp (a) + vp (c) > vp (a) b
vp (c) > vp (a)(b 1)

(6)

1. Nu c l th ta c v2 ((a + 1)c 1) = v2 (a). Do b = 1. Nh vy th ta c a + 1 = (a + 1)c


suy ra c = 1.
2. Nu c chn th v2 (c) > 1 v b > 2. Theo nh l 4 th
v2 ((a + 1)c 1) = v2 (a) + v2 (a + 2) + v2 (c) 1 = v2 (a) b

(7)

Nu v2 (a) = 1 th ta lun c v2 (c) > v2 (a). Kt hp vi (6) ta suy ra c > a(b1) > b,
mu thun v lc th (a + 1)c > ab + 1.
Nu v2 (a) > 2 th (7) v2 (c) = v2 (a) (b 1). Kt hp vi (6) ta dn n
c > a(b 1) > b, mu thun
Vy phng trnh c nghim (a, b, c) = (k, 1, 1) vi k l s nguyn dng ty .
Nhn xt. T bi ton trn, ta c thm mt s m rng sau:
M rng 1. Tm cc s nguyn dng m, l, n, k tha mn (1 + mn )l = 1 + mk .

26
M rng 2. (IMO Shortlist 2000) Tm b ba s nguyn dng (a, m, n) tha mn am +1|(a+1)n .
Ngoi vic phng php LTE c ng dng trc tip vo li gii th phng php ny cn
c dng tm dng v hn ca bi ton chia ht.
V d 6. Chng minh tn ti v hn s t nhin n tha mn n|3n + 1.
Phn tch v nh hng li gii. iu by gi ta cn lm v i tm mt trong cc dng ca n
tha mn n|3n + 1.
Trc ht, nhn thy 5|32 + 12 . By gi ta n cc iu kin a, b, p trong nh l 2, p
k
k
k
dng v ta s c 5k+1 |325 + 125 . Do 2 5k |325 + 1. Vy ta ch cn chng minh n = 2 5k
vi k N th n|3n + 1.
k1
Li gii. Trc ht, ta s i chng minh 345 1 (mod 5k ). p dng nh l 1 ta c
 k1

45
v5 3
1 = v5 (34 1) + v5 (5k1 ) = k
k1

Vy 345




k
k
k
k
1 (mod 5k ) hay 5k | 325 1 325 + 1 . Do 5k |325 + 1. Li c 2|325 + 1
k

nn 2 5k |325 + 1.
V k N nn tn ti v hn s t nhin n = 2 5k sao cho n|3n + 1.
V d 7. (Romanian Master of Mathematics Competition 2012) Chng minh tn ti v hn
n
s nguyn dng n tha mn 22 +1 + 1 chia ht cho n.
Phn tch v nh hng li gii. Ta s tm mt s n tha mn iu kin trn. D thy n = 3
tha mn. Ta mnh dn th vi n = 9, 27 cng u tha mn. T y ta d dng tm c
mt dng ca nn l n = 3k . y mnh xin gii thiu hai li gii:
Li gii 1. Ta s i chng minh s nguyn dng an = 3n tha mn yu cu bi ton. Tht
vy, theo nh l 2 ta c
v3 (2an + 1) = v3 (3) + v3 (an ) = k + 1
V
an +1

v3 (22

+ 1) = v3 (3) + v2 (2an + 1) = k + 2

an

Vy an |22 +1 + 1.
Li gii 2. Ta s i chng minh s nguyn dng an =

23 +1
9

tha mn yu cu ra.

p dng nh l 2 ta c
v3 (an ) = v3 (3) + v3 (3n ) 2 = n 1
t an = 3n1 m vi m N , gcd(3, m) = 1. Ta c
an +1

v3 (22
an +1

Vy 3n1 |22

+ 1) > v3 (2an + 1) > v3 (an ) = n 1.

+ 1. Mt khc, tip tc p dng nh l 2 th


v3 (2an + 1) = v3 (3) + v3 (an ) = n
n

Do 3n |2an + 1. Vy ta suy ra 23 + 1|22


an
V gcd(m, 3) = 1 nn an |22 +1 + 1.

an +1

an +1

+ 1. M m|2an + 1 nn m|22

+ 1.

27

Bi tp vn dng
1. Chng minh phng trnh x7 + y 7 = 1998z khng c nghim nguyn dng.
2. Tm tt c s nguyn dng n tha mn 72013 |5n + 1.
2016 1

3. Tm s nguyn dng n ln nht sao cho 2n |20112013

1.

4. Chng minh tn ti v hn s nguyn dng n N tha mn n2 |2n + 3n + 6n + 1.


5. (Japan MO Finals 2012) Cho p l s nguyn t. Tm mi s nguyn n tha mn vi mi
s nguyn x, nu p|xn 1 th p2 |xn 1.
6. Cho a > b > 1, b l mt s l, n l mt s nguyn dng. Nu bn |an 1. Chng minh
n
ab > 3n .
7. Tm s nguyn dng n tha mn 9n 1 chia ht cho 7n .
8. (IMO Shortlist 2007) Tm mi hm s ton nh f : N N sao cho vi mi m, n N v
vi mi p nguyn th, f (m + n) chia ht cho p khi v ch khi f (m) + f (n) chia ht cho p.
9. (IMO 2000) Tn ti hay khng s nguyn n tha mn n c ng 2000 c nguyn t v
2n + 1 chia ht cho n ?
10. Vi mt s t nhin n, cho a l s t nhin ln nht tha mn 5n 3n chia ht cho 2a .
Ly b l s t nhin ln nht tha mn 2b 6 n. Chng minh rng a 6 b + 3.
11. Chng minh rng nu n > 2 sao cho n|7n 3n th n chn.
12. Tm s nguyn dng n tha mn
i) n|5n + 1.
ii) n2 |5n + 1.
iii) n3 |5n + 1.
13. Tm mi s nguyn dng k sao cho k s nguyn t l u tin p1 , p2 , pk u tn ti
hai s nguyn dng a, n tha mn
p 1 p 2 p k 1 = an
14. (MOSP 2001) Tm cc s nguyn dng (x, r, p, n) tha mn xr 1 = pn .
15. Tm tt c cc b s (m, p, q) vi p, q nguyn t v m nguyn dng sao cho 2m p2 +1 = q 5 .
16. (Iran TST 2009) Cho n l mt s nguyn dng. Chng minh rng
3

n
52 1
2n+2

(5)

n
32 1
2n+2

(mod 2n+4 )

28
17. (IMO Shortlist 2010) Tm cc cp s nguyn khng m (m, n) tha mn

m2 + 2 3n = m 2n+1 1 .
18. (Iran Third Round 2011) Cho s t nhin k > 7. C bao nhiu cp nguyn dng (x, y)
tha mn
x
y
7373 99 (mod 2k )?
19. Gii phng trnh nghim nguyn dng trong p l s nguyn t:
pa 1 = 2n (p 1)

Ti liu tham kho


[1] Amir Hossein Parvardi, Lifting The Exponent Lemma: (ti liu pdf)
[2] Cc din n ton:
diendantoanhoc.net/forum
forum.mathscope.org
mathlinks.ro

Chuyn 3:
Cc bi ton s hc hon v vng quanh
Nguyn Anh Huy, Nguyn Vit Tm

S bnh ng v hon v gia cc n s l mt np p ca Ton hc, thng c


gp trong cc bi ton h phng trnh v bt ng thc. Tng t, S hc cng
c nhng bi ton hon v vng quanh m cc n l s nguyn, s nguyn dng,
tuy nhin li gii a dng v phc tp hn rt nhiu. Bi vit ny s cp n
hai hng i c th gii dng bi trn, l i xng ha v bt ng thc.
Trong bi vit c s dng mt s kin thc v bc nhy Viete v hm vp (n), bn
c c th tham kho hai chuyn trc.

Phng php i xng ha


Phng php ny thng c dng trong cc bi ton chia ht hon v vng: a | f (b); b | f (a).
.
Nu c (a; b) = 1 ta xy dng hm g tha g(x) .. x x Z v h tha
h(a; b) = f (a) + g(b) = f (b) + g(a) i xng theo a, b. Khi
a | h(a; b); b | h(a; b) ab | h(a; b) h(a; b) = kab
y l phng trnh nghim nguyn vi a, b i xng. Ta c th dng bt ng thc nu
deg h = 1 hoc bc nhy Viete nu deg h = 2.
Nu khng c (a; b) = 1 th t gi thit ta suy ra ab | f (a)f (b), sau khai trin v phi v b
cc hng t chia ht cho ab c phng trnh nghim nguyn c dng tng t.

Bi tp 3.1. Tm cc s nguyn t p > q tha

q 1 | 3p 1
p 1 | 3q 1
Li gii
t a = p 1, b = q 1 (a > b > 1), ta c

a | 3(b + 1) 1
.
ab | (3a + 2)(3b + 2) () 6a + 6b + 4 .. ab
b | 3(a + 1) 1
1

Lp 12CT THPT chuyn L Hng Phong

29

30
T ta c
6a + 6b + 4 > ab

6 6
4
+ +
>1
a b ab

Li c
12
4
4
6 6
+ 2 > + +
b
b
a b ab
Suy ra
12
4
+ 2 > 1 b2 12b 4 6 0 1 6 b 6 12
b
b
Do b + 1 nguyn t nn b {1; 2; 4; 6; 10; 12}. Xt cc trng hp sau, vi lu a + 1 nguyn
t:
.
.
* Nu b = 1 : () 6a + 6 + 4 .. a 10 .. a a {1; 2; 10}.
.
.
* Nu b = 2 : () 6a + 12 + 4 .. 2a 8 .. a a {2; 4}.
.
.
* Nu b = 4 : () 6a + 24 + 4 .. 4a a + 14 .. 2a a {6; 10}.
.
.
* Nu b = 6 : () 6a + 36 + 4 .. 6a 40 .. 6a (loi).
.
.
* Nu b = 10 : () 6a + 60 + 4 .. 10a 3a + 32 .. 5a a = 16.
.
* Nu b = 12 : () 6a + 72 + 4 .. 12a (loi).
T ta tm c cp (p, q) nguyn t tha bi ton l
(2, 2), (3, 3), (5, 3), (7, 5), (17, 11). 2

Bi tp 3.2. Tm s b s nguyn dng (a; b; c) i mt nguyn t cng nhau tho a < b < c
v
a | bc 31; b | ca 31; c | ab 31
Li gii
Do a | bc 31 v a | a(b + c) nn ta c
a | ab + bc + ca 31.
Tng t vi b v c. Li do (a; b) = (b; c) = (c; a) = 1 nn
abc | ab + bc + ca 31 ()
Xt cc trng hp sau:
~ Trng hp 1: a > 3
Suy ra b > 4; c > 5, do ab + bc + ca > 31. Ta cng c
abc > 3bc > ab + bc + ca > ab + bc + ca 31
iu ny mu thun vi (*).
~ Trng hp 2: a = 2
Suy ra b | 2c 31 v c | 2b 31, do
bc | 2b + 2c 31

31
Chng minh tng t ta c b = 3; c = 5.
~ Trng hp 3: a = 1
Suy ra b | c 31 v c | b 31. Nu b + c = 31 th hai iu kin trn hin nhin tho. Khi ta
c a = 1 < b < c v b + c = 31. Do a < b < c nn 2 6 b 6 15. D thy c 14 b s (a; b; c) tho.
Nu b + c 6= 31 th ta chng minh 1 < b < c < 31. Nu ngc li:
* Nu b > 31 c > b > b 31 > 0. Do c khng l c ca b 31.
* Nu b = 31 th 31 | c 31 31 | c, loi do (b; c) = (31; c) = 1.
* Nu b < 31 < c th |c| > |31 b| > 0 do c khng l c ca b 31.
Nh vy 1 < b < c < 31. Ngoi ra, t b | c 31 v c | b 31 vi (b; c) = 1 ta cng c
bc | 31 b c.
* Nu b > 5 c > 6 bc > 30 > 11 > 31 (b + c), v l.
* Nu b = 4 th 4 | c 31 v c | 27 c = 27 (loi do b + c 6= 31).
* Nu b = 3 th 3 | c 31 v c | 28, suy ra c {4; 7}.
Vy trng hp 3 cho 16 b (a; b; c) tho bi ton.
Kt lun: C 17 b s (a; b; c) tho bi ton. 2

Bi tp 3.3. Tm cc s nguyn t p, q tho p < q < 1000 v


q | p3 1; p | q 3 1
Li gii
Ta c q | p3 1 = (p 1)(q 2 + q + 1) m q > p 1 nn
q | p2 + p + 1.
V p | q 3 1 = (q 1)(q 2 + q + 1). Do ta xt 2 trng hp:
~ Trng hp 1: p | q 1
t p2 + p + 1 = nq (n N ). Do p2 + p + 1 q 1 (mod p) nn n 1 (mod p).
Li c
 2
 

p +p+1
1
p2 + p + 1
6
= p+
= p (do q > p + 1)
n=
q
p+1
p+1
Suy ra n = 1, vy q = p2 + p + 1. Do p 6 31, v 372 + 37 + 1 > 1000.
Th vi tt c s nguyn t p < 31 ta tm c cc b s tho bi ton l
(p; q) = (2; 7), (3; 13), (5; 31), (17; 307).
~ Trng hp 2: p | q 2 + q + 1
Do p | q 2 + q + 1 v q | p2 + p + 1 nn ta c
p | q 2 + q + 1 + p2 + p; q | p2 + p + 1 + q 2 + q
hay
pq | p2 + q 2 + p + q + 1

32
Nh vy ta xt phng trnh nghim nguyn dng
a2 + b2 + a + b + 1 = mab () (m N )
Vit li (*) di dng
a2 + (1 mb)a + b2 + b + 1 = 0 ()
Gi S l tp cc b s nguyn dng (a; b) tho (**). Theo nguyn l cc hn trong S tn ti
cp s (a0 ; b0 ) tho a0 + b0 nh nht. Khng gim tng qut gi s a0 > b0 .
Theo nh l Viete, (**) cng c nghim (a0 ; b0 ) trong a0 tho

a + a0 = mb 1
0
0
a0 a0 = b2 + b0 + 1
0

T phng trnh u suy ra a0 Z. T phng trnh sau suy ra a0 > 0. Vy (a0 ; b0 ) S. Do


a0 + b0 6 a0 + b0 a0 6 a0 =

b20 + b0 + 1
b20 + b0 + 1 > a20
a0

Nu a0 > b0 th a0 > b0 + 1 a20 > (b0 + 1)2 > b20 + b0 + 1, mu thun. Vy a0 = b0 , suy ra
a20 + (1 a0 ).a0 + a20 + a0 + 1 = 0
.
Suy ra 1 .. a0 a0 = b0 = 1. Vy ta c m = 5.
Khi (*) tr thnh
a2 + b2 + a + b + 1 = 5ab ()
Theo chng minh trn, nu (a0 ; b0 ) l nghim ca (*) th (5a0 b0 1; a0 ) cng l nghim ca
(*). Do t nghim (1; 1) ta c nghim (3; 1), sau l (13; 3), ...
Tm li cc nghim ca (*) c cho bi cng thc

x = x = 1
0
1
(ai ; bi ) = (xi+1 ; xi ) vi (xn ) :
xn+1 = 5xn xn1 1 n > 1
Vic chng minh iu trn hon ton tng t bi s hc VMO 2012, v xin c dnh cho
bn c.
Tip theo ta tm cc nghim l s nguyn t nh hn 1000. D thy ch cn xt i 6 4, do
x6 > 1000. T ta tm c (p; q) = (3; 13), (13; 61).
Kt lun: Cc b s (p; q) tho bi ton l (2; 7), (3; 13), (5; 31), (13; 61), (17; 307). 2

Phng php dng bt ng thc


Ta c th sp xp cc n s i xng theo th t hoc ch ra mt n c mt i lung no
ln nht hoc b nht. Lu vic sp xp y khng ch gii hn a > b > c, m cng c
th l f (a) > f (b) > f (c), trong f l mt hm s hc.

33
Bi tp 3.4. Cho cc s nguyn dng a, b, c. Chng minh
(a; b).(b; c).(c; a)
[a; b].[b; c].[c; a]
=
2
(a; b; c)
[a; b; c]2
Trong (a; b) v [a; b] ln lt l l c chung ln nht v bi chung nh nht ca a, b.
Li gii
Gi p l s nguyn t bt k. Nu p khng l c ca [a; b; c] th d thy vp (V T ) = vp (V P ) = 0.
Nu p | [a; b; c] th p l c ca a, b hoc c. t a = px .a1 ; b = py .b1 ; c = pz .c1 v khng gim
tng qut gi s x > y > z. Khi ta c
vp (V T ) = y + z + z 2z = y
v
vp (V P ) = x + y + x 2x = y
Suy ra vp (V T ) = vp (V P ) p P. Do V T = V P. 2
Nhn xt l trong li gii trn ta gi s vp (a) > vp (b) > vp (c), v s dng nh l c bn
ca s hc: Mi s nguyn ln hn 1 u c mt v ch mt cch phn tch ra tha s nguyn t.

Bi tp 3.5. Tm cc s nguyn t p, q tha


.
(5p 2p )(5q 2q ) .. pq ()
Li gii
Gi s p 6 q. Nhn xt p = q = 3 tha.
> Nu p = 3, q > 3 th ta c
.
.
117(5q 2q ) .. 3q 39(5q 2q ) .. q
Do 5q 2q 5 2 3 [q] theo Fermat nn q | 39, do q = 13.
> Nu 5 < p < q ( V T () khng chia ht cho 5 nn p, q 6= 5) th tng t ta c
5p 2p 5 2 3 [p], do t gi thit suy ra
.
5q 2q ..p
.
Li c 5p1 2p1 1 [p] nn 5p1 2p1 .. p.
Do q > p 1 nn (q, p 1) = 1. Theo nh l Bezout, tn ti m, n N tha
mq (p 1)n = 1 hoc m(p 1) nq = 1 ()
Xt ng d mod p ta c

5p1

5q

2q

p1

5n(p1)

2n(p1)

5mq

2mq

34
Suy ra
5n(p1) .2mq 2n(p1) .5mq
Kt hp vi (**), sau khi rt gn hai v ta c 5 2 [p] hay p = 3 (loi v ang xt p > 5).
Kt lun: (p; q) = (3; 3), (3; 13), (13; 3). 2

Bi tp 3.6. Tm cc cp s nguyn t (p, q) tha


.
2p + 2q .. pq
Li gii
Nhn xt l (x 1, x + 1) = (2, x + 1) 6 2 x Z.
.
* Nu p = q th 2p+1 .. p2 p = q = 2.
* Nu p > q :

.
.
Xt q = 2 ta c 22 + 2p .. 2p 2 + 2p1 .. p p {2; 3}.
.
Xt q > 2 th do 2p + 2q = 2q (2p1 + 1) .. q nn
.
2p1 + 1 .. q (1) 22(pq) 1 [q]
.
.
Gi a l s nh nht tha 2a 1 [q]. Theo tnh cht ca ord suy ra q 1 .. a v 2(p q) .. a.
.
.
t p q = 2k .m; q 1 = 2l .n; a = 2r .s (k + 1 > r; l > r; m .. s; n .. s; m, n, s l ).
> Trng hp 1: r = k + 1. Ta suy ra l > k + 1 v

2(p q) = 2k+1 .m
2(p q)s = am

q 1
(q 1)m = (p q).n.2lk1
= 2l .n

(q 1).m
q1
2l .n
n.2lk1
=
=
= r
2s
2s(p q)
a
2 .s

2lk2 = 2lr r = k + 2 (mu thun)


> Trng hp 2: r 6 k
.
.
.
Suy ra p q .. a 2pq 1 .. 2a 1 .. q (2)
T (1) v (2) suy ra q | (2pq 1, 2pq + 1).
M (2pq 1, 2pq + 1) 6 2 theo chng minh trn, do q = 2 (mu thun do ang xt q > 2).
Kt lun: (p, q) = (2; 2), (2; 3), (3; 2). 2

Bi tp 3.7. Cho cc s nguyn dng x, y, z tho (xy + 1)(yz + 1)(zx + 1) l s chnh phng.
Chng minh xy + 1, yz + 1, zx + 1 u l s chnh phng.
Li gii

35
Trong cc b s (x; y; z) tho bi ton, xt b (x; y; z) c x + y + z nh nht. (1)
Khng gim tng qut gi s z = max{x; y; z}.
Gi t l s tha phng trnh bc hai
t2 + x2 + y 2 + z 2 2(xy + yz + zt + tx + zx + ty) 4xyzt 4 = 0 ()
t2 2t(x + y + z + 2xyz) + x2 + y 2 + z 2 2(xy + yz + zx) 4 = 0
Nhn xt rng (*) tng ng vi 3 phng trnh sau:
(x + y z t)2 = 4(xy + 1)(zt + 1)
(x + z y t)2 = 4(xz + 1)(yt + 1)
(x + t y z)2 = 4(xt + 1)(yz + 1)
V t nguyn do (*) c 2 nghim nguyn
p
t1,2 = x + y + z + 2xyz 2 (xy + 1)(yz + 1)(zx + 1)
Nn nhn c 3 phng trnh trn v theo v, ta suy ra (xt + 1)(yt + 1)(zt + 1) l s chnh
phng.
Ngoi ra ta cng c
xt + 1 > 0; yt + 1 > 0; zt + 1 > 0
Suy ra
t>

1
> 1 (do x = y = z = 1 khng tho)
max{x; y; z}

* Nu t = 0 th t (*) ta suy ra
(x + y + z)2 = 4(xy + yz + zx + 1) (x + y z)2 = 4(xy + 1)
Suy ra xy + 1 l s chnh phng. Chng minh tng t ta cng c yz + 1, zx + 1 l s chnh
phng. * Nu t > 0 th t (1) suy ra t > z vi mi t tho (*).
Nhng t ch c th bng t1 hoc t2 , v ta li c
t1 t2 = x2 + y 2 + z 2 2(xy + yz + zx) 4 6 z 2 x(2z x) y(2z y) < z 2
Suy ra iu mu thun. Vy ta c pcm. 2

Bi tp 3.8. Tm cc b s nguyn (a; b; c) tha

a bc = 91
b2 ca = 91

c2 ab = 91
Li gii

36
Khng gim tng qut gi s a 6 b 6 c. Do 91 khng l s chnh phng nn a, b, c 6= 0.
Nu a > 0 th b, c > 0 do a2 bc 6 0 < 91, vy a < 0.
Nu (a; b; c) tha h th (a; b; c) cng tha h. Do ta xt cc b s c c > 0.
Nu b = c th ta c
"
ab=0
a2 b2 = 91 = b2 ab
a = 2b
D thy a = b = c v l, v a = 2b cng dn n 5b2 = 91, v l.
Do b 6= c. Tng t ta c a 6= b.
~ Trng hp 1: b > 0
T h ta c
91b = b(b2 ac); 91c = c(c2 ab)
91b 91c = b3 c3
91 = b2 + bc + c2 > 3b2
b {1; 2; 3; 4; 5}
Thay vo h ta tm c cc nghim (a; b; c) = (10; 1; 9), (11; 5; 6).
~ Trng hp 2: b < 0
T h ta c
91b = b(b2 ac); 91a = a(a2 bc)
91(b a) = b3 a3
91 = a2 + ab + b2 > 3b2
b {1; 2; 3; 4; 5}
Thay vo h ta tm c cc nghim (a; b; c) = (9; 1; 10), (6; 5; 11).
Tm li cc b (a; b; c) tha gi thit l
(10; 1; 9), (10; 1; 9), (11; 5; 6), (11; 5; 6), (9; 1; 10), (9; 1; 10), (6; 5; 11), (6; 5; 11)
v cc hon v. 2

Bi tp t luyn
Bi 1: (APMO 2002) Tm cc s nguyn dng a, b tho
b2 a | a2 + b; a2 b | b2 + a
Bi 2: Chng minh c v hn b s nguyn dng (a; b; c) tho ab + 1, bc + 1, ca + 1 u l
s chnh phng.
Bi 3: Tm cc s nguyn dng x, y, z i mt nguyn t cng nhau tho
x y z
+ + N
y z x
Bi 4: Tm cc b s nguyn (x; y; z) tho 2 6 x 6 y 6 z v
z | xy 1; y | zx 1; x | yz 1

37
Bi 5: Tm cc s nguyn dng a, b, c > 1 i mt khc nhau tha
(a 1)(b 1)(c 1) | abc 1
Bi 6: Chng minh vi mi s nguyn dng a, b, n th
(36a + b)(36b + a) 6= 2n
Bi 7*: (VMO 2013) Tm s cc b (a; b; c; a0 ; b0 ; c0 ) vi a, b, c, a0 , b0 , c0 {0; 1; 2; ...; 14} v
tho
ab + a0 b0 bc + b0 c0 ca + c0 a0 1( mod 15)
Bi 8: (VMO 2012) Tm cc s nguyn dng l a, b tho
a | b2 + 2; b | a2 + 2
Bi 9: Cho a, b, c Z tha

a b c
+ + =3
b c a

Chng minh abc l lp phng mt s nguyn.

Ti liu tham kho


1. Trang web brilliant.org.
2. Cc chuyn s hc - Phan Huy Khi.
3. Cc phng php gii ton qua cc k thi Olympic - Trn Nam Dng (Ch bin), V
Quc B Cn, L Phc L.
4. Problems in Elementary Number Theory - Peter Vandendriessche, Hojoo Lee

38

Chuyn 4:
DY S S HC
Ninh Vn T1

Dy s l mt vn kh thit yu trong gii tch v c ng dng vo kh nhiu


cc lnh vc khc nh phng trnh hm, t hp, s hc. . . Nhng bi ton v gii
hn dy s dng nh tr thnh vn kh quen thuc v xut hin nhiu trong
cc k thi hc sinh gii cp trng, cp tnh, cp thnh ph cng nh cc k thi
Olympic, VMO. . . Nhng mt mng kh c bit ca dy s trong vic ng dng
s hc cng nh cc bi ton s hc gii cc bi ton dy s l mt vn kh
th v trong mng dy s. Hi vng chuyn ny s gip ch cho cc bn trong vic
tip thm kinh nghim v mng th v ny.

Dy s nguyn v tnh cht s hc


Ta s ln lt xt tng quan cc bi ton vi dng tng qut c th c ci nhn
bao qut ht v th gii dy nguyn phong ph, a mu. T ta s c c cc
tng khi gp mt bi ton chng minh dy nguyn hoc cc bi ton lin quan
n dng y.

Bi tp 4.1. Cho a, b, c Z tha mn a2 = b + 1. Dy s (un ) c xc nh


(

u0 = 0
un+1 = aun +

bun 2 + c2 , n N

Chng minh rng mi s hng ca dy u l s nguyn.


Li gii
Cch gii bi ton ny cng tng t nh cch tm cng thc truy hi ng ca mt dng cng
thc truy hi cn thc tm cng thc tng qut ca mt dng dy ny nhng vi iu kin
ngt. Ta c:
p
un+1 aun = bun 2 + c2
un+1 2 2aun un+1 + a2 un 2 = bun 2 + c2
un+1 2 = 2aun un+1 un 2 + c2
1

Hc sinh THPT chuyn Trn i Ngha.

39

(1)

40
Gim n xung 1 n v ta c:
un 2 = 2aun un1 un1 2 + c2

(2)

Ly (1) (2), ta c:
"
un+1 2 un1 2 = 2aun (un+1 un1 )

un+1 = un1
un+1 = 2aun un1

*Vi un+1 = un1 , n > 1 th dy (un ) l dy tun hon theo chu k 2. Nh th , ta suy ra
(
u0 = u2 = u4 = ... = u2k = 0
u1 = u3 = u5 = ... = u2k+1 = |c|
Nn (un ) c cc s hng u l s nguyn.
*Vi (un ) tha
(
u0 = 1; u1 = |c|
un+1 = 2aun un1 , n > 1

Ta c cc h s trong h thc truy hi u l s nguyn v 2 s hng u tin ca dy cng l


s nguyn nn t ta suy ra (un ) c cc s hng u l s nguyn.
Tm li, cc s hng ca dy trn u l s nguyn. 2

Bi tp 4.2. Cho dy (an ) tha mn iu kin


(
a0 = 2; a1 = 5
an+1 an1 an 2 = 6n1 , n > 1
Chng minh rng tn ti duy nht dy s nguyn dng tha mn iu kin trn.
Li gii
tng ca bi ton ny l tm mt cng thc truy hi ng cho dy trn. Nh vy, ta s
dng phng php quen thuc nh gi bi ton, l phng php sai phn.
Ta c
(
an+1 an1 an 2 = 6n1
an an+2 an+1 2 = 6n
an (an+2 + 6an ) = an+1 (an+1 + 6an1 )

t vn =

(an+1 +6an1 )
,
an

(an+2 + 6an )
(an+1 + 6an1 )
=
an+1
an

ta c

vn+1 = vn = vn1 = ... = v1 = 5 an+1 = 5an 6an1 , n = 1, 2, 3...


Do cc s hng u ca dy l s nguyn: a0 = 2, a1 = 5 v cng thc truy hi ng ca dy
u c cc h s l s nguyn nn t ta suy ra dy trn l dy s nguyn.

41
c thm iu kin dy trn c mi s hng l s nguyn dng, ta cn phi c thm yu
t n iu tng. Vic chng minh n iu khng th s dng cng thc truy hi m ta va
tm c v ta cha chng minh c cng thc y l duy nht. Chnh v th m ta s chng
minh trc tip bng quy np thng qua cch din t ca bi.
Ta thy iu trn ng vi n = 1 : a1 > a0 (5 > 2).
Gi s iu ny ng n n = k : ak > ak1 .
Xt n = k + 1, ta c:
ak+1 ak1 ak 2 = 6k1 ak+1 =

6k1
6k1 + ak 2
>
+ ak > ak (ak > ak1 )
ak1
ak

Vy iu trn cng ng vi n = k + 1 nn theo nguyn l quy np ta suy ra (an ) l dy n


iu tng vi mi n N.
By gi ta s chng minh dy trn l dy duy nht tha mn iu kin bi:
Cch thng thng m chng ta s ngh n l dng phn chng chng minh tn ti duy
nht.
Gi s tn ti dy an 0 sao cho vi n > 2 tn ti an+2 0 : an+2 > an+2 0 tha:
(
an an+2 an+1 2 = 6n
an an+2 0 an+1 2 = 6n
Suy ra
an (an+2 an+2 0 ) = 0 (v l do an+2 > an+2 0 )
Nh vy ta suy ra dy trn l dy duy nht tha mn iu kin bi. 2
Nhng bi ton tng t dng trn c th tn ti nhiu dng khc nhau. i lc
nhng cch din t bng truy hi khin ta mt i phng hng sai phn. Nhng
ta cn phi hiu mc ch ca vic chng minh dy s nguyn l tm c cng
thc truy hi ng ca n.

Bi tp 4.3. (GER 2003, Ngy 2) Cho dy s (an ) c xc nh bi

a1 = 1; a2 = 1; a3 = 2
an+3 = an+2 an+1 + 7 , n N
an
Chng minh mi s hng ca dy trn u l s nguyn dng.
Li gii
Ta c:
an+3 an = an+1 an+2 + 7
Do lch cc th t ca phn t trong dy khng u nhau nn s kh cho ta ngh n vic
dng sai phn v nu dng sai phn s xut hin thm phn t th 5. Tuy vy, nhng vic p
dng sai phn trong dy s nguyn l mt cng c kh mnh v ta khng nn b qua n chp

42
nhong trong ngh.
H n xung 1 bc, ta c:
an+2 an1 = an an+1 + 7
Tr 2 biu thc trn cho nhau, ta c
an+3 an an+2 an1 = an+1 an+2 an an+1
an (an+3 + an+1 ) = an+2 (an+1 + an1 )
an+3 + an+1
an+1 + an1

=
an+2
an
n1
, ta c vn+2 = vn .
t vn = an+1a+a
n
Vy dy (vn ) tun hon theo chu k 2.
Nh vy ta cn xt thm tnh chn l ca dy, iu ny dn n y l dy gm 2 cng thc
truy hi song song nhau.
*Vi n chn ta suy ra:

v2k+2 = v2k = ... = v2 = 3 an+1 = 3an an1


*Vi n l ta suy ra:
v2k+3 = v2k+1 = ... = v3 = 5 an+1 = 5an an1
T ta xc nh c cng thc truy hi ca dy l:

a = a2 = 1, a3 = 2

1
(k N)
an+1 = 3an an1 , n = 2k

a
= 5a a , n = 2k + 1
n+1

n1

Do cc s hng u ca dy u l s nguyn v h s ca h thc truy hi ca dy cng l s


nguyn nn ta suy ra mi s hng ca (an ) u l s nguyn.
dy trn l dy s nguyn dng, ta cn chng minh dy trn l dy tng ngt vi mi
n > 2.
D thy: a3 > a2 nn iu trn ng vi n = 2.
Gi s iu trn ng vi n = k : ak > ak1
Xt n = k + 1, ta c:
ak ak1 + 7
ak ak1 + 7
7
ak+1 =
>
> ak +
> ak
ak2
ak1
ak1
Vy iu ny cng ng vi n = k + 1 nn theo nguyn l quy np ta suy ra (an ) l dy tng
ngt.
T , ta chng minh c dy (an ) c mi s hng u l s nguyn dng. 2
Bi tp 4.4. (Croatia TST 2011) Vi a, b l 2 s nguyn t phn bit, cho dy (xn ) tha
mn

x1 = a, x2 = b
xn+1 2 + xn 2

, n N
xn+2 =
xn+1 + xn
Chng minh rng xn khng l s nguyn vi mi n > 3.

43
Li gii
Ta d dng chng minh c xn > 0, x N theo quy np.
Ta s chng minh mt b ca dy ny:
B : Nu k t mt phn t n0 sao cho xn0 khng l s nguyn ca dy m c
dng phn thc, th s khng tn ti n xn N, n > n0 .
Gi s x3 l mt s hu t.
Ta c:

c
a2 +b2

x3 = a+b = d (c, d N; (c, d) = 1)

c 2
2

x4 = ( d c) +b =
+b
d

c2 +b2 d2
d(c+bd)

Do (c, d) = 1 nn x4
/ N, t
e
x4 =
f

.
e, f N; (e, f ) = 1; f ..d

.
Ta c: f .. d f > d. Ta chia lm 2 trng hp:
> Trng hp 1: f > d 1 > fd , ta c:
 2

x5 =


c 2
+ fe
d
c
+ fe
d

(cf )2 + (ed)2
=
(cf + de) df

Gi s x5 N, suy ra
.
.
.
(cf )2 + (de)2 .. df (de)2 .. df de2 .. f (v l)
Mt khc t s cng khng th bng mu s do f < (cf )2 + (de)2 nn f khng th chia ht
cho (cf )2 + (de)2 .
iu ny v l do (e, d) = 1 v d < f .
Vy x5 l s v t.
> Trng hp 2: f = d, ta c:
 2

x5 =
Gi s x5 N, suy ra


c 2
+ fe
d
c
+ fe
d

(cf )2 + (ed)2
c2 + e2
=
(cf + de) df
f (c + e)

.
.
c2 + e2 .. c + e 2ce .. c + e

iu ny hin nhin v l.
Vy tm li, ta chng minh rng, nu xut pht vi x3 l mt s hu t th ta s thu c
mt dy s hu t v khng tn ti bt k s nguyn no trong dy vi n > 3.
By gi ta ch vic chng minh x3 khng th l s nguyn. Do cng thc truy hi c dng phn

44
thc nn x3 s c dng phn s ti gin.
Gi s x3 N, suy ra

.
.
a2 + b2 .. a + b 2ab .. a + b

Xt mt trong hai s l s nguyn t chn. Gi s l a, suy ra


.
.
a = 2 4b .. b + 2 4 .. b + 2 (v l do b > 2)
Suy ra 4b = b + 2 (v l do b l s nguyn t l nn tnh chn l ca 2 v khng ng nht).
K hiu {a, b} l c ca a hoc b.
Vy a + b | {a, b, 2} do (a, b, 2) = 1.
Ta c nhn xt rng nu a + b | {a, b, 2} do:
* a + b | {a, b} th
(
(
a>a+b
0>b

(sai do a, b > 0)
b>a+b
0>a
* a + b | {2}: Do a, b l s nguyn t nn a + b > 2. Suy ra

a + b ... a
cb ... a
a + b = 2ab

a = b (sai do a 6= b)
.

..
a.b
a + b .. b
Nh vy, ta kt lun rng iu gi s l sai nn x3 khng th l s nguyn.
Vy theo b trn ta suy ra xn khng th l s nguyn vi mi n > 3. 2
Bi tp 4.5. Cho mt dy s nguyn dng sao cho ta c th chn 1998 phn t bt k ca
dy to thnh mt h thng d khng y v ring bit nhau khi xt modulo 1999. Liu c
tn ti cch chia 1998 phn t ny thnh 2 tp con A, B sao cho tch ca cc phn t trong
mi tp con bng nhau khng?
Li gii
Mt suy lun n gin cho bi ny: 2 tch bng nhau th chng phi c cng s d khi
xt modulo vi mt s nguyn dng.
.
S dng b quen thuc: Nu x2 + y 2 .. p (p nguyn t ) th p = 4k + 3 hoc p = 4k + 1. Khi
.
p = 4k + 3 th x, y .. p.
Gi 1998 phn t bt k ca dy tha iu kin bi l x1 ; x2 ; x3 ; ...; x1998 .
Ta c: A B = {x1 ; x2 ; ...; x1998 }.
Gi s xi 0 (mod 1999). Do 1998 phn t ny lp thnh mt h thng d khng y v
ring bit nhau nn
xj r 6= 0 (mod 1999) j 6= i
V vy nu xi A th A chia ht cho 1999 v B khng chia ht cho 1999. Tng t nu xi B.
Vy khng c phn t no trong dy chia ht cho 1999. Vy h thng d ca dy: {1; 2; 3...; 1998}.
Ta c:
!
Y
Y
Y
xk 2 =
xi .
xj 1.2.3...1998 (mod 1999)
xk A

xi A

xj B

45
Suy ra
!2
Y

xk

1998! (mod 1999)

xk A

Mt khc theo nh l Wilson ta c: 1998! 1 (mod 1999), do


!2
Y
xk + 1 0 (mod 1999)
xk A

p dng b trn ta suy ra iu v l do 1999 l s nguyn t dng 4k + 3 nn 1 khng th


chia ht cho 1999.
Vy khng th chia 1998 phn t bt k ca dy nguyn dng ny thnh 2 tp con c tch cc
phn t trong tp bng nhau. 2

Bi tp 4.6. Cho dy s (un ) tha

u1 = 1, u2 = 2
un un1 (n + 1)

un+1 =
Pn1
1
a) Tm s nguyn t p S = up2
l s nguyn.
p1
p
 un1 
b) Chng minh n 0 (mod n 1).

Li gii
tng ca bi ny n gin l tm pl s nguyn t up2 1 0 (mod pp1 ). Qua , ta
d on cng thc tng qut ca un c th a v mt bi ton s hc n thun.
> Cu a:
Ta chng minh theo qui np un = n! n
( 1, 2, 3....
u1 = 1 = 1!
Tht vy iu ny ng vi n = 1; 2 :
u2 = 2.u1 = 2!
Gi s iu ny ng n n = ktc uk = k! Xt uk+1 , ta c:
uk+1 =

(k + 1)uk uk1
(k + 1)!(k 1)!
=
= (k + 1)!
Pk1
(k 1)!

Vy iu ny ng n n = k + 1nn theo nguyn l quy np ta suy ra un = n!n = 1; 2; ....


Vy S = (p2)!1
.Gi s S l s nguyn, ta suy ra S.pp1 + 1 = (p 2)!.
pp1
Xt p > 5. ta c:
(p 2)! 0 (mod p 1) S.pp1 + 1 0 (mod p 1) S + 2 0 (modp 1)
Mt khc: S + 2 < p 1 (p 2)! < 1 + pp2 (p 3) (ng).
T ta suy ra iu v l. Vy p 6 5. D thy p = 3 tha mn iu kin bi.

46
> Cu b:
+Trng hp 1: n l s nguyn t.
Theo nh l Wilson ta c:
(n 1)! 1 (mod n)

(n 1)! + 1
N
n

Li c
hu

n1

 

(n 1)!
(n 1)! + 1 1
(n 1)! + 1
1
=
=

=
1 (do 0 < < 1)
n
n
n
n
n

Suy ra
hu

n1

Mt khc, do (n; n 1) = 1

(n 1)! (n 1)
(n 1) [(n 2)! 1]
=
n
n
(n2)!1
n

N nn ta suy ra
hu i
n1
0 (mod n 1)
n

+ Trng hp 2: n khng l bnh phng ca 1 s nguyn t n = rs vi 1 < r < s 6 n 1.


Do r < s 6 n 1 nn ta suy ra r < s 6 n 2 nn r, sl c s ca mt trong cc s
{1; 2; 3...; n 2}.
Vy
h u i (n 1).(n 2)!
(n 2)!
(n 2)!
n1
=
N
=
= (n 1).k 0 (mod n 1)
n
rs
n
n
+Trng hp 3: n l bnh phng ca 1 s nguyn t. t n = p2 vi 3 6 p < n 1 v nu
p = 2 th hin nhin ta c iu phi chng minh.
Lp lun tng t nh trng hp 2, p l c ca mt trong cc s {1; 2; 3...; n 2}. By gi
ta xt xem liu p cn l c ca s no na khng.
Tht vy, ta c: n 1 = p2 1 = (p 1)(p + 1) > 2p > p.
Vy 2p cn l c ca mt s bt k trong cc s {1; 2; 3...; n 2}. Chnh v th m
(n 1)! 0 (mod 2p.p.(n 1)) (n 1)! 0 (mod 2n(n 1))
hu i
(n 1)!
n1

0 (mod n 1)
0 (mod n 1)
n
n
T cc iu trn ta c pcm. 2

Bi tp 4.7. Cho dy s xc nh bi:


an+1 = {an } [an ] , n > 0
Chng minh rng an = an+2 khi n ln.
Li gii

47
> Xt 0 < a0 < 1, ta c
[a0 ] = 0 an = [an ] {an } = 0, n = 1, 2, 3...
> Xt a0 > 1, ta c:
an+1 = {an } [an ] < [an ] < an 0 < [an+1 ] < [an ] < ... < [a1 ] [an0 ] = c1 khi n0 ln
> Xt a0 < 0 an < 0, n = 1, 2, 3... ta c:
an+1 = {an } [an ] > an 0 > [an+1 ] > [an ] > ... > [a1 ] [an0 ] = c2 khi n0 ln
Nh vy [an0 ] = k vi n0 ln.
Ta c:
an+1 = {an } [an ] = an [an ] ([an ])2 = kan k 2 , n > n0
S dng phng php sai phn vi tng hng t ca dy vi h s tng ng ta c:

k n1 an+1 = k n an k n+1

k n an+2 = k n1 an+1 k n




k2
k2
k 2 (1 k n )
n
n
=
k
a
+

a
=
k
a

...
n
i+n
n

1k
1k
1k

2
3

kai+n1 = k ai+n2 k

2
a
i+n = kai+n1 k
Nu |k| > 1 th
an+i



k2
k2
= k an +

(v l do dy trn b chn)
1k
1k
n

Suy ra 1 6 k 6 1 k {1; 0; 1}.


* Nu k = 0 : an = 0, n > n0
* Nu k = 1 : an+1 = an 1, n > n0 (v l do [an0 ] = k n > n0 m khong cch gia 2 hng
t lin tip ca dy l 1).
* Nu k = 1 suy ra
(
an+1 = 1 an
an+1 = 1 an , n > n0
an+2 = 1 an+1
Tr nhau ta c an = an+2 , n > n0 (pcm). 2

Dy s nguyn v tnh chnh phng


Bi tp 4.8.

!n
3+ 5
+
2

!n
3 5
a) Chng minh
l s nguyn.
2
!n
!n
3+ 5
3 5
b) Chng minh mi s hng l ca dy
+
2 u l s chnh phng.
2
2

48
Li gii
> Cu a:
Xt dy s sau:
(

x1 = 3; x2 = 7
xn+2 = 3xn+1 xn n = 1; 2; ....

Ta chng minh theo quy np


!n
!n
3+ 5
3 5
xn =
+
n = 1; 2; ...
2
2

!
!

3 5
3+ 5

+
=3
x =

1
2
2
Tht vy iu ny ng vi n = 1; 2 :
!2
!2

3
+
5
5
3

x =

+
=7

2
2
2
!k
!k
3 5
3+ 5
+
.
Gi s iu ny ng n n = ktc xk =
2
2
Xt xk+1 , ta c:

!k
!k1
!k
!k1
3 5 3+ 5
3 5
3+ 5

+
+

xk+1 = 3xk xk1 = 3


2
2
2
2
!
!k1
!
!k+1
!k1
!k+1
7+3 5
3 5
73 5
3+ 5
3+ 5
3 5
+
=
xk+1 =
+
2
2
2
2
2
2

Vy iu ny ng n n = k + 1 nn theo nguyn l quy np ta suy ra


!n
!n
3+ 5
3 5
xn =
+
n = 1; 2; ...
2
2
 n  n
Do xn = 3+2 5 + 32 5 n = 1; 2; ... c cng thc truy hi vi cc h s nguyn v x1 ; x2
!n
!n
3+ 5
3 5
nguyn nn t ta suy ra xn nguyn. Vy
+
nguyn n = 1; 2; ....
2
2
> Cu b:
Tip tc vi tng bin lun dy nguyn theo cng thc truy hi v mt cht bin i kho
lo chng minh s chnh phng.
Ta c:
!n
!n
3+ 5
3 5
+
2
2
2

!2n
!2n

5+1
51
=
+
2
2
2
"
!n
!n # 2

5+1
51
=

2
2

49

!n

5+1
Vy bi ton tr thnh bi ton th nht v ta ch vic chng minh xn =

2
!n

51
nguyn vi mi n l.
2
!n
!n

5+1
51
Chng minh theo quy np xn =

c cng thc truy hi l


2
2
(

x1 = 1; x2 = 5
v x2n = 5; x2n+1 =

xn+2 = 5xn+1 + 2xn n = 1; 2; ...

!
!

5
+
1
5

=1
x =

1
2
2
Tht vy iu ny ng vi n = 1; 2 :
!2
!2

5
+
1
5

1
5
4

x
=
= 5

2
2
4

Gi s iu ny ng n n = 2k tc x2k+2 = 5x2k+1 x2k v x2k+1 nguyn do x1 ; x3 ; ...

nguyn v x2k = 5.

Vy x2k+2 = 5 ( = x2k+1 )
Xt x2k+3 ; x2k+4 . Ta c:

x2k+3 = 5x2k+2 x2k+1

!2k+2
!2k+2
!2k+1
!2k+1

5+1
51
5+1
51

= 5

2
2
2
2
!2k+1
!
!2k+1
!

5+1
51
5+ 5
5 5
1
1
x2k+3 =
2
2
2
2
!2k+1
!2k+1

!
3+ 5
3 5
5+1
51
x2k+3 =

2
2
2
2
!2k+3
!2k+3

5+1
51
=

(1)
2
2
V

x2k+4 = 5x2k+3 x2k+2

!2k+3
!2k+3
!2k+2
!2k+2

5+1
51
5+1
51

= 5

2
2
2
2
!2k+2
!2k+2

!
5+1
3+ 5
51
3 5
x2k+4 =

2
2
2
2
!
!

2k+4
2k+4
5+1
51
=

(2)
2
2

Li c x2k+3 = 5x2k+2 x2k+1 = 5 x2k+1 l s nguyn (3).

v x2k+4 = 5x2k+3 x2k+2 = 5( ) khng l s nguyn (4).

50
T (1);(2);(3);(4) ta suy ra nhng iu trn ng vi n = k + 1 nn theo nguyn l quy np ta
chng minh c
(

x1 = 1; x2 = 5
v x2n = 5; x2n+1 =

xn+2 = 5xn+1 + 2xn n = 1; 2; ...


Do x2n+1 = nn ta suy ra mi s hng l ca dy trn u l s chnh phng. 2
Da vo bi trn ta c th xt thm mt bi th v sau:

Bi tp 4.9. Cho dy s (xn ) tha mn:


(
x0 = 1; x1 = 3
xn+2 = 6xn+1 xn
Chng minh rng vi mi n > 1 th (xn ) khng l s chnh phng.
Li gii
Tng t tng ca bi 2, ta s bin i kho lo a v y nh dng ca bi
2. Nhng iu c bit y chnh l ta cn 2 dy chng minh song song cng
nhau bng quy np cng c cho li gii ca bi ton thm cht ch. (Lu : Li
gii ch b sung v cho ta thy mt ng dng kh mnh ca bi 2).
D dng chng minh theo quy np:
n
n
3+2 2 + 32 2
xn =
2
Ta bin i xn theo 2 hnh thc sau:

n
n i2

2n
2n

2+1 +
21

2+1
+
21

xn =
=
1
2
2
h
i

n
n 2

2n
2n

2
+
1

2+1
+
21

xn =
=
+1
2
2

n
n
V ta c th dung phng php truy hi tm cng thc truy hi ca
2+1 21
ln lt c cng thc truy hi l:
(
(

v0 = 2; v1 = 2 2
z1 = 2; z2 = 4 2
v

vn+2 = 2 2vn+1 vn
zn+2 = 2 2zn+1 zn
Theo nh bi trn ta c th chng minh chng ln lt c tnh cht
(
(

v2n = a
z2n = c 2
(c; d Z)
(a; b Z) v
v2n+1 = b 2
z2n+1 = d
T ta rt ra c xn c dng A2 + 1 hoc B 2 1 nn r rng xn khng th l s chnh
phng vi mi n > 1. 2

51
Mt vn th v rng ti sao ta li phi cn n 2 dy song song chng minh.
Chnh 2 dy song song ny to nn s th v ca bi ton, khin n tr nn
cht ch hn trong li gii. V khi xt n chn v l ta c th din t xn theo 2
2
hnh thc v do 2 hnh thc trn u c dng 2x2 1 nn hin nhin n khng th
l s chnh phng vi mi n > 1.
Mt li gii khc chung tng nhng li gii ngn gn hn:
Xt dy s nguyn
(
u0 = 0; u1 = 2
un+2 = 2un+1 + un
Ta d dng chng minh theo quy np

n
n
( 2 + 1) 1 2

un =
2
(
Mt khc, dy

x0 = 1; x1 = 3
xn+2 = 6xn+1 xn

c cng thc tng qut l

n

n
2n
2n
+
3+2 2 + 32 2
2+1
21
xn =
=
2
2
Vy xn + (1)n+1 = un 2 . T ta kt lun rng xn khng th l s chnh phng vi mi
n > 1. 2

Bi tp 4.10. (CIS 1992)


Cho dy s an xc nh bi
(
a1 = 1
an+1 = a1 2 + a2 2 + a3 2 + ... + an 2 + n n > 1
Chng minh rng an khng l s chnh phng vi mi n > 2.
Li gii
Vi nhng bi ton c cng thc truy hi phc tp nh trn, vic u tin ta cn ngh n l
s dng sai phn a v mt biu thc ngn gn v n gin hn bin lun. Qua , ta
vn dng cc tnh cht ca s chnh phng chng minh. C th s chnh phng ch c tn
cng l 0, 1, 4, 5, 6, 9.
Ta c: a2 = 2
Vi mi n > 2; ta c:

n
X

a
=
ak 2 + n

n+1
k=1
an+1 an = an 2 + 1 > 0
n1

ak 2 + n 1
an =
k=1

52
Ta suy ra dy ny l mt dy tng ngt.
By gi ta c mt dy mi
(
a2 = 2
an+1 = an 2 + an + 1 n > 2
Vn dng tnh cht trn, ta s chng minh an khng th l s chnh phng theo phng php
quy np n thun, v nu an l s chnh phng th n phi c tn cng l 1 trong cc s
0, 1, 4, 5, 6, 9
Ta thy r s hng trong dy c tn cng l 7 xut pht t n = 3: a3 = 7; a4 = 57
Gi s iu ny ng n n = k; tc ak c tn cng l 7.
Xt n = k + 1, ta c: ak+1 = ak 2 + ak + 1
Do ta ch xt ch s tn cng ca ak+1 nn ta s xt ch s tn cng ca tng s hng trong
biu thc trn.
Ta c ak c tn cng l 7nn ak 2 c tn cng l 9. Nh vy ch s tn cng ca

ak+1 = .b1 b2 ...9 + c1 c2 ...7 + 1 = d1 d2 ...7 9 + 7 + 1 = 6 + 1 = 7
iu ny cng ng vi n = k + 1, theo nguyn l quy np ta suy ra an c tn cng l7 n > 3.
p dng tnh cht trn ta suy ra an khng th l s chnh phng vi mi n > 3.
Mt khc a2 = 2 khng l s chnh phng.
Vy ta suy ra an khng l s chnh phng vi mi n > 2. 2

Bi tp 4.11. (Balkan 2002)


Cho dy s an tha mn
(

a1 = 20; a2 = 30
an+2 = 3an+1 an n > 1

Tm n A = 5an+1 an + 1 l s chnh phng.

Li gii

Tng t, ta i tm cng thc tng qut ca dy s v bin lun. i vi nhng


bi tm gi tr n mt biu thc no l s chnh phng trong dy s. Ta cn
lu n nhng tnh cht ca dy chnh cng nh dy con (nu c) nh n iu,
ng d. . .
D dng chng minh theo quy np cng thc tng qut ca an l
!n1
3
+
5
an = 10
+
2

!n1
3 5

53
Suy ra
A = 5an+1 an + 1

"
!n
!n1
!n #
3+ 5
3+ 5
3 5
A = 5.102 .
.
+
+
2
2
2

!2n1
!2n1
3+ 5
3 5
A = 500
+
+ 3 + 1
2
2

!4n2
!4n2

5+1
51
+
+ 3 + 1
A = 500
2
2

!2n1
!2n1 2

5+1
51
+ 501
A = 500
+
2
2
Xt dy ph sau:
(

x1 =
xn+2

!n1
3 5
+1
2

5; x2 = 6

= 5xn+1 xn n > 1

Dy trn c cng thc tng qut l


xn =

!n

5+1
+
2

!n

51
n N
2

Ta c:

!n
!n
!n1

5+1
51
5+1
xn xn1 =
+

2
2
2
!n2
!n2

5+1
51
=
+
>0
2
2

!n1

51
2

Suy ra xn > xn1 n N. Vy xn l dy tng ngt.


Tng t bi 2, dy trn c mt tnh cht c th chng minh c bng quy np theo 2 dy
song song l:
(
x2n = c
(c; d N)

x2n1 = d 5
2
Chnh v th m ta suy ra: A = 500 d 5 + 501.
Gi s n0 N : A = 5an0 +1 an0 + 1 l s chnh phng, ta c:
(50d)2 + 501 = k 2 (k N) 501 = (k 50d) (k + 50d)
Mt khc ta phn tch c 501 = 3.167, m
(
k + 50d + k 50d = 2k
k + 50d > k 50d

54
nn ta suy ra
(

k + 50d = 501

d=5

k 50d = 1

Vy n0 N : xn0 = 5 5. Ta tnh ton c x3 = 5 5 v xn l dy tng ngt nn ta suy ra


n = 3.
Vy vi n = 3 th A = 5an+1 an + 1 l s chnh phng. 2

Bi tp 4.12. Cho dy s tn tha mn:


(

t1 = 9; t2 = 25; t3 = 81
tn+3 = 7tn+2 14tn+1 + 8tn n > 1

a) Chng minh rng: vn = tn+1 2n+1 tn+1 + 22n n > 1 l s chnh phng.

b) Chng minh rng nu T = 2 + 2 12tn + 1 N th n l s chnh phng.


c) Gi s tn ti dy s xn sao cho xn .tn c l s c. Chng minh dy xn gm ton s chnh
phng.
Li gii
> Cu a:
Ta thy biu thc cn chng minh c dng cn thc. iu ny gi cho ta vic chng minh tn
l mt dy s chnh phng.
Bng vic th ln lt cc s hng u tin ca dy, ta rt ra nhn xt rng dy tn gm ton
s chnh phng. By gi vic ta cn lm l chng minh dy tn gm ton s chnh phng.
Thng thng nhng bi ton chng minh dy l s chnh phng hoc khng l s chnh
phng ta thng phi tm ra cng thc tng qut ca dy a v bi ton s hc thng
thng. Nhng trong dy s ta c th ch ng bin i linh hot gia chng to nn li
n
gii hp l. in hnh vi bi ton ny ta thy vic chng minh tn = 22 + 2n+1 + 1 gm ton
s chnh phng r rng l mt cng vic khng kh thi.
Song ta c th da vo phng php ca dy s v chng minh thng d on v chng
minh tn l bnh phng ca mt ng thc no .
Nh phng php sai phn ta c nh gi nh sau:
3

5
21

Hay

t1
21

9
22

23

t2
22

17

33
24

t3
23

t4

t5

24

2
Do , ta s chng minh theo quy np tn+1 = (2n + tn ) .
Do t1 = 9 = 32 nn iu trn ng vi n = 2 : t2 = (2 + 3)2 = 25.

55
2

Gi s iu ny ng vi n = k; tc tk = 2k1 + tk1 .
Ta xt n = k + 1; ta c:
p
2
tk+1 = 7tk 14tk1 + 8tk2 = 7tk 14tk1 + 8
tk1 2k2
p
2


tk+1 = 7tk 6tk1 8.2k1 tk1 + 8.22(k2) = 7tk 6 tk 2k1 8.2k1 tk 2k1 + 8.22(k2)
2

tk+1 = tk + 2.2k tk + 22k =


tk + 2k
Vy theo nguyn l quy np ta suy ra
tn+1 = 2n +

2
tn n N

Mt khc do t1 = 9 l s chnh phng nn t ta suy ra mi s hng ca dy u l s chnh


phng.

2
Vy vn = tn+1 2n+1 tn+1 + 22n = ( tn+1 2n ) l s chnh phng.
> Cu b:
Vic chng minh s chnh phng gip ta a bi ton v mt dng quen thuc hn:

Nu A = 2 + 2 12a2 + 1 N th n l s chnh phng".

Tht vy, gi s A N 12tn + 1 = k (k N) 12tn = (k 1)(k + 1)


Do k l nn ta suy ra k = 2l + 1 (l N). T ta suy ra: 3tn = l(l + 1).
Do (l; l + 1) = 1 v tn l s chnh phng nn ta suy ra
(
(
l = 3a2
l = b2
hoc
(a; b N)
l + 1 = b2
l + 1 = 3a2
(
* Trng hp 1:

l = b2
l + 1 = 3a2

3a2 = 1 + b2

.
Mt khc 3 = 4.0 + 3 nn t ta suy ra b2 + 1 c c dng 4k + 3 nn ta suy ra 1 .. 3 (v l).
Vy trng hp ny khng th xy ra.
(
* Trng hp 2:
Ta c:

l = 3a2
l + 1 = b2

A = 2 + 2 12tn + 1 = 2 + 2k = 2 (k + 1) = 4(t + 1) = 4b2

T ta suy ra A l s chnh phng.

Vy nu A = 2 + 2 12n2 + 1 N th n l s chnh phng.


> Cu c:
Mt tnh cht c bn ca s chnh phng: Nu a l mt s chnh phng bt k th n lun
c l s c v ngc li. (tnh cht c chng minh nh vic s dng php m).
Chng minh: Gi s a l s chnh phng, ta vit dng tng qut ca a l
a = p1 21 p2 22 p3 23 ...pn 2n

56
S c ca a chnh l s tch s ca cc s nguyn t vi cc ln thay i khc nhau ca cc
s m i , i = 1; n. Chnh v th m theo quy tc nhn ta m c a c
(21 + 1) (22 + 1) (23 + 1) ... (2n + 1)
s c v hin nhin tng s c trn l mt s l.
Ngc li gi s s a c l s c, ta c dng tng qut ca a l
a = p1 r1 p2 r2 p3 r3 ...pn rn
Theo quy tc nhn ta m c ac
(r1 + 1) (r2 + 1) (r3 + 1) ... (rn + 1)
Do a c l s c nn ri + 1, i = 1; n u phi l s l, suy ra
ri = 2k (k N) , i = 1; n
Vy a chnh l s chnh phng.
Tnh cht c chng minh hon tt. Quay li bi ton trn v s dng tnh cht ny
tm ra li gii thch hp cho bi ton.
Ta c: xn .tn c l s c. p dng tnh cht trn ta suy ra xn .tn l s chnh phng. Mt khc
tn l dy cc s chnh phng xn l dy cc s chnh phng.

Bi tp 4.13. Cho dy (un ) tha mn


(
u0 = 2; u1 = 5
un+1 = 5un 6un1 , n > 1
Xt hm s


x3 5x2 un

+ 6un 2 + 6n x + C (C R)
3
2
c o hm ti x0 (a; b) vi a; b N.
Chng minh rng f (x) c im cc i v cc tiu c honh lun l s nguyn.
f (x) =

Li gii
u tin, ta c nhn xt sau: 2 im cc tr ca hm s trn thuc phng trnh
f 0 (x) = x2 5xun + 6un 2 + 6n = 0
Nh th iu kin u tin nghim ca phng trnh trn l s nguyn th = un 2 4.6n phi
l s chnh phng vi mi n > 1. Nh vy ta a bi ton v vic chng minh vn = un 2 4.6n
l s chnh phng vi mi n > 1.
Trong bi ton ny, ta s chng minh bi ny bng phng php dng tam thc bc hai gii
quyt bi ton nhanh gn hn (ngha l ta s a bi ton v dng X 2 + Xun + + 6n = 0
do s 6n xut hin mt cch khng t nhin v mt c s hng un bc nht trong bi ton).

57
Nh vy dy s s c dng un+1 2 + un un+2 + 6n + = 0 (do dng ny ta c th tch un+2
theo cng thc truy hi v a phng trnh trn v dng nu trn). Ngoi ra ta vn c th
chng minh biu thc trn thng thng theo quy np ca mt biu thc on c.
Nh din t trn, ta c:
un+1 2 5un+1 un + 6un 2 + 6n + = 0
Xem biu thc trn nh phng trnh bc 2 theo tham s un n un+1 , ta c
= 252 un 2 24un 2 4.6n 4 = 2 un 2 4.6n 4
ng nht h s vi biu thc cn chng minh ta c = 1; = 0.
Nh vy ta s chng minh theo quy np
un un+2 un+1 2 = 6n , n N
Tht vy iu ny ng vi n = 0 do 2.13 52 = 60 = 1 v n = 1 do 5.35 132 = 6.
Gi s iu ny ng n n = k, tc uk uk+2 uk+1 2 = 6k .
Xt n = k + 1, ta c:
uk+1 uk+3 uk+2 2 = uk+1 (5uk+2 6uk+1 ) uk+2 2 = 5uk+1 uk+2 6uk+1 2 uk+2 2
uk+1 uk+3 uk+2 2 = 5uk+1 uk+2 + 6k+1 6uk uk+2 uk+2 2
=uk+2 (5uk+1 6uk ) + 6k+1 uk+2 2 = 6k+1 (do uk+2 = 5uk+1 6uk )
Vy iu ny ng vi n = k + 1, theo nguyn l quy np ta chng minh c
un un+2 un+1 2 = 6n , n N
Do , ta c:
u2n+1 un un+2 + 6n = 0 u2n+1 5un un+1 + 6u2n + 6n = 0 ()
Xem (*) nh phng trnh bc 2 theo tham s un n un+1 , ta c:
X 2 5un X + 6un 2 + 6n = 0
Do dy s trn l dy cc s nguyn nn ta suy ra X cng phi l s nguyn. Nh vy
= 25un 2 24u2n 4.6n = un 2 4.6n = vn
phi l s chnh phng.
Xt phng trnh tng qut
x2 5un x + 6un 2 + 6n = 0 (x R)
Phng trnh c nghim khi v ch khi = vn = un 2 4.6n > 0.

M vn l s chnh phng (chng minh trn) nn phng trnh ny lun c nghim v N.


Nh vy 2 nghim ca phng trnh trn ln lt l

5un un 2 4.6n
x=
2

58

+Vi un l cng nh un chn, th 5un un 2 4.6n lun l mt s chn nn hin nhin n


chia ht cho 2. Cho nn x lun l mt s nguyn.
Mt khc
Z
 x3 5x2 un

x2 5xun + 6n + 6un 2 =

+ 6un 2 + 6n x + C (C R) = f (x)
3
2
Mt khc hm s ch c th t cc i hoc cc tiu ti a, b, x0 nn t ta suy ra iu phi
chng minh. 2

Bi tp 4.14. Xt hm s f (t) = t +
(

 
t . Dy s (an ) c xc nh

a0 = m > 2, (m N)
an+1 = f (an ) , n = 0, 1, 2...

 

v dy (tb ) : tb = b 2 b N. Chng minh rng tn ti n0 ; b0 : an0 .tb0 Z.


Li gii
Vi iu kin bi, ta c th nhn ra rng tch s trn phi l s chnh phng. Nh vy hoc
an0 ; tb0 u l s chnh phng, hoc an0 = tb0 , hoc ch an0 .tb0 l s chnh phng. Vic ch ra
tn ti hoc an0 = tb0 , hoc ch an0 .tb0 l s chnh phng l cng on tnh ton vt v nht l
i vi hm phn nguyn. Nh vy ta s chng minh tn ti v s s hng l s chnh phng
khng nh s tn ti ca 2 bin c lp m khng cn ch ra c th iu kin ca 2 bin
bng vic tnh ton. Ta s tun t chng minh bi ton theo tnh cht ca hm phn nguyn
p dng khai trin Newton trong dy s cng nh lch ca cc hng t trong dy.
Xt dy (an ), ta c: m < f (m) < f (f (m)) < ... nn dy trn l dy tng ngt. Vy ta chia
lm hai trng hp sau:
> Trng hp 1: Nu m khng phi l s chnh phng.
Gi t2 l s chnh phng ln nht khng vt qu m = a0 . Theo cch xc nh trn ta suy ra

c d = [ m] .
t m = d2 + k. Ta c d2 < m 6 (d + 1)2 0 < k 6 2d + 1.
* a) Xt 0 < k < d + 1.Theo cch xc nh dy ta c:
 
d2 < a1 = f (a0 ) = m + m = d2 + k + d < (d + 1)2
hp
i
p

d < f (a0 ) < (d + 1) d =


f (a0 ) = [ a1 ]
Ta c:
 

a2 = f (a1 ) = a1 + [ a1 ] = m + m + [ a1 ] = m + 2d = (d + 1)2 + k 1
Nh vy ta thy s chnh phng ln nht khng vt qu a2 l (d + 1)2 v lch ca chng
gim i 1 sau 2 ln thc hin qu trnh trn.

59
p dng tng t vi phn t bt u l a1 . Qu trnh trn c lp li v lch gim dn
v 0 nn sau hu hn bc ta s gp s chnh phng.
* b) Xt d + 1 6 k 6 2d + 1 ta c:
a1 = f (m) = m +

 
m = d2 + k + d = (d + 1)2 + k d 1

Do 0 6 k d 1 < d + 1 nn ta c:
q
hp
i
p

a1 = f (m) = (d + 1)2 + k d 1 [ a1 ] =
f (m) = d + 1
Mt khc, ta li c:
a2 = f (a1 ) = f (m) +

hp
i
f (m) = d2 + k + 2d + 1 = (d + 1)2 + k

Vy (d + 1)2 l s chnh phng ln nht khng vt qu a2 v k chnh l lch ca n. Do


0 6 k < d + 1 nn ta quay li trng hp a) vi phn t bt u ca dy ny l a2 .
Nh vy, ta chng minh c sau mt hu hn bc th dy (an ) s gp phn t l s chnh
phng.
> Trng hp 2: Nu m l s chnh phng th hoc dy (an ) c v hn s chnh phng
hoc tn ti n0 N : an0 khng l s chnh phng. Khi y ta quay li trng hp 1 v sau hu
hn bc ta s gp li phn t ca dy l s chnh phng.
Do qu trnh trn lp li v hn ln nn ta suy ra dy trn c v hn s chnh phng.
Xt dy (tb ). Theo khai trin Newton, ta c:

l

2 + 1 = zl 2 + yl
khi l l

l

2 1 = zl 2 yl
(2 1)l = 2zl 2 yl 2 1 + yl 2 = 2zl 2
 2
yl 4 + yl 2 = yl zl 2
p

yl 4 + yl 2 = yl zl 2
Mt khc:
i
hp
p

2
4
2
4
2
yl < yl + yl < yl + 1
yl + yl = yl 2
2

Li c

hp
i h i
h i
4
2
yl + yl = yl zl 2 yl zl 2 = yl 2

 
Do yl zl 2 l mt dy con ca dy tb nn ta suy ra dy trn c v hn s chnh phng.
Nh vy
p
b0 , n0 N : an0 .tb0 Z an0 +1 = f (an0 ) = d2 + d = d (d + 1)
Vy ta c pcm. 2

60

Chuyn 5:
Mt s hm s hc v ng dng
L Phc L

Hm s hc f l hm nhn i s v gi tr trn mt tp ri rc l tp con ca


s nguyn, thng thng th ta xt f : N N. Cc hm s hc quen thuc v
gp nhiu ng dng nh: hm phn nguyn, hm phn l, hm tng cc ch s,
hm Euler... Cn cc hm tng cc c v hm s cc c r rng l cng c
gii thiu nhiu nhng phn ng dng ca n kh t. Bi vit di y lc dch
t mt chng trong cun Elementary Number Theory and Its Application ca tc
gi Kenneth Rosen. Mong rng vi mt s lng bi tp kh phong ph, chng ta
s c tip cn tt hn i vi hai hm s hc quen m cng l ny.

Hm tng cc c s v s cc c s
Kin thc cn nh
nh ngha v tnh cht. Vi mi s nguyn dng n, k hiu l tng cc c dng ca n (k
c n) v (n) l s cc c dng ca n (k c n).
Ta xt biu din ca n l
n = pa11 pa22 pa33 ...pakk
vi p1 , p2 , p3 , ..., pk l cc s nguyn t cn a1 , a2 , a3 , ..., ak l cc s nguyn dng. T y ta
quy c biu din ca n di dng ny.
Khi ta c:
pa1 +1 1 pa22 +1 1 pa33 +1 1 pkak +1 1
...
(n) = 1
p 1 1 p2 1 p3 1
pk 1
v
(n) = (a1 + 1)(a2 + 1)(a3 + 1)...(ak + 1)
Ch rng ta cng c th k hiu (n) =

d v (n) =

1.

d|n

d|n

Chng minh.
Ta thy rng mt s d = pb11 pb22 pb33 ...pbkk l c ca n khi v ch khi 0 6 bi 6 ai vi i = 1, 2, 3, ..., k..
Nh th gi tr ca bi c th c ai + 1 cch chn v nh th, theo nguyn l nhn, s c ca
1

Sinh vin i hc FPT

61

62
n chnh l . Tip theo, ta xt biu din
!
ai
k
Y
X



pji = 1 + p1 + p21 + ... + pa11 1 + p2 + p22 + ... + pa22 ... 1 + pk + p2k + ... + pakk
i=1

j=1

D thy khai trin ny c tt c (a1 + 1)(a2 + 1)(a3 + 1)...(ak + 1) v cc s hng l cc c


phn bit ca n. Do , i lng trn chnh l tng cc c ca n v c th thu gn thnh
(n) =

pa11 +1 1 pa22 +1 1 pa33 +1 1 pkak +1 1


...
p 1 1 p2 1 p3 1
pk 1

Nhn xt. Nu f l hm nhn tnh, ngha l f (m)f (n) = f (mn) vi mi (m, n) = 1 th ta c


P
F (n) = f (d) cng l mt hm nhn tnh.
d|n
P
Tht vy, gi s m, n l cc s nguyn dng nguyn t cng nhau th F (mn) =
f (d).
d|(mn)

R rng ta c th vit d = d1 d2 mt cch duy nht sao cho d1 l c ca m v d2 l c ca n


vi (d1 , d2 ) = 1. Khi , ta c
X
X
X
X
X
F (mn) =
f (d) =
f (d1 d2 ) =
f (d1 )f (d2 ) =
f (d1 )
f (d2 ) = F (m)F (n)
d|(mn)

d1 |m, d2 |n

d1 |m, d2 |n

d1 |m

d2 |n

Nhn xt c chng minh. T y, ln lt thay f (x) = x v f (x) = 1, ta suy ra c rng


cc hm (n) v (n) u l cc hm nhn tnh.

V d p dng
V d 1.
1. Tm tt c cc s nguyn dng n sao cho (n) ln lt bng 12, 18, 24, 48, 52, 84?
2. Tm s nguyn dng n nh nht sao cho (n) ln lt bng 1, 2, 3, 6, 14, 100?
Li gii
1. lm cc bi ny, ta cn chn cc s nguyn t v s m thch hp sao cho



1 + p1 + p21 + ... + pa11 1 + p2 + p22 + ... + pa22 ... 1 + pk + p2k + ... + pakk = (n)
Ta lit k cc s c dng 1 + pk + p2k + ... + pakk t nh n ln:
1 + 2, 1 + 3, 1 + 5, 1 + 2 + 22 , 1 + 7, 1 + 3 + 32 , 1 + 2 + 22 + 23 , ...
Vi (n) = 12, ta thy ch c th vit thnh 12 = (1 + 2)(1 + 3) v s cn tm l n = 6.
Cc s cn li thc hin tng t.
2. bi ny, ta cng chn cc s m thch hp trc ri tip n chn cc s nguyn t nh
cho gi tr ca n cng nh cng tt. Chng hn vi (n) = 14, ta c th chn ngay s m l
13 ng vi c nguyn t 2, tc l n = 213 . Tuy nhin, ta c th lm cho gi tr ny nh hn
bng cch vit 14 = 2 7, ng vi 2 s m 1 v 6, ta chn s n = 3 26 . y l s nh nht cn
tm.
Cc s cn li thc hin tng t.

63
V d 2.
1. Chng minh rng (n) l s l khi v ch khi n l s chnh phng.
2. Vi cc gi tr no ca n th (n) l s l?
Li gii
1. Ta thy rng (n) = (a1 + 1)(a2 + 1)(a3 + 1)...(ak + 1) nn nu (n) l khi v ch khi tt c
cc s m u phi l s chn, ngha l n l s chnh phng.
2. Ta cn chn cc c nguyn t v s m thch hp (n) l s l. Ta c



(n) = 1 + p1 + p21 + ... + pa11 1 + p2 + p22 + ... + pa22 ... 1 + pk + p2k + ... + pakk
nn nu c c nguyn t no l 2 th vn tha mn, nu n c c nguyn t l th trong
tng trn, chng phi xut hin chn ln. Do , s cn tm c dng n = 2k m2 vi m l mt
s chnh phng l.

V d 3. Mt mt khu hp l c di n, n > 6 gm 2 phn: 4 k t u l 1 trong 2 ch


s 0 hoc 1, n 4 k t sau l 1 trong 26 k t ca bng ch ci ting Anh nhng khng cha
ton ch A. Chng hn: 0100XYZ l mt mt khu c 7 k t, hp l nhng 0111AAA hay
0123ABC l cc mt khu c 7 k t nhng khng hp l.
a. Hi tng s mt khu hp l l bao nhiu? t s lng l sn .
b. Gi s tt c cc cch biu din sn thnh tng ca cc s nguyn dng bng nhau (c lng
s hng ty v c th ch gm 1 s hng) l tn . Chng minh rng tn chia ht cho 10.
Li gii
Cu a:
Xt mt khu c dng X1 X2 X3 ...Xn vi

X {0; 1} i = 1, 4
i
Xk {A, B, C, ..., Z} k > 4
Mi s X1 , X2 , X3 , X4 c 2 cch chn nn c tt c 2 2 2 2 = 16 cch chn 4 k t u.
Mi s Xk , 4 < k 6 n c 26 cch chn nn c tt c 26n4 cch chn nhng tr trng hp
ton b l A i nn c tng cng 26n4 1 cch chn n 4 k t sau.
Do , tng s mt khu hp l c th c l sn = 16(26n4 1).
Cu b:
D dng thy rng tn chnh l s c ca sn . Ta cng c (16, 26n4 1) = 1 nn c th vit sn
di dng sn = 24 p vi p l mt s nguyn dng l.
Do n > 6 nn n 4 > 2 v 26n4 chia ht cho 4, tc l 26n4 1 c dng 4k + 3, khng th l
mt s chnh phng, tc l n c s c chn.
Thm vo , 4 + 1 = 5 nn theo cng thc tnh s c ca mt s nguyn dng th tn va
chia ht cho 5 v va chia ht cho 2 nn n phi chia ht cho 10.
Ta c pcm.

64
V d 4.
a) Cho n l mt s t nhin tha mn n + 1 chia ht cho 24. Chng minh rng tng cc c
dng ca n (k ca n) cng chia ht cho 24.
b) Xt s nguyn
A = 20102011 20112010 20521994
Hy chng minh rng A l mt hp s dng v tng cc c s dng ca A chia ht cho 24.
Li gii
a) Gi d(n) l tng cc c dng ca n. Trc ht, ta s chng minh rng d(n) chia ht cho
3.

Tht vy, gi a l mt c no ca n th na cng l c ca n v nu xt 0 < a < n th



cc b a; na i mt khc nhau (do n + 1 chia ht cho 24 nn n chia 3 d 2 v n khng th
l s chnh phng).
V n chia 3 d 2 nn trong hai s a, na c mt s chia 3 d 1 v mt s chia 3 d 2; suy ra tng
ca chng phi chia ht cho 3. Do
X 
n  ..
d(n) =
a+
.3
a

0<a< n

Tng t, ta thy rng n + 1 chia ht cho 8 nn n chia 8 d 7. Do n l s l nn chia 8 c cc


s d l 1, 3, 5, 7; khi , d dng thy rng trong hai s a, na c mt s chia 8 d 1, mt s chia
8 d 7 hoc mt s chia 8 d 3, mt s chia 8 d 5.
D thy khi tng ca hai c ny cng chia ht cho 8.
T suy ra d(n) chia ht cho 8.
Kt hp hai iu trn li, ta thy rng tng cc c dng ca n chia ht cho 24.
b) Trc ht, ta s chng minh rng n l hp s dng. Tht vy, ta s chng minh rng nu
x > y > 3 th
y x > xy ()
Bt ng thc ny tng ng vi
x ln y > y ln x

ln y
ln x
>
y
x

t
Hm s f (t) = lnt t , t > 3 c f 0 (t) = 1ln
< 0 nn y l hm nghch bin, suy ra f (x) < f (y)
t2
hay (*) ng.
Cng bng cch dng hm s, ta c th chng minh rng vi n ln v 0 < a < n th c
nh gi
nn+a
>2
(n + a)n

Do 20102011 > 2.20112010 > 20112010 + 20521994 nn A > 0. Xt trong modun 5 th



A 1 + 21994 (mod5)

65
M
24 1(mod5) 21994 = 4 24

498

4(mod5)

Suy ra A chia ht cho 5 hay A l mt hp s.


Tip theo, ta s chng minh rng A + 1 chia ht cho 24. Tht vy, do 2010, 2052 chia ht cho
3 nn

A + 1 (1)2010 + 1 = 0(mod3)
Hn na 2052 chia ht cho 8 v
A + 1 32010 + 1 32

1005

+ 1 11005 + 1 = 0(mod8)

Suy ra A + 1 chia ht cho 3 v chia ht cho 8 nn A + 1 chia ht cho 24.


Theo kt qu cu a, ta c tng cc c dng ca A chia ht cho 24, suy ra pcm.

Bi tp c hng dn, gi
Bi tp 5.1. Chng minh rng phng trnh (n) = k c v s nghim nguyn dng n vi
mi k, cn phng trnh (n) = k th c hu hn nghim nguyn dng n.
Gi .
Ta thy rng gi tr ca (n) c s tham gia ca cc s nguyn t, cn gi tr ca (n) th
khng (iu ny c ngha l ta c th chn gi tr ca cc c nguyn t ca s n ln ty ).

Bi tp 5.2. Tch ca tt c cc c ca n bng bao nhiu? Chng minh rng hm tch cc


c ca n l mt n nh trn tp hp s nguyn dng.
Gi .

Ch rng cc c ca n c th chia thnh tng cp c dng d; nd v nh th khi nhn chng
li, ta s c mt gi tr n mi. Kt qu l n (n)/2 .
Cn vic chng minh tnh n nh th c th s dng phn tch thnh tha s nguyn t.

Bi tp 5.3. Tm tt c cc s nguyn dng n sao cho (n) + (n) = 2n, trong k hiu
(n) l s cc s nguyn dng khng vt qu n v nguyn t cng nhau vi n.
Gi .
C th s dng trc tip cng thc ca cc hm s trn ri dng bt ng thc chng minh
rng (n) + (n) > 2n hoc dng lp lun trc tip da theo ngha ca cc hm s.
Bi tp 5.4.
a. Chng minh rng s cp c th t cc s c BCNN ng bng n l (n2 ).
b. Chng minh rng vi mi s nguyn dng n th
(2n 1) > (n).
c. Chng minh rng s nguyn dng n l hp s khi v ch khi (n) > n +

n.

66
Gi .
a. Ta tnh s cch chn b (a, b) m [a, b] = n l (2a1 + 1)(2a2 + 1)...(2ak + 1) bng vic lp
lun trn cc s m tng t nh chng minh cng thc tnh (n).
.
b. Ch rng (2ab 1)..(2a 1) vi a, b Z+ nn d thy s c ca 2n 1 nhiu hn ca n.

c. Gi s n = ab vi 1 < a 6 b < n th (n) > 1 + a + b + n > 1 + n + n > n + n. Chiu


ngc li chng minh kh d dng bng vic kim tra cc s nguyn t.

Bi tp 5.5.
1. Chng minh rng
n
X

(i) = 2

i=1

T tnh tng

100
P

[ n] h i
X
n
i=1

 2
n

(i).

i=1

2. Cho a, b l cc s nguyn dng. Chng minh




(a) (b)
(ab)
(a)(b)
max
,
6
6
a
b
ab
ab
3. Chng minh rng nu a, b l cc s nguyn dng th (a)(b) =

X
d|(a,b)


d


ab
.
d2

Gi .
a. Ta s dng phng php quy np. Ch rng trong bc quy np, ta kim tra 2 trng
100
P
hp n l s chnh phng hoc khng. Tng
(i) = 482.
i=1

b. Ta chia thnh 2 bt ng thc b(a) 6 (ab) v (ab) 6 (a)(b) ri s dng trc tip cng
thc ca hm (n).
c. t a =
rng

pai i , b =

pibi v ci = min(ai , bi ) vi i = 1, 2, 3, ..., k. Trc ht, ta chng minh


ci
YX

pji (piai +bi 2j )

pi j=0

Sau xem xt cc tng c dng

c
P

X
d|(a,b)


d

ab
d2

(pa+bj + pa+bj1 + ... + pj ) vi c = min(a, b) v ch s

j=0

ln xut hin ca cc tha s nguyn t trong (a)(b).

Bi tp t gii
Bi tp 5.6.
a) Tm tng cc c dng ca cc s sau: 35, 2 3 5 7 11, 196, 25 34 53 72 11, 1000, 10!, 2100 .
b) Tm s cc c ca cc s sau 36, 99, 2 3 5 7 11 13, 99, 144, 20!.

67
Bi tp 5.7.
a. Hi cc s nguyn dng no c ng 2 c nguyn dng?
b. Hi cc s nguyn dng no c ng 3 c nguyn dng?
c. Hi cc s nguyn dng no c ng 4 c nguyn dng?

Bi tp 5.8. Cho s nguyn dng a. Xt dy s (un ) xc nh bi


(
u1 = a,
un+1 = (un ), n = 1, 2, 3, ...
Chng minh rng vi n ln th uN = 2, N > n.
Bi tp 5.9. S n c gi l highly composite nu (m) < (n) vi mi 1 6 m < n.
1. Chng minh rng vi mi k Z+ , tn ti s HC n tha mn k 6 n < 2k. T suy ra c
v hn s HC v nh gi chn trn cho s HC th k no .
2. Chng minh rng nu n l mt s HC th tn ti s nguyn dng k no sao cho ta c
biu din n = 2a1 3a2 5a3 ...pakk vi pk l s nguyn t th k v a1 > a2 > ... > ak > 1.
3. Tm tt c cc s HC c dng n = 2a 3b vi a, b l cc s nguyn dng.
Bit s square-free l s c dng n = p1 p2 p3 ...pk vi p1 , p2 , p3 , ..., pk l cc s nguyn t. Hy
tr li cc cu hi sau y.
Bi tp 5.10.
a. Hi trong cc s sau y, s no l s square-free?
A.44

B.50

C.10000000

D.95

b. Mt s square-free no c s cc c l A. Hi A c th nhn gi tr no?


A.25

B.30

C.1024

D.99

c. Cho tp hp B = {2, 3, 5, 7}. Hi c tt c bao nhiu s square-free c cc c nguyn t


thuc tp B?
A.15
B.16
C.17
D.14
Bi
tp 5.11.
2 Cho n l s nguyn dng. Chng minh cc ng thc sau:
X
X
a.
(n) =
( (d))3 .
d|n

d|n

b. (n ) =

(d)

vi (n) l s cc c nguyn t ca n.

d|n

c.

X n(d)
d|n

d (d).

d|n

Bi tp 5.12. Chng minh rng tn ti v s cp s (m, n) sao cho (m) = (n) nu nh


tn ti v s cp s nguyn t Mersense (s nguyn t dng 2n 1) hoc tn ti v s cp s
nguyn t sinh i.

68

Mt s hm s khc
Hm phn nguyn
Cc kin thc cn nh
Cc tnh cht ca phn nguyn:
[x] = x khi v ch khi x l s nguyn.
[x + n] = [x] + n vi n l s nguyn.
[x + y] 6 [x] + [y] vi mi s thc x, y.
[x] + [y] = x + y 1 vi x, y l cc s khng nguyn v x + y nguyn.
S cc s nguyn dng khng vt qu n v chia ht cho k l

n
.
k

Khai trin Legendre: s m ca s nguyn t p trong khai trin ca s nguyn dng n!


thnh tha s nguyn t l
+  
X
n
i=1

pi

     
n
n
n
=
+ 2 + 3 + ...
p
p
p

Cc bi tp p dng
Bi tp 5.13. Hi c bao nhiu s nguyn khng vt qu 2013 v chia ht cho 2 hoc 3 hoc
5?
Bi tp 5.14. Chng minh rng s cc s nguyn khng vt qu n v khng chia ht cho 2
hoc 3 l
h n i h n i h n i
n+

+
6
2
3
Bi tp 5.15. Chng minh rng vi mi s nguyn dng x, y th

(x+y)!
x!y!

l s nguyn.

Bi tp 5.16. . Phn l {x} chnh l i lng tnh bng x[x]. Chng minh rng 0 6 {x} < 1
v vi mi s thc x, y th {x + y} 6 {x} + {y}.
Bi tp 5.17. Cho x, y, z l cc s thc tha mn {x} + {y} + {z} = 2. Tnh gi tr ca biu
thc
P = [x + y + z] ([x] + [y] + [z])
Bi tp 5.18. Rt gn biu thc sau:
  2  3
 100 
21
2
2
2
+
+
+ ... +
A=
3
3
3
3


69

Hm tng cc ch s
Cc kin thc cn nh
K hiu S(n) l tng cc ch s ca n. Ta c cc tnh cht:
n S(n) chia ht cho 9.
S(m + n) 6 S(m) + S(n) vi m, n l cc s nguyn dng.
S(mn) 6 S(m)S(n) vi m, n l cc s nguyn dng.
+ h
X
n i
.
S(n) = n 9
10k
k=1

Bi tp p dng
Bi tp 5.19. Tm s nguyn dng n nh nht sao cho S(n), S(n + 1) u chia ht cho 7.
Bi tp 5.20. Khi vit 44444444 trong h thp phn th tng cc ch s ca n l A. Gi B l
tng cc ch s ca A. Tnh tng cc ch s ca B.
Bi tp 5.21. Chng minh rng vi mi s nguyn dng n th ta c cc bt ng thc:
a) S(2n) 6 S(n) 6 5S(2n).
b) S(5n) 6 S(n) 6 2S(5n).
Bi tp 5.22. Ngi ta gi cc s c th biu din thnh tng a + b vi S(a) = S(b) l cc s
p.
a. Chng minh rng cc s 999, 2999 khng phi l s p v tn ti v s s khng l s p.
b. Chng minh rng tt c cc s tha mn 999 < k < 2999 u l s p.

Hm Euler
Kin thc cn nh
Vi mi s nguyn dng n, k hiu (n) l s cc s nguyn dng khng vt qu n v nguyn
t cng nhau vi n. y c gi l hm Euler.
Ta c cc kt qu quen thuc:
(p) = p 1 vi p l s nguyn t.
(pk ) = pk1 (p 1) vi p l s nguyn t.
(mn) = (m)(n) vi (m, n) = 1.
Trong trng hp tng qut, ta c
(pa11 pa22 pa33 ...pakk )




 

1
1
1
1
=n 1
1
1
... 1
p1
p2
p3
pk

70

Bi tp tng hp
Bi tp 5.23.
1. Cho n l s nguyn dng ln hn 2. Chng minh rng (n) chn.
2. Vi gi tr no ca n th (n) = n2 ? (n) |n? (3n) = 3(n)? (n) chia ht cho 4?
3. Chng minh rng nu n c k c nguyn t l phn bit th (n) chia ht cho 2k .
4. Chng minh rng nu m, n Z+ v m|n th (m)|(n).
5. Chng minh rng hm f (n) = (n)
l hm nhn tnh y .
n
Bi tp 5.24.
1. Gi s m, n Z+ v (m, n) = p vi p nguyn t . Chng minh (mn) = p (m)(n)
.
p1
2. Chng minh rng
(mk ) = mk1 (m)
vi mi m, k Z+ .
3. Cho a, b Z+ , chng minh rng
(ab) = (a, b)

(a)(b)
,
((a, b))

t suy ra (ab) = (a)(b) vi (a, b) > 1.


Bi tp 5.25.
1. Chng minh rng nu phng trnh (n) = k vi k Z+ c nghim duy nht n th n chia
ht cho 36.
2. Chng minh rng phng trnh (n) = k vi k Z+ c hu hn nghim n.
3. Chng minh rng nu p l s nguyn t, 2a p + 1 l hp s vi a = 1, 2, 3, ..., r v p khng
phi l s nguyn t Fermat th
(n) = 2r p
v nghim.
4. Chng minh rng tn ti v hn s nguyn dng k sao cho phng trnh
(n) = k
c ng 2 nghim nguyn dng n.
Gi : xt cc s nguyn dng c dng k = 2 36j+1 , j = 1, 2, 3, ...
Bi tp 5.26.

1. Chng minh rng nu n 6= 2, n 6= 6 th (n) > n.

2. Chng minh rng n l hp s khi v ch khi (n) 6 n n.


3. Tm s nguyn dng n nh nht tha mn (n) > 105 .
Bi tp 5.27. Chng minh rng nu n l hp s v (n) |(n 1) th n l s square-free v n
l tch ca t nht 3 c nguyn t phn bit.

71
Bi tp 5.28. Cho s nguyn dng a. Xt dy s (un ) xc nh bi
(
u1 = a,
un+1 = (un ), n = 1, 2, 3, ...
Chng minh rng vi n ln th uN = 1, N > n.
Bi tp 5.29.
X
1. Chng minh rng
(d) = n vi mi s nguyn dng n.
d|n

2. Chng minh rng

X
16a6n,(a,n)=1

a=

n(n)
.
2

3. Gi s p > 4 l mt s nguyn dng v p 1, p + 1 l hai s nguyn t sinh i . Chng


minh rng 3(p) 6 p.
4. Chng minh rng iu kin cn v n l s nguyn t l (n) + (n) = n (n).

72

Chuyn 6:
Thng d bnh phng
Nguyn Huy Hong, Trn Hy ng

Thng d bnh phng l mt trong nhng cng c mnh gii quyt cc bi ton
s hc. Thng d bnh phng rt n gin, d hiu nhng li cc k hu dng.
Tuy vy, c gi cn c mt nn tng kin thc tt (nh v cp ca 1 s, nh l
Fermat,. . . ) c th s dng thnh thc.

Tnh cht c bn ca thng d bnh phng v k hiu Legendre


nh ngha 6.0.1.
Gi s p l s nguyn t l . Khi s nguyn a c gi l s chnh phng mod p nu
(a, p) = 1 v phng trnh ng d x2 a(modp) c nghim.
V d: -1 l s chnh phng mod 5 ( v 72 1(mod5) )
nh l 6.0.1.
Nu p l mt s nguyn t l th trong cc s 1, 2, 3, . . . , p 1 c ng
mod p.

p1
2

s chnh phng

Phn chng minh hai nh l trn xin dnh cho bn c.

nh ngha 6.0.2.
 
Cho a Z , p l s nguyn t l . K hiu Legendre ap c nh ngha nh sau:
Nu (a, p) = 1 v a l s bnh phng mod p th

 
1
a
= 1

0
Nu (a, p) = 1 v a khng l s bnh phng mod p th
(a, p) 6= 1

Lp chuyn Ton trng Ph Thng Nng Khiu v trng THPT Chuyn L Hng Phong

73

74
V
 d:
 Ta
 thy
 rng:
     
1
3
4
5
9
=
=
=
=
=1
11
11
11
11
11
         
2
6
7
8
10
=
=
=
=
= 1
11
11
11
11
11
nh l 6.0.2.
Gi s p l mt s nguyn t l , (a, p) = 1 . Khi :
 
p1
a
a 2 (modp)
p

Chng minh:
 
a
> Trng hp 1:
=1.
p
Khi phng trnh ng d x2 a(modp) c nghim x0 . Theo nh l Fermat nh, ta c :
a

p1
2

(x20 )

p1
2

= xp1
a (modp) (pcm)
0

 
a
> Trng hp 2:
= 1 .
p
Khi phng trnh ng d x2 a( mod p) v nghim . Vi mi i t 1 n p 1 , tn ti duy
nht j,1 6 j 6 p 1 sao cho i.j a(modp) .
R rng l i 6= j nn c th nhm cc s t 1 n p 1thnh p1
cp , sao cho tch trong mi
2
cp u ng d a mod p . Nhn tt c cc s 1, 2, 3, . . . , p 1, ta c :
(p 1)! a

p1
2

(modp)

Theo nh l Wilson , (p 1)! 1(modp), nh l c chng minh .


nh l 6.0.3.
Gi s p l 1 s nguyn
 t l , a v b l nhng s nguyn khng chia ht cho p. Khi :
a
b
1/ a b(modp)
=
.
p
p
    
b
ab
a
=
.
2/
p
 p2  p
a
= 1.
3/
p
Bn c d chng minh nh l 4 bng nh l 3.
nh l 6.0.4.
Vi mi s nguyn p > 2, ta c


1
p


(1)

p1
2

T y suy ra phng trnh ng d x2 1(modp) c nghim khi v ch khi p = 2 hoc


p 1(mod4)
 
.
V d: Ta d thy 1
= 1 p 1 .. 4.
p

75
nh l 6.0.5.
Gi s (x; y) = 1, a, b, c l cc s nguyn p l 1 c nguyn t ca ax2 + bxy + cy 2 , p khng
l c ca abc th D = b2 4ac l thng d bc 2 mod p.
c bit nu p l c ca x2 Dy 2 v (x, y) = 1th D l thng d bc 2 mod p.
Chng minh:
D bin i

p (2ax + by)2 Dy 2
Gi s p |y , khi p |2ax + by p |2ax
M (a, p) = 1 nn p |x . Vy (x, y) > 1 ( V l )
Do (p, y) = 1 nn tn ti y 0 sao cho yy 0 1(modp).
Suy ra
2
(2axy 0 + byy 0 ) Dyy 0 D(modp)
Vy D l thng d chnh phng mod p.

nh l 6.0.6. (B Gauss)
Gi s p l 1 s nguyn t l , a l s nguyn khng chia ht cho p . Nu trong s cc thng
d b nht ca cc s nguyn a, 2a, 3a, ..., p1
a c s thng d ln hn p2 th:
2
p1
 
 
2
P
[ 2ka
a
a
p ]
s
k=1
= (1) hay
= (1)
p
p

Chng minh:


Cho a l 1 s nguyn , p l 1 s nguyn t sao cho (a, p) = 1. Vi mi k 1; 2; 3; ...; p1
u
2


p1
tn ti rk 1; 2; 3; ...; 2 sao cho
ka rk (modp)
D thy khng tn ti hai rk c cng tr tuyt i , do




|r1 | , |r2 | , |r3 | , ..., r p1
2

p1
2

l 1 hon v ca 1; 2; 3; ...;
.
p1
Cho k chy t 1 2 ri nhn tt c cc v li vi nhau ta c :
a

p1
2

r1 r2 r3 ...r p1
2

1.2.3....( p1
)
2

r1 r2 r3 ...r p1
2
(modp)


|r1 | |r2 | |r3 | ... r p1
2

t bk =

rk
; bk = 1.Ta c
|rk |
a

p1
2

b1 ...bk (modp)

bk = 1 khi v ch khi phn d khi ly ka chia cho p ln hn p0 tc l ka = pq + r.


r > p1
khi v ch khi
2


 
2ka
2ka
2r
2ka
ka
= 2p +

= 2p + 1 = 2
+ 1 bk = (1)[ p ]
p
p
p
p

76
p1
 
2
P
[ 2ka
a
p ]
= (1) k=1
(pcm).
Vy a

p
T nh l trn ta c th thu c cc h qu sau.
p1
2

nh
 6.0.7.
 l
p+1
2
= (1)[ 4 ]
a/
p
b/-2 l thng d bc 2 mod p khi v ch khi p 1; 3(mod8)
c/ -3 l thng d bc 2 mod p khi v ch khi p 1(mod6)
d/ 3 l thng d bc 2 mod p khi v ch khi p 1(mod12)
e/ 5 l thng d bc 2 mod p khi v ch khi p 1(mod10)
nh l 6.0.8. Lut tng h
Nu p,q l cc s nguyn t l v p khc q th
  
p1
q1
p
q
= (1)( 2 )( 2 )
q
p

Bi tp v d

x ... p
2
2 ..
Bi tp 6.1. Chng minh rng vi mi s nguyn t p = 4k + 3, nu x + y . p th
..
y.p
Li gii
 2
   
y
y
x
..
,
6= 0, ta c
= 1.
Gi s x, y 6 . p
p
p
p     
 2
y
1
y2
1
Nhng theo nh l 4b, 4c v 5, ta c
=
=
= 1(v l!)
p
p
p
p

x ... p
Do ta c
..
y.p

m ... p
.
Bi tp 6.2. Chng minh rng vi mi s nguyn t p = 3k +2 th m2 +mn+n2 .. p
..
n.p
Li gii
.
Gi s m, n 6 .. p

  
n
m
,
6= 0.
p
p

Ta c

 
3
3n2
=1
=1
p
p

m ... p
.
Theo nh l 8c, ta suy ra y l iu v l, do m2 + mn + n2 .. p
..
n.p
Nhn xt :Cc tnh cht ny d dng chng minh c bng nh l Fermat, nhng y ch l
 .
.
.
m +mn+n .. p 4 m2 + mn + n2 .. p (2m + n)2 +3n2 .. p
2

77
mt ng dng kh c bn ca thng d bnh phng.
Cc nh ngha ca thng d bnh phng i khi c s dng chng minh mt s bi
ton kh th v. Chng ta ch cn p dng nhng tnh cht c bn lp lun nhng n vn
th hin c sc mnh ca thng d bnh phng.

Bi tp 6.3. Chng minh rng vi mi s nguyn t l p:


 

a = 1
+
p
a Z :

a < 1 + p
Li gii
Gi a l 
s t
nhin nh nht khng l thng d bc hai mod p.
p
t b =
+ 1 0 < ab p < a do hiu ab p l thng d bc 2 mod p. Vy
a

     
 
ab p
ab
a
b
b
1=
=
=
=
p
p
p
p
p
Suy ra b khng l thng d bc 2 mod p.
Do
p

a6b< +1a< p+1


a
Bi tp 6.4. (Korea Final 2000) Cho s nguyn t p = 4k + 1, hy tnh

p1 
X
2x2
x=1

x2
2
p




Li gii
u tin, ta c nhn xt:
[2x] 2 [x] 6 1x R
1
ng thc xy ra khi v ch khi {x} > .
2
Th nn, nhim v ca chng ta by gi l tm s ng d ca mt s chnh phng khi chia
p1
cho s nguyn t p = 4k + 1 sao cho n ln hn
.
2
p1
Theo tnh cht s 1 ca thng d bnh phng, ta c ng
s chnh phng mod p.
 
 2
  2  

1
a
a
a
pa
Bi v p = 4k + 1
=1
=1
=
=
.
p
p
p
p
p
p1
p1
Nh vy ,ta s c ng p1
s chnh phng mod p khng ln hn hn
v ng
s
4
2
4
chnh phng mod pln hn p1
.
2
Do

 2 
p1 
X
2x2
x
p1
2
=
p
p
2
x=1

78

i, j N

v nu 1 6 i, j 6 p 1 th i2 j 2 (modp).

i + j = p
Bi tp 6.5. (Iran TST 2004) Cho trc s nguyn t p v s nguyn dng k, chng minh
rng tn ti s nguyn dng n sao cho
  

n
n+k
=
p
p
Li gii



n (n + k)
Bi ton tng ng vi vic chng minh tn ti n sao cho
= 1.
p 

n (n + k)
Ta gi s iu ngc li, tc l gi s tn ti 1 6 k 6 p 1 sao cho
= 11 6
p
n 6 p 1.
V vi mi s nguyn t p bt k, c ng p1
s khng chnh phng mod p. Do nu
2
p1
f (n, k, p) = n (n + k) mod p nhn nhiu hn 2 gi tr phn bit th tn ti t nht 1 s chnh
phng mod p.
Vy tp gi tr ca f (n, k, p) nhn khng qu p1
phn t, do theo nguyn l Dirichlet, ta
2
c t nht ba s x, y, z nguyn phn bit sao cho
(
1 6 x, y, z 6 p 1
f (x, k, p) = f (y, k, p) = f (z, k, p)
Tc l

p|x + y + k

x (x + k) y (y + k) z (z + k) (modp) p|y + z + k x y z (modp)

p|z + x + k
iu ny v l v x, y, z phn bit v 1 6 x, y, z 6 p 1. Do ta c iu phi chng minh.
Nhn xt: R rng l trong hai bi ton ny, ta ch yu dng nh l s 1 v nh l s 2 lm
c s lp lun.
Thng d bnh phng cng rt hu dng trong gii phng trnh nghim nguyn, ta cng xt
n v d sau:
Bi tp 6.6. (Serbia-2008) Tm tt c nghim nguyn khng m ca phng trnh :
12x + y 4 = 2008z
Li gii
Nu z > 0 th y > 0.
Nu x chn th VT c dng a2 + b2 , nu x l th VT c dng a2 + 3b2 .

79
D thy 2008 c c nguyn t l 251 , a,b u khng chia ht cho 251 . T y ta c -1 hoc
-3 s l cc s chnh phng mod 251 .Tc l




1
3
=1
=1
251
251
M

1
251

= (1)

3  =
251

2511
2

3
251

= 1

= (1)

2511
2

251
3

2
3

= 1

Suy ra x = y = z = 0.

K hiu Jakobil
Khng dng li tp s nguyn t, cc tnh cht ca thng d bnh phng cn c th m rng
ra cho c hp s. Do khng s nhm ln nn tng t nh k hiu Legendre, k hiu Jakobil

c vit di dng na .
Mt s tnh cht ca k hiu Jakobil:
Vi n l s t nhin l v a nguyn, (a, n) = 1 ta c cc nh l sau:
nh l 6.0.9.
Vi mi a, b: a b (modn)

a
n

b
n

nh l 6.0.10.
Y
t n =
pi ai , vi pi l c nguyn t ca n, ta c:
 a  Y  a  Y  a ai
=
=
n
pi ai
pi
Hay ni mt cch khc

 a  a a 
=
m, n Z+
mn
n
m

Nhng lu : a l s chnh phng mod n khi v ch khi a phi l s chnh phng mod pi vi
mi pi l
t ca n. Ta s tin hnh chng minh hai bc:
 c
 nguyn
ai
a
a
1. pi ai = pi
    
a
a
2. pq
= ap
khi gcd (p, q) = 1
q
nh l 6.0.11. Ta c k hiu Jakobil l mt hm nhn tnh hon ton, bi v:
 a   b   ab 
=
a, b Z
n
n
n
 2
H qu: an = 1
nh l 6.0.12.


1
n


= (1)

n1
2

80
nh l 6.0.13.
 
(n1)(n+1)
2
8
= (1)
n
nh l 6.0.14.

m  n 
n

n1
m1
= (1)( 2 )( 2 )

Bi tp v d
.
Bi tp 6.7. Cho m, n Z+ sao cho n + 1 .. 4m, hy chng minh

m
n

= 1.

Li gii

Ta gi s iu ngc li l m
= 1.
n
V tnh chn l ca m khng quan trng cho nn ta c th coi nh m l s l.

.
u tin, ta c nhn xt: 1
= 1 k 1..4
k

.
= 1 theo nhn xt trn.
V n + 1 .. 4m 1

n 1 
m
m
Do n = n
= 1.
n
..
V n + 1.4m gcd (m, n) = 1 nn ta c th p dng lut nghch o bnh phng, ta c:
(m1)
 
1
(1) 2

= (1)
=
do
= 1
n
m
m
1
n



n
n
=
1

=
1

= 1.
Nu m = 4t + 1 1
m
m
m

.
Nhng v n + 1 = 12 + n .. 4m n
= 1, mu thun.
m

= 1 l khng th,
Vi trng hp cn li, ta cng d dng suy ra c iu v l, do m
n
suy ra pcm.

m  n 

(m1) (n1)
. 2
2

n

.
Bi tp 6.8. (Korea 1999) Tm mi s nguyn dng n sao cho 2n 1 .. 3 v tn ti s
nguyn m sao cho
2n 1 | 4m2 + 1
Li gii
.
.
.
Ta c nhn xt nu x2 + 1..p p = 4k + 1 v 2n 1..3 n .. 2.
iu kin cn:
Ta s chng 
minh s nguyn
 dng cn tm l mt lu tha ca 2.
.
.
.
Gi s n .. q q > 1, q 6 .. 2 , ta c: 2n 1 .. 2q 1.

2q 1 6 ... 3
Nhng
, do 2q 1 c c nguyn t dng 4k + 3 khc 3, v l.
..
q
2 +1 . 4
Vy ta c nu tn ti s nguyn m sao cho 2n 1|4m2 + 1 th n l mt lu tha ca 2.
iu kin :
k
Theo chng minh trn, ta c 22 1 khng c c nguyn t dng dng 4k + 3 khc 3, hn na

81
.
k
22 1 6 .. 9, ta phn tch thnh cc tha s nguyn t

2n 1
3

pai i

p dng k hiu Jakobil, ta c:



 
 Y   Y  ai
1
1
1
1
= Q ai =
=
=1
ai
2n 1
pi
pi
pi
3
Do , tn ti s nguyn t sao cho
. 2n 1
t2 + 1 ..
3
k

. 22 1
2 t + 1 + 2 1 ..
3
2k
. 2 1
k
22 t2 + 1 ..
3
2k
. 2 1
4u2 + 1 ..
3
2k 2

2k

Kt thc chng minh.


Bnh lun: Trong cun sch s hc ca Titu Andreescu th c li gii lin quan n s nguyn
t Fermat v nh l thng d Trung Hoa, nhng y khi s dng k hiu Jakobil, bi ton
ny tr nn n gin.

Khai thc mt b
By gi xin gii thiu c gi mt b rt th v sau:

Chng minh rng vi mi s nguyn dng a khng phi l s chnh phng, tn ti v s


s nguyn t p sao cho
 
a
= 1
p
Chng minh b
n
Q
Phn tch a thnh cc tha s nguyn t, ta c a = 2t
pai i , cho n gin, ta gi s s l s
i=1

square-free (tc l s khng c c l bnh phng ca mt s nguyn t) ai = 1, t {0; 1}.


Chn s l s khng chnh phng modulo pn .
S dng nh l thng d Trung Hoa, ta d dng tm c mt s nguyn v sao cho

v 1 (mod8)

v 1 (modpi ) (1 6 i 6 n 1)

v s (modp )
n

t v =

m
Q

qj , vi qj l c nguyn t ta c:

j=1
m  
Y
2
i=1

qi

 
v 2 1
2
=
= (1) 8
v

82
V theo nh l nghch o bnh phng, ta c:

m 
Y
pi
qj

j=1

m
Y

(pi 1)(qj 1)

(1)

j=1

qj
pi

 
v
=
1 6 i 6 n
pi

Do

m 
Y
a
j=1

qj

"
=


m 
Y
2
j=1

qj

Suy ra rng phi c qj sao cho

#2


m Y
n 
Y
pi
j=1 i=1

 
a
qj

qj

n  
Y
v

pi

i=1


=

v
pn


=

s
pn


= 1

= 1, m v c v s s nh vy nn ta c dpcm. Vi

trng hp a = 2 th bi ton hin nhin ng.


B ny lc u nhn vo th ta tng nh khng c ng dng nhiu, nhng trn thc t n
c th c nhng ng dng rt bt ng, in hnh l bi ton sau:
Bi tp 6.9. (CWMO 2011) Tm mi cp s nguyn (a, b) sao cho vi mi s nguyn dng
n, ta c
n | an + bn+1
Li gii
Chn n = p vi p l s nguyn t l ln, ta c
p

a +b

p+1

a + b 0 (modp)

a
p


= 1 p > p0

Vi p0 l s nguyn t l ln tho mn gcd (a, p0 ) = 1.


M theo b

 quen thuc l nu x khng l s chnh phng th tn ti v s s nguyn t l
x
p sao cho p = 1, do ta phi c a l mt s chnh phng.
Tng t, chn n = 2p vi p l s nguyn t l ln th ta c
 2p+1 
 
b
b
= 1 p > p0
= 1 p > p0
p
p
T y suy ra bl mt s chnh phng. Tip tc chn n = p vi p l s nguyn t l, t
a = k 2 , b = l2 ta c
k 2

p

+ l2

p+1 ..
.p l4 k 2 0 (modp) (theo nh l Fermat nh)

Do ta li c l2 k 0 (modp) vi v s s nguyn t p, v vy ta li suy ra tip (k)2 l


mt lu tha bc 4.
Tng t, chn n = 2p vi p l s nguyn t l th ta li c tip (l)2 l mt "lu tha bc 4.
a = b = 1
Lp li qu trnh ny v hn ln, ta c a, b l lu tha bc 2t tu , do
.
a = b = 0
Vy ta c (a, b) {(0, 0) , (1, 1)}.

83
Bi tp 6.10. (Mathlink contest 2004) Cho 2004 s nguyn khng m a1 , ..., a2004 sao cho
2004
X
ani l s chnh phng vi mi n. Tm s s hng nh nht bng khng.
i=1

Li gii
Chn s nguyn t l p sao cho p khng l c ca bt k s no trong cc s cho, p dng
nh l Fermat nh, ta c:
2004
X
i=1

ap1
i

 
k
k (modp)
=1
p

2004
P

ani l s chnh phng vi mi n.


i=1
 
V ch c hu hn p sao cho kp = 1 nn theo b trn, ta c k l s chnh phng, m
Do

k 6 2004 < 2025 = 452 k 6 442


Do , s s hng bng khng nh nht l 2004 1936 = 68.
Ta ch ra rng a1 = a2 = ... = a1936 = t2 , a1937 = ... = a2004 = 0 tho mn bi.

Bi tp ngh
1. (Bulgaria NMO 1998)Cho m, n l 2 s t nhin sao cho A =
Chng minh rng A l.

(m+3)n +1
3m

l s nguyn.

2. (Poland 2013) Cho cc s nguyn a,b sao cho 3 + a + b2 chia ht cho 6a. Chng minh
rng a m.
3. Chng minh rng 3n + 1khng c c nguyn t c dng 3k + 2 nu n l s nguyn dng
l.
4. Chng minh rng 2n 1lun c c nguyn t c dng 8k + 7 nu n l s nguyn dng
l.
5. (Vietnam TST 2003 P6) Cho s nguyn dng n. Chng minh rng 2n + 1 khng c
c nguyn t dng 8k 1.
.
6. Chng minh rng vi mi s nguyn t p, tn ti cc s nguyn x, y sao cho x2 + y 2 + 1..p.
7. (Korea Final 2000 P1) Chng minh rng vi mi s nguyn t , tn ti cc s nguyn
x, y, z v sao cho x2 + y 2 + z 2 = wpv 0 < w < p.
8. (IMO Shortlist 1998 N5)Tm mi s nguyn dng n sao cho tn ti s nguyn m
2n 1 l c ca m2 + 9.

84
9. (Czech-Polish-Slovak MO 2008) Chng minh rng tn ti s nguyn dng n sao cho
vi mi s nguyn dng k th k 2 + k + n khng c c nguyn t nh hn 2008.
10. (AMM) Tm mi s nguyn dng n sao cho 2n 1|3n 1.
11. Nu p = 2n + 1 (n > 2) l s nguyn t th chng minh rng
3

p1
2

.
+ 1 .. p

12. (Sierpinski) Chng minh rng ta khng th tm c s nguyn dng n > 1 sao cho
.
2n1 + 1..n.
13. (AoPS) Cho 3 s nguyn a, b, c, chng minh rng a, b, c, abc khng l s chnh phng
khi v ch khi tn ti v s s nguyn t p sao cho
     
a
b
c
=
=
p
p
p
14. (Iran TST 2013) C tn ti cc s nguyn a,b,c sao cho
.
a2 + b2 + c2 .. 2013(ab + bc + ca)
15. (IMO Shortlist 2009 N7) Cho a, b l hai s nguyn phn bit, chng minh rng tn
ti s nguyn dng n sao cho (an 1) (bn 1) khng l s chnh phng.

Cp v cn nguyn thy
Phm Tin Kha

Cp v cn nguyn thy l cc cng c ht sc hu dng trong vic gii cc bi


ton s hc s cp. Khng nhng th, chng cn l cu ni ta bc ln cc khi
nim, nh l cao cp trong ton hc. Bi vit ny xin trnh by mt s ng dng
ca cp v cn nguyn thy trong cc bi ton Olympiad.

Cp ca mt s nguyn dng
NH NGHA: Cho (a, n) = 1. S h > 1 c gi l cp ca a modulo n , k hiu h = on (a),
nu h l s nguyn dng nh nht tho mn
ah 1

mod n

T nh ngha v cp, ta rt ra c tnh cht quan trng sau:


Nu n|ak 1 th on (a)|k
Chng minh tnh cht trn n gin. t on (a) = h, v gi s k = lh + r. Ch rng:
ak ahl .ar ar 1

mod n

Do r < h nn theo nh ngha v cp, suy ra r = 0, hay h|k.


Ta bt u vi mt bi ton quen thuc v cp:
n

Bi 1. Chng minh rng mi c nguyn t l p ca s a2 + 1, trong a > 1 l s t


nhin bt k, u tho mn
p 1 mod 2n+1
n+1

Gii. T gi thit suy ra p|a2 1. t h = op (a) th h|2n+1 , v theo nh l Fermat nh th


h|p 1.
n
n
Gi s h < 2n+1 , tc l h|2n . Suy ra p|a2 1. Mt khc, theo gi thit th p|a2 + 1. Suy ra
p|2 (!). Do h = 2n+1 , hay
p 1 mod 2n+1 .
Bi ton ny quen thuc n ni n tr thnh mt b cho rt nhiu bi ton khc v cp.
Ta ln lt xt cc bi ton nh vy.
1

Lp 12CT THPT chuyn L Hng Phong

85

86

Bi 2. Chng minh rng tn ti v hn s nguyn t c dng 2n k + 1 vi n c nh.


Gii. R rng bi ton ny c lin quan mt thit, hay ni cch khc l mt h qu trc
n
tip ca Bi 1. Vic cn li ca ta l cn tm a dy xn = a2 + 1 c cc s hng i mt
nguyn t cng nhau.
n
Xt dy Fermat Fn = 22 + 1. Ta chng minh (Fn , Fm ) = 1 vi mi m 6= n.
Tht vy, ch ng thc sau:
Fn 2 = Fn1 F2 F0
T y d dng suy ra vi mi m 6= n th (Fn , Fm ) = 1, v bi ton cng c gii quyt.
n

Nhn xt: Dy Fermat Fn = 22 + 1 v i lng 22 rt ph bin trong cc bi ton v


cp, v ta s tip tc gp dy ny trong li gii cc bi ton sau.
Bi 3. Cho n > 1, a l s nguyn dng tho mn n|an + 1. Chng minh rng
(a + 1, n) > 1

Gii. Gi p l c nguyn t b nht ca n, v t on (a) = h. T gi thit suy ra h|2n


v h|p 1. Do tnh b nht ca p nn (n, p 1) = 1. Do
h|(2n, p 1) = (2, p 1) = 1, 2
Nu h = 1 th p|a 1, m p|an + 1 nn p = 2. Khi 2|(a + 1, n).
Nu h = 2 th p|a2 1, li theo nh ngha ca h nn suy ra p|a + 1. Khi p|(a + 1, n).Nhn xt: Nu p l s nguyn t v n|(p 1)n + 1 th p|n.
Bi 4. Tm tt c cp s nguyn dng (m, n) tho mn

n|2m1 + 1
m|2n1 + 1

Gii. Vi m = 1 th n = 1, 2 v ngc li. Ta xt trng hp m, n > 1.


t m 1 = 2a .x, n 1 = 2b .y, trong a, b, x, y nguyn dng v xy l. Gi p l mt c
nguyn t ca n th
a
p|2m1 + 1 = (2x )2 + 1
Theo Bi 1 th p 1 mod 2a+1 . Suy ra n cng c dng 2a+1 .k + 1, hay n 1 = 2a+1 k. Suy ra
b > a + 1.

87
Tng t, ta cng suy ra c a > b + 1, v iu ny mu thun vi b > a + 1.
Tm li, (m, n) = (1, 1), (1, 2), (2, 1).
Bi 5. Cho (a, b) = 1 l hai s nguyn dng khng ng thi bng 1, v p l s nguyn
n
n
t l tho mn p|a2 + b2 th
p 1 mod 2n+1

Gii. Li mt bi ton vi hnh thc gn nh tng t vi Bi 1. y ta ch cn mt bc


chuyn nh: ch rng (b, p) = 1 (nu khng th p|a v p|(a, b), tri gi thit) nn tn ti b0
tho mn
bb0 1 mod p
Do t gi thit suy ra
n

p|(ab0 )2 + 1
V ta bt gp li Bi 1. Bi ton c gii quyt.
Mt khi u nh nhng cho bi vit, v n lc y kh ca cc bi ton ln mt
bc. Nhng bi ton tip sau s cn nhng suy lun di hi v tinh t hn.
Bi 6. (Hn Quc 1999) Tm tt c s n nguyn dng sao cho tn ti m tho mn
2n 1
|4m2 + 1
3
Gii. R rng vi n l th 2n 1 2 mod 3 nn 2n 1 khng chia ht cho 3. Do n chn.
Ta c b quen thuc sau:
Cc s c dng x2 + 1 th khng c c nguyn t dng 4k + 3
Do n chn nn 2n 1 3 mod 4. Nu n c c nguyn t l th 2n 1 c c nguyn t p 6= 3
dng 4k + 3. Khi p|(2m)2 + 1, mu thun b .
Do n = 2k . Ta chng minh vi mi n nh vy lun tn ti m tho mn bi.
n
Ch k hiu v s Fermat Fn = 22 + 1. Nh vy
2n 1
= Fk1 Fk2 F1
3
Li do tnh i mt nguyn t cng nhau ca cc s Fermat nn theo nh l Thng d Trung
Hoa, tn ti c chn tho
i1
i
c2 22
mod 22 , i = 1, k 1
Chn m = 2c , ta c pcm.
Bn ang thc mc v bi ton trn, v n khng lin quan n cp? Hy tip tc theo di bi
ton sau.

88
Bi 7. Cho n l s tho mn tn ti m sao cho

2n 1
|4m2
3

Chng minh rng vi mi c nguyn dng d ca 2


cho
d 1 mod 2q

n 1

+ 1.
, lun tn ti q nguyn dng sao

Gii. R rng vi b l Bi 6, bi ton ny tr nn n gin hn rt nhiu. Theo kt qu


Bi th:
2n 1
= Fk1 Fk2 F1
3
Theo Bi 1 th vi mi i, mi c nguyn t pi ca s Fi u tho mn pi 1 mod 2i+1 . Do
vi mt c nguyn dng bt k d = pai11 pai22 paill , chn q = min {i1 , i2 , , il }, ta c:
mod 2q .

d1

Ta tip tc vi mt kt qu ca nh ton hc ngi Php Francois douard Anatole Lucas:


n

Bi 8. Cho n > 1. Nu p l s nguyn t tho p|Fn = 22 + 1 th


mod 2n+2

p1

Gii. y l mt m rng ca Bi 1. gii bi ton ny, trc ht ta cn mt kin thc v


thng d chnh phng:
Nu p l s nguyn t tho p 1 mod 8 th tn ti x x2 2 mod p
y l mt kt qu c bn, v trong khun kh bi vit ny xin khng nu chng minh y.
Tr li bi ton ca chng ta. Do n > 1 v p 1 mod 2n+1 nn p 1 mod 8 . Do tn ti
x
x2 2 mod p
Lu tha 2n hai v, ta c
n+1

x2

22 1

mod p

Suy ra
p|x2

n+1

+1

S dng kt qu Bi mt ln na, ta c pcm.


Nhn xt: M rng hn na th vi mi a tho mn tn ti x m a x2 mod p (hay a
n
thuc h thng d chnh phng ca p), trong p l c ca a2 + 1 th
p1

mod 2n+2

89
Ta tip tc n vi mt kt qu ht sc th v trong k thi chn i tuyn Trung Quc:
Bi 9. (Trung Quc TST 2005) Chng minh rng vi mi n > 2, c nguyn t ln nht
n
ca s 22 + 1 khng b hn (n + 1)2n+2 + 1.
n

Gii. t 22 + 1 = pk11 pk22 pkr r vi p1 < p2 < pr .


Ch rng pi 1 mod 2n+1 nn tn ti qi sao cho
pi = 1 + 2n+1 qi
n

Vit li h thc 22 + 1 = pk11 pk22 pkr r theo modulo 22n+2 , ta c:


11+2

n+1

r
X

ki qi

mod 22n+2

i=1

Suy ra
n+1

r
X

ki qi 6 qr

i=1

r
X

ki

i=1

Mt khc
n

22 + 1 > 1 + 2n+1

k1 +k2 ++kr

> 2(n+1)(k1 +k2 +kr )

Suy ra
r
X

ki <

i=1

Do
qr >

2n
n+1

2n+1
> 2(n + 1)
r
X
ki
i=1

Vy
pr > (n + 1)2n+2 + 1.

Nhn xt: Vi kt qu ca Bi 8, ta c mt pht biu mnh hn:


n

Vi mi n > 3 th c nguyn t ln nht ca 22 + 1 khng b hn (n + 2)2n+4 + 1.


Mt cu hi th v c t ra: gi tr b nht ca c nguyn t b nht ca cc s dng
n
22 + 1 l bao nhiu?
Tip theo, ta s i n mt nh l ht sc p v s nguyn t - nh l Dirichlet:
Nu a, b l hai s nguyn dng nguyn t cng nhau th dy a + b, 2a + b, 3a + b, . . . cha
v hn s nguyn t.

90
Chng minh nh l ny hon ton khng d, v phi s dng n cc kin thc ca ton
cao cp. Trong khun kh bi vit ny, ta ch xt n mt trng hp rt nh ca nh l
Dirichlet, khi b = 1 v a l s nguyn t.
Bi 10. (Hn Quc TST 2003) Cho mt s nguyn t p bt k, t
fp (x) = xp1 + xp2 + + x + 1
a) Chng minh rng nu p|m th mi c nguyn t ca fp (m) s nguyn t cng nhau vi
m(m 1).
b) Chng minh rng tn ti v hn n nguyn dng pn + 1 l s nguyn t.
Gii. a) Gi q l mt c nguyn t ca fp (m). R rng (q, m) = 1. Nu (q, m 1) 6= 1
th q|m 1. Khi 0 fp (m) m mod q, suy ra q|m (v l!).
b) Ta s chng minh tn ti v hn s nguyn t q m p|q 1.
Gi q l mt c nguyn t bt k ca fp (m). Suy ra q|mp 1. Do p l s nguyn t nn
oq (m) = 1 p.
Nu oq (m) = 1 th q|m 1, mu thun vi kt qu cu a. Do oq (m) = p, suy ra p|q 1.
Vic cn li l xy dng mt dy {mk }k>1 fp (mk ) chia ht cho v hn s nguyn t.
Xt dy m1 = p, mk = pfp (m1 )fp (m2 ) fp (mk1 ). Khi
fp (m1 )fp (m2 ) fp (mk1 )|fp (mk ) fp (0) = fp (mk ) 1
Suy ra fp (mk ) nguyn t cng nhau vi fp (m1 ), fp (m2 ), , fp (mk1 ).
Ta cng nhn mt m rng ca bi ton trn:
Vi mi a nguyn dng th dy a + 1, 2a + 1, . . . cha v hn s nguyn t.
Sau y ta s xt n mt vi ng dng ca nh l Dirichlet trong cc bi ton S hc.
Bi 11. Cho r l s nguyn t. t
fr (x) = xr1 + xr2 + + x + 1
Chng minh rng fr (x) chia ht cho v hn s nguyn t vi x l s nguyn t.
Gii. Gi s fr (p) ch chia ht cho hu hn s nguyn t q1 < q2 < qk
Theo nh l Dirichlet, tn ti l sao cho p = l.q1 q2 qk + 1l s nguyn t. Suy ra
fr (p) r 0

mod qi

vi i bt k thuc {1, 2, k}.


Suy ra r = qi . Hn na, fr (p) ch cha c nguyn t duy nht qi , v nu tn ti qj 6= qi m
qj |fr (p) th r = qj = qi , mu thun vi s phn bit ca qi v qj . t fr (p) = qis = rs . Ch
r|p 1. Xt
vr (fr (p)) = vr (pr 1) vr (p 1) = vr (p 1) + vr (r) vr (p 1) = 1

91
Suy ra fr (p) = r. Tuy nhin, chn p ln th fr (p) > r, mu thun. Ta c pcm.
Nhn xt: Kt qu bi ton trn tng qut hn so vi phn chng minh trong Bi 11. V ta
cng c th kt lun hm s fr (x) chia ht cho v hn s nguyn t vi mi x Z.
Bi 12. (A.Makowski) Cho k > 2 l s nguyn dng. Chng minh rng tn ti v hn
hp s n tho mn n|ank 1 vi mi a nguyn t cng nhau vi n
Gii. Chn n = kp vi p l s nguyn t. Ta cn p|ank 1 v k|ank 1. Chn n sao
cho p 1|n k, iu kin u tin c tho mn. Li chn n sao cho (k)|n k, iu kin
th hai c tho mn. Tm li, ta cn chn p sao cho (k)|p 1, v chng minh rng tn ti
v hn p nh vy. Tuy nhin y li l kt qu trc tip t nh l Dirichlet, v bi ton c
chng minh hon ton.
Bi 13. Cho k > 1 l s nguyn. Chng minh rng tn ti s nguyn t p v mt dy tng
nghim ngt a1 , a2 , . . . tho mn p + ka1 , p + ka2 , . . . l s nguyn t.
Gii. Theo nh l Dirichlet th tn ti v hn s nguyn t p tho p 1 mod k. Trc
ht chn mt s nguyn t p bt k nh vy, v t p = lk + 1
Khi p + kai = k(l + ai ) + 1. Vic tn ti mt dy tng nghim ngt a1 , a2 , . . . l hin nhin
theo nh l Dirichlet.
Bi 14. (nh l Hardy-Wright) Cho p l s nguyn t l. Chng minh rng lun tn
ti x, y nguyn dng sao cho
p|x2 + y 2 + 1
Gii. Nu p = 4k + 1 th 1 l s chnh phng mod p, tc tn ti x sao cho p|x2 + 1. Chn
y = p, ta c p|x2 + y 2 + 1.
Nu p = 4k + 3:
Ch rng (4p, 2p 1) = 1. Xt dy
1.4p + 2p 1, 2.4p + 2p 1, 3.4p + 2p 1, . . .
Theo nh l Dirichlet, tn ti l sao cho l.4p + 2p 1 l s nguyn t. Khi
l.4p + 2p 1 = (4l + 2)p 1 1

mod 4

Theo mt kt qu quen thuc: mi s nguyn t dng 4k + 1 biu din c di dng tng


ca hai s chnh phng, suy ra tn ti x, y
l.4p + 2p 1 = x2 + y 2
Khi p|x2 + y 2 + 1. Bi ton c chng minh hon ton.

92
Bi 15. Chng minh rng vi mi s nguyn t p v a nguyn dng, s ap 1 lun c t
nht mt c nguyn t q tho mn
q1

mod p

Gii. Ta xt trng hp a > p. t ap 1 = q11 q22 . . . qk k . Ta c oqi (a) = p 1 v oqi (a)|qi 1.


Gi s khng tn ti i sao cho oqi (a) = p. Khi vi mi j = 1, k th qj |a 1 . Mt khc:
ap1 + ap2 + + a + 1 p

mod qi , i = 1, k

Suy ra (ap1 + ap2 + + a + 1, qi ) = 1, qi 6= p. Do vqi (ap 1) = vqi (a 1) = i , qi 6= p.

Gi s p = qj . Suy ra ap 1|(a 1)qj j , hay ap1 + ap2 + + a + 1 6 qj j = pj 6 pp1 (v


l!). Trng hp cn li khi a 6 p th (p, qi ) = 1, i = 1, k. T tip tc gii nh trn, ta cng
c pcm.
Do , lun tn ti c nguyn t q ca ap 1 tho q 1 mod p.
Nhn xt: Vi cch l lun nh trn, ta cng c th thay p bi mt lu tha ca p. C
th thy bi ton ny gn ging vi Bi 1, tuy nhin thay v "vi mi c nguyn t" nh Bi
1 th y ta ch khng nh "tn ti mt c nguyn t". Dng tng qut ny c v yu hn,
nhng y li l bt ngun ta i n mt nh l mnh hn rt nhiu-nh l Zsigmondy:
Dng 1:
Vi mi a > b > 1 v (a, b) = 1 th an bn lun c mt c nguyn t p tho mn
p|an bn , nhng p 6 |ak bk vi mi 1 6 k < n.
(tr cc trng hp 26 16 v a2 b2 vi a + b l mt lu tha ca 2)
Dng 2:
Vi mi a > b > 1 th an + bn lun c mt c nguyn t p tho mn p|an + bn
nhng p 6 |ak + bk vi mi 1 6 k < n
(tr trng hp 23 + 13 )
Ta s khng cp n chng minh kh di ca nh l ny y, m ch xt mt s ng dng
ca n.
Bi 16. (Romanian TST) Chng minh rng dy an = 3n 2n khng c ba s hng no
lp thnh mt cp s nhn.
Gii. Gi s tn ti x < y < z sao cho
(3y 2y )2 = (3x 2x )(3z 2z )
T ng thc trn suy ra mi c nguyn t p ca 3x 2x u chia ht 3y 2y . Tuy nhin, do
x > y nn iu ny mu thun nh l Zsigmondy. Ta c pcm.

93

Bi 17. (IMO SL 2000) Tm tt c a, m, n nguyn dng tho mn


am + 1|(a + 1)n

Gii. Nu a 6= 3 th theo nh l Zsigmondy, am + 1 tn ti mt c nguyn t p m p 6 |a + 1,


mu thun vi yu cu bi.
Nu m = 3, a = 2 . Khi mi n > 2 u tho mn
23 + 1|(2 + 1)n

Nu a = 1 th chn m, n nguyn dng bt k.


Vy (a, m, n) = (2, 3, x + 2) vi x N hay (1, m, n). 
Bi 18. (Nht Bn 2011) Tm a, n, p, q, r nguyn dng tho mn
an 1 = (ap 1)(aq 1)(ar 1)

Gii. Gi s p > q > r, Nu a, n khng ri vo cc trng hp ngoi l ca nh l Zsigmondy,


ta d dng suy ra iu v l.
Nu a = 2, n = 6, suy ra p = 6, q = r = 1.
Nu n = 2 th khng tn ti a, p, q, r.
Nu a = 1 th chn n, p, q, r nguyn dng bt k.
Tm li, (a, n, p, q, r) = (1, n, p, q, r), (2, 6, 6, 1, 1), (2, 6, 1, 6, 1), (2, 6, 1, 1, 6).
Bi 19. (Iran) Cho A l mt tp hu hn cc s nguyn t v a > 1 l s nguyn dng.
Chng minh rng tn ti hu hn n sao cho tt c cc c nguyn t ca an 1 u thuc A.
Gii. Gi s A = {p1 , p2 , , pk }. Ta xt N = [p1 1, p2 1, , pk 1]. R rng
pi |aN 1, i = 1, k
Khi , theo nh l Zsigmondy, vi mi n0 > N , s an0 1 lun c mt c nguyn t q khng
phi l c ca aN 1, hay q 6 A.
Suy ra nu n l s tho iu kin bi th n 6 N , tc l ch c hu hn n nh vy.
Bi 20. (Ba Lan) Cho 2 < q < p l hai s nguyn t l. Chng minh rng 2pq 1 c t
nht ba c nguyn t phn bit.
Gii. Theo nh l Zsigmondy, 2pq 1 c c nguyn t r m r 6 |2p 1 v r 6 |2q 1.
Li theo nh l Zsigmondy, 2p 1 c c nguyn t s m s 6 |2q 1.
Hn na, nu tt c cc c nguyn t ca 2q 1 u chia ht 2p 1 th q|p, v l v p nguyn

94
t. Do , 2q 1 cng c mt c nguyn t t khng chia ht 2p 1.
Ba c nguyn t r, s, t chnh l ba c cn tm.
Bi 21. Cho a N . Gi A l tp hp cc s t nhin n sao cho n|an + 1, v B l tp
tt c cc c nguyn t ca cc phn t trong A. Chng minh rng B hu hn.
Gii. Trng hp n l:
Gi B1 l tp hp cc c nguyn t ca cc phn t l trong A.
T gi thit suy ra n|a2n 1. Gi p l c nguyn t b nht ca n v t h = op (a). Khi
h|2n v h|p 1. Suy ra h|(2n, p 1) = (2, p 1) = 1 2. Suy ra p|a2 1, hay B1 hu hn.
Trng hp n chn:
Gi B2 l tp hp cc c nguyn t ca cc phn t chn trong A.
n 2
t n = 2n1 th n1 l (nu khng th 4|n| a 2 + 1, v l). T l lun tng t trng hp
n l, ta cng suy ra B2 hu hn.
Do B1 , B2 hu hn nn B = B1 B2 cng hu hn.
Nhn xt: Vi cch l lun tng t, ta c bi ton quen thuc (v c bn) sau y:
Nu n|2n 1 th n = 1
Tuy vy, tht bt ng rng bi ton n gin ny cng c mt ng dng ht sc th v:
Bi 22. (Ba Lan) Tm tt c a thc vi f (n) c h s hu t tho mn
f (n)|2n 1, n N

Gii. Ch rng f (n)|f (n + f (n)) vi mi n. Suy ra


f (n)|f (n + f (n))|2n+f (n) 1
Suy ra

f (n)|2n+f (n) 2n = 2n 2f (n) 1
Do (2n , f (n)) = 1 vi mi n nn
f (n)|2(f (n) 1
Suy ra f (n) = 1, n N.
Nhn xt: Mt bi ton "c v tng t" dnh cho bn c:
Tm tt c a thc h s nguyn f (n) tho mn f (n)|2n + 1, n N
21

Bi 23. Tm s nguyn t p nh nht sao cho tn ti a tho mn p|a2 a v p 6 |a2 a,


1 6 k < 21.

95

21 1

Gii. R rng (a, p) = 1 nn p|a2

1. t h = op (a). Khi h tho h iu kin sau:

21

h|2 1

h 6 |2k 1, 1 6 k < 21

h|p 1
Vi mi 1 6 k < 21, ta c mt b quen thuc
(221 1, 2k 1) = 2(21,k) 1 6 27 1 = 127
Do vi mi p > 127 th iu kin h 6 |2k 1, 1 6 k < 21 c tho mn. Chn h = 49, khi
gi tr nh nht ca p l 197.
Tm li, gi tr nh nht ca p l 197.
Nhn xt: Bn c hy gii bi ton tng t sau:
Tm p nguyn t nh nht sao cho tn ti a tho

p|a215 +1 1
p 6 |a2k +1 1, 1 6 k < 15
Bi 26. (Fermat) Cho p > 3 l s nguyn t. Chng minh rng mi c dng ca
c dng 2kp + 1.
Gii. Trc ht nhn thy rng

2p +1
3

v3 (2p + 1) = v3 (2 + 1) + v3 (p) = 1
p

nn mi c nguyn t ca s 2 3+1 u ln hn 3. Gi q l c nguyn t bt k ca 2 3+1 . Khi


q|2p + 1, suy ra q|22p 1. Gi h = oq (2). Ta c h|2p, suy ra h = 1 2 p 2p. Nu h = 1
th q|1. Nu h = 2 th q|22 1 = 3. Nu h = p th q|2p 1, suy ra q|h. Tt c cc trng hp
ny u v l. Do h = 2p. Suy ra h = 2p|q 1 hay q 1 mod 2p. T y d dng kt lun
p
rng mi c dng ca 2 3+1 u c dng 2k + 1.
Nhn xt: Bi ton c th c tng qut nh sau:
Cho p, q > 2 l cc s nguyn t phn bit. Khi tt c c dng ca s
(q1)p +1
u c dng 2kp + 1.
q
Cho p, q > 2 l cc s nguyn t phn bit. Khi tt c c dng ca s
(q+1)p 1
u c dng 2kp + 1.
q
Mt s bi ton th v pht trin t bi ton ny:
F Cho p > 2 l s nguyn t. S nguyn dng a tho t nht mt trong hai iu kin

96
sau:
i) a l bi s ca 4;
ii) a l v (a, p) = 1.
Chng minh rng mi c dng ca

(a2)p +2p
a

(a1)p +1
a

u c dng 2kp + 1.

F Cho s nguyn t p > 3. Chng minh rng mi c dng ca


k p mod 4 nu c l c nguyn t.

2p +1
3

u c dng 2kp + 1, v

Bi 27. (VN TST 1997) Chng minh rng tn ti mt hm s f nhn gi tr nguyn tho
mn
2n |19f (n) 97
vi mi n nguyn dng.
Gii. D thy rng
v2 (192

n2

1) = v2 (19 1) + v2 (2n2 ) = n

nn o2n (19) = 2n2 .ngSuy ra 19t cha 2n2 thng d phn bit trong h thng d ca 2n .
Mt khc, rng 19t ch cha cc thng d l, c th 19t 1, 3 mod 8, m 2n2 l mt
na s thng d l ca 2n , suy ra 19t cha tt c thng d ng d vi 1, 3 mod 8 ca 2n .
Do 97 1 mod 8, suy ra tn ti f 19f (n) 97 mod 2n .
Bi 28. (Italian TST 2006) Cho s nguyn dng n. Gi An l tp tt c a Z, 1 6 a 6 n
tho mn n|an + 1
a) Tm tt c n An 6= .
b) Tm tt c n |An | chn v khc khng.
c) Liu c tn ti n |An | = 130 khng?
Gii. a) Vi n l, ta chn a = n 1.
Xt n chn, t n = 2n1 . Khi 2n1 |(an1 )2 + 1. Suy ra n1 l v n1 ch c c nguyn t dng
4k + 1. Tm li, An 6= khi n l hoc n = 2n1 , trong n1 l v ch gm cc c nguyn t
dng 4k + 1.
b) Xt n > 2 chn. Khi nu n|an + 1 th n|(n a)n + 1 (ch rng a 6= n1 ). Do
nu n chn th |An | chn.
Xt n l. Ch rng (n) chn vi mi n > 2, suy ra (n) 2 cng chn.
R rng (a, n) = 1. Xt a 6= 1, n 1 tho n|an + 1. Do (a, n) = 1 nn tn ti 2 6 b 6 n 2 tho
ab 1 mod n. Li c n|an + 1, suy ra n|bn + 1. iu ny c ngha l |An \ {n 1} | chn, hay
|An | l.
Tm li, |An | chn khi n chn v n > 2.
c) Gi s tn ti n tho |An | = 130. Theo cu b th n chn. t n = 2p1 1 p2 2 pk k , trong
pi 1 mod 4 vi mi i = 1, k. Ta chng minh b sau:
Cho s nguyn t p. Phung trnh x2 1 mod p hoc khng c nghim, hoc c

97
ng 2 nghim modulo p.
Xt x tho x2 1 mod p. Gi s tn ti y 6= x tho mn y 2 1 mod p. Khi x2 y 2
mod p, suy ra x y mod p, hay y = p x.
Tr li bi ton. Do pi u c dng 4l + 1 nn 1 l s chnh phng mod pi i . Theo b
trn th s nghim theo modulo n ca phng trnh x2 1 mod n l 2k . Suy ra khng tn
ti n |An | = 130.
Bi 29. (IMO 2003) Chng minh rng vi mi s nguyn t p, lun tn ti s nguyn t
q khng phi l c ca np p vi mi n > 1.
Gii. Vi p = 2 th q = 5. Xt p l. Ta c
pp 1 = (p 1)(1 + p + p2 + + pp1 )
Ch rng 1 + p + p2 + + pp1 khng ng d 1 mod p2 , nn s c mt c nguyn t
q khng c dng kp2 + 1. Ta chng minh q tho iu kin bi. suy ra q|p 1. Suy ra
0 1 + p + p2 + + pp1 p mod q, hay p = q, v l.
Bi 30. Tm cc s nguyn k, m, n > 0 sao cho k m |mn 1 v k n |nm 1.
Gii. Gi p > 3 l c nguyn t ca k. Gi s m > n. t d = op (m). Suy ra
n
n
d
= op (md 1)
vp (m 1) = vp (m 1) + vp
d
Suy ra pm |md 1. Suy ra pm 6 md 1 6 mp1 1 < mp . (1)
Mt khc, k m 6 mn 1 < mm 1 < mm . Suy ra p < k < m.
Nu p > 3 th m > 3. Khi hm s f (x) = logx x ng bin trn [3, +), suy ra
hay pm > mp , mu thun (1).
Suy ra p = 2. Suy ra m, n l.
Nu n = 1 d dng suy ra m = 1. Khi k bt k.
Xt m > n > 1. Ta c 4|k m |mn 1, m n l nn m 1 mod 4. Suy ra

m
log m

>

p
,
log p

v2 (mn 1) = v2 (md 1)
Suy ra 2m |md 1, suy ra 2m 6 md 1 = m 1, v l.
Tm li, (m, n, k) = (1, 1, k), (m, n, 1).

Cn nguyn thy
NH NGHA: S nguyn dng a c gi l cn nguyn thy ca n nu on (a) = (n).
Lu rng khng phi s nguyn dng n no cng c cn nguyn thy. Ta chng minh
c rng n c cn nguyn thy khi v ch khi n = 2, 4, pk , 2pk vi p l s nguyn t l. Sau

98
y ta xt mt s bi tp v cn nguyn thy.
Bi 1. Chng minh rng 2 l cn nguyn thy ca 3n vi mi n > 1.
Gii. Ta chng minh bng quy np. D thy khi n = 1, kt lun ca bi ton l ng. Gi s
k
k1
kt lun trn ng vi n = k, tc l 2(3 ) 22.3 1 mod 3k . Gi d l bc ca 2 mod 3k+1 .
Do 2d 1 mod 3k+1 nn 2d 1 mod 3k , suy ra 2.3k1 |d. Mt khc, d|3k+1 = 2.3k . Suy ra
d = 2.3k1 hay d = 2.3k . Ta chng minh b sau bng quy np:
n1

22.3

1 + 3n mod 3n+1

B ng vi n = 1. Gi s b ng vi n = k, suy ra
22.3

k1

= 1 + 3k + 3k+1 .m

Lp phng hai v
k

23 = 1 + 3k+1 + 3k+2 .M
Suy ra
k

22.3 1 + 3k+1

mod 3k+2

Theo nguyn l quy np, b c chng minh. Suy ra d = 2.3k . 


Nhn xt: Ta cng chng minh c bi ton sau: 2 l cn nguyn thy ca 5n vi mi
n>1
Bi 2. Chng minh rng nu n = 3k1 th 2n 1 mod 3k .
Gii. Theo Bi 1 th 2 l cn nguyn thy ca 3k , suy ra bc ca 2 mod 3k l 2n. Suy ra
k1
22n 1 (2n 1)(2n + 1) 1 mod 3k . Mt khc, 2n 1 (1)3 1 1 mod 3, suy ra
2n + 1 1 mod 3k .
Bi 3. Cho n > 2 v p = 2n + 1. Chng minh rng nu 3
nguyn t.
n1

p1
2

+ 1 0 mod p th p l s

Gii. Do p1
= 2n1 nn 32
1 mod p, suy ra 32 1 mod p, suy ra bc ca 3 mod p
2
l 2n , hay p1. Suy ra p1|(p), hay p1 6 (p). Suy ra (p) = p1, hay p l s nguyn t. 
Bi 4. (AMM) Xt f (n) l c chung ln nht ca 2n 2, 3n 3, 4n 4, . . . Xc nh f (n)
v chng minh rng f (2n) = 2.
Gii. Gi p l mt c nguyn t bt k ca f (n). D thy vp (f (n)) = 1, do p||pn p. Gi a l
cn nguyn thy ca p (do p nguyn t nn tn ti a nh vy). Do (a, p) = 1 v p|an a, suy
ra p 1|n 1. Vi p 1|n 1 th p|mn k vi mi s nguyn dng m. Do hm f c
xc nh nh sau

99
f (n) = p1 p2 pk vi pi 1|n 1, i = 1, k
Xt f (2n). Theo cch xc nh trn th nu f (2n) c c nguyn t q > 2 th q 1 l mt c
chn ca 2n 1, v l. Do f (2n) = 2.
Bi 5. Cho p > 2 l s nguyn t. Tm tt c s nguyn dng k sao cho
p|1k + 2k + + (p 1)k

Gii. t Sk = 1k + 2k + + (p 1)k . Vi k l bi ca p 1, ta c
Sk p 1 6 0

mod p

Ta chng minh rng vi mi k khng phi l bi ca p 1 th p|Sk .


Tht vy, gi a l cn nguyn thy ca p. D dng chng minh rng {0, a1 , a2 , . . . , ap1 } l h
thng d y mod p. Suy ra (a1 , a2 , . . . , ap1 ) l mt hon v ca (1, 2, . . . , p 1).
Suy ra
k
k
k
1k + 2k + + (p 1)k a1 + a2 + ap1

ak 1 + ak + a(p2)k
a(p1)k 1
a1
0 mod p.
ak .

Vy vi mi k khng phi l bi ca p 1 th p|Sk . 


Nhn xt: Bi ton trn c mt ng dng c sc nh sau:
Cho a, b, c l cc s nguyn v p l s nguyn t. Chng minh rng tn ti cc s nguyn x, y, z
khng ng thi chia ht cho p sao cho p|ax2 + by 2 + cz 2 .

TI LIU THAM KHO


1.
2.
3.
4.

Titu Andreescu, Gabriel Dospinescu, Prolems from the book.


Naoki Sato, Number theory.
H Duy Hng, Mt s phng php gii ton s hc s cp.
Cc din n AoPS, MathScope.

100

Chuyn 8: Hm phn nguyn v phn l


Lu Giang Nam

Phn nguyn l mt lnh vc hay v c o ca ton s cp, cao cp v ng


dng.C nhiu bi ton hay v phn nguyn c s dng lm thi hc sinh
gii cc cp, trong c rt nhiu cc thi hc sinh gii quc gia v Olympic quc
t. Mt khc, hm phn nguyn c nhng ng dng quan trng khng ch trong
ton hc ph thng, m cn trong nhiu vn ca ton ng dng v cng ngh
thng tin (lm trn s, tnh gn ng,...). Phn nguyn cng th hin s kt ni
gia tnh lin tc v tnh ri rc, gia ton gii tch v ton ri rc nn kh th v.
Tuy nhin v s gii hn ca kin thc nn trong bi vit ny tc gi xin trnh by
cc tnh cht, ng dng ca phn nguyn trong phm vi THPT.

nh ngha, tnh cht v bi tp c bn


nh ngha
Phn nguyn ca s thc x, k hiu [x], l s nguyn ln nht khng vt qu x.
Phn l ca s thc x, k hiu {x}, l phn cn li ca x b phn nguyn: {x} = x [x].
Gi tr nh nht gia hai s x z v x + 1 z c gi l khong cch t x n s nguyn gn
n nht v c k hiu l ]x[.
S nguyn gn mt s thc x nht c k hiu l (x) v c gi l s lm trn ca x

Cc tnh cht quen thuc


1. x = [x] nu x Z
2. x = {x} nu 0 6 x < 1
3. x 1 < [x] 6 x
4. Nu k Z th [x + k] = [x] + k, {x + k} = {x} + k
5. [x + y] [x] [y] {0; 1} suy ra [x + y] > [x] + [y], {x + y} 6 {x} + {y}
  h i
x
[x]
=
,nN
6.
n
n
1

Chuyn Phan Ngc Hin C Mau

101

102
7. S cc s nguyn dng chia ht cho n khng vt qu x l

8.

hxi
n

x
x+a
= , nu n
+
b
b

9. x > y [x] > [y]


10. Nu [x] = [y] th |x y| 6 1
11. Trong hai s x v y c mt s nguyn v mt s khng phi l s nguyn th 0 <
{x}+{y} < 1
12. Vi mi x v y l cc s thc ta c [2x] + [2y] > [x] + [y] + [x + y] > 2([x] + [y])
13. Nu max{{x}, {y}} <

1
2

th

[2{x}] + [2{y}] = 0 = [{x} + {y}] v [2x] + [2y] + [x] + [y] + [x + y] = 2[x] + 2[y]
14. Nu min{{x}, {y}} <

1
2

6 max{{x}, {y}} 6 2[x] + 2[y] th

[2{x}] + [2{y}] = 1 = [{x} + {y}] + 1 v [2x] + [2y] = [x] + [y] + [x + y] = 2[x] + 2[y] + 1
15. Nu

1
2

6 min{{x}, {y}} th

[2{x}] + [2{y}] = 2 = [[{x} + {y}] + 1 v [2x] + [2y] = [x] + [y] + [x + y] + 1 = 2[x] + 2[y] + 2
16. Vi x R ta c





1
1
{x} +
= [2{x}] v x +
= [2x]-[x]
2
2
hni n + 1
H qu : Vi mi s nguyn dng ta lun c
+
=n
2
2

17. Vi mi s t nhin n v vi mi s thc x R ta c n[x] 6 [nx] 6 n[x] + n 1


18. Vi mi s thc x khng phi l s nguyn v vi mi s nguyn n ta lun c [x]+[nx] =
n1
19. Cho k1, k2,.., kn, l b n s nguyn dng. Khi y


k1 + k2 + ... + kn
k1 + k2 + k3 + ... + kn >
+n1
n
20. Cho a v b > 2 l cc s t nhin bt k . Khi y [logb a] + 1 chnh l s cc ch s ca
mt s a vit trong h m c s b.
21. Gi s r l phn d khi chia mt s nguyn m cho mt s nguyn dng n , m = pn + r
vi r {0, 1, ..., n 1} . Khi y
hmi
r =mn
n

103
22. Nu p v q l nhng s nguyn dng sao cho

p
khng phi l s nguyn th
q

 
p
p
1
>
+
q
q
q
23. Cho q l s t nhin, x l s thc dng bt k. C ng

h i
x
s t nhin khng vt qu
q

x v chia ht cho q .
24. Cc qui tc i ch (hon v), kt hp ca php ton cng v php ton nhn; qui tc
kt hp gia php ton nhn v php ton cng vn ng cho phn nguyn v phn d
25. nh l Legendre: S m ca s nguyn t p trong phn tch tiu chun ca n! c tnh
theo cng thc:
+  
X
n
vp (n) =
pi
i=1
26. nh l Hermite : Vi n N , x l s thc bt k ta c :




n1
1
+ .. + x +
[nx] = [x] + x +
n
n

Bi tp c bn
1. Phng trnh x4 3x3 6 = 0 c ng 2 nghim thc p v q . Tnh [p] + [q].
2. Gii phng trnh [x2 ] = [x]2
hxi hxi hxi
3. Gii phng trnh
+
+
=x
2
3
5
4. Gii bt phng trnh [x] + {x} < x 1
5. Cho x l s thc dng tha

x+

n N :

 


x+1 =
4x + 2 . Chng minh rng

1
+ n2 6 4n + 2 < (n + 1)2
n2

6. (THTT s 408 ) Gii phng trnh x2 (1 [x])x + 2011 = 0


7. (THTT s 424) Gii phng trnh [x]3 + 2x2 = x3 + 2[x]2
8. ( THTT s 411) Tm tt c cc hm s lin tc f : R
R tha mn {f (x + y)} =
{f (x) + f (y)} vi mi x, y R
9. ( APMO 1993) Tm tt c cc gi tr khc nhau ca hm s
 
5x
+ [3x] + [4x]
f (x) = [x] + [2x] +
3

104
n

1 X
10. (THTT s 416) Cho n > 1 s hu t r1 , r2 , ..rn tha mn 0 < ri 6 ,
ri = 1 v hm s
2 i=1


1
f (x) = [x] + x +
2
Hy tm gi tr ln nht ca biu thc P (k) = 2k

n
P

f (kri ) khi k chy trn tp cc s

i=1

nguyn.
11. ( Romania MO 2003 ) Cho A =

4n2 + n, n N. Chng minh rng


{A} 6

1
4

12. (Austrian MO 1974, Hong Kong TST 1988) Chng minh rng
h
i h
i

n+ n+1 =
4n + 2
13. ( Canada MO 1987) Cho n l s t nhin. Chng minh rng:
i h
i h
i h
i
h

n+ n+1 =
4n + 1 =
4n + 2 +
4n + 3
14. ( ngh Olympic 30/4, THPT L Qu n Qung Tr) Tm tt c cc nghim khng
nguyn ca phng trnh
96
96
= [x] +
x+
x
[x]
15. ( ngh Olympic 30/4, THPT L Qu n Qung Tr) Gii phng trnh

 

8x + 1
4x 1
16x 7
+
=
6
3
9
16. (Sweden MO 1982) Vi mi n N , hy xc nh xem phng trnh x2 [x2 ] = {x2 } c
bao nhiu nghim trn on [1;n]
17. (VMO 1979) Tm tt c nhng s sao cho phng trnh x2 2x[x] + x = 0 c hai
nghim s phn bit khng m.
18. (Olympic Czech and Slovak, 1998) Tm tt c cc s thc x sao cho
x[x[x[x]]] = 88
19. (Belarusian Olympiad 1999) Chng t rng phng trnh {x3 } + {y 3 } = {z 3 } c v s
nghim nguyn.

20. (Australian MO 1999) Gii h phng trnh

x + [y] + {z} = 200

{x} + y + [z] = 190, 1

[x] + {y} + z = 178, 8

105

ng dng nh l Hermite v nh l Legendre


ng dng nh l Hermite qua mt bi ton
nh l Hermite : Vi s t nhin n v s thc x ta lun c :
 




2
n1
1
+ x+
+ ... + x +
= [nx]
[x] + x +
n
n
n
hay
[nx] =

p1 
X
i=0

i
x+
n


()

H qu :



1
[x] + x +
= [2x] v
2

X x + i
= n [x]
j
06i6j6n

ng thc (*) l 1 ng thc p trong s hc v t hp, n c ng dng nhiu trong cc


bi ton tnh tng lin quan n phn nguyn. Trc ht ta xt bi ton m u:
V d 1: Cho n R, m N, m > 2, tnh tng
m1
X
X  n + jmi 
S=
mi+1
i=0 j=0
Gii : Ta c

 X
m1
h
X
X n
j
n i h n i
S=
+

= [n]
=
mi+1 m
mi
mi+1
i=0 j=0
i=0

Vy S = [n]. Bi ton c chng minh kh n gin nhng c ng dng kh hay trong nhiu
P
bi ton tnh tng, c bit l cc bi ton c nhiu du
. Ta s xt 1 s bi ton ng dng
cho b trn :
Bi ton 1: Chng minh rng
n1 X
m1
X
X  2k + jmi 
= 2n 1
i+1
2
k=0 i=0 j=1
HD : ng dng b trn ta c V T =

n1
X

2n 1
= 2n 1 (PCM)
21

2k =

k=0

Bi ton 2: Chng minh rng


"
p1 m1
X
XX x+
k=0 i=0 j=0

k
p

+ jmi

mi+1

#
= [px]


p1 
X
k
HD: ng dng b trn ta c V T =
x+
= [px] ( p dng nh l Hermite 2 ln )
p
k=0
..
Bi ton 3: Tm m y . 2 vi
y=

"
p1 m1
X
X X Cn2 +
k=0 i=0 j=0

k
+
p
mi+1

jmi

"
#
[ p2 ] X
m1
X
X Cp2k + jmi
k=0 j=0 j=0

mi+1

106
p2 (p 1)
.
.
2p1 . y .. 2 th p2 (p 1) .. 4.
2
iu ny dn ta ti p 0( mod 2) hoc p 1( mod 4).
Bi ton 4: ( IMO 1968) Tnh tng


X
x + 2k
2k+1
k=0

HD: D thy y =

HD: p dng b vi j = 1, m = 2 ta d dng thu c



X
x + 2k
k=0

2k+1

= [x].

Bi ton 5: Tnh tng


S=

m1
X
i=0

HD : Ta c
S=

m1
X
i=0

ai + b
am

 m1

m1 
X ai + b m1
X b
i
ai + b X ai + b

+
,
am
am
am
am m
i=0
i=0
i=0

n y p dng b trn ta c kt qu.

ng dng ca nh l Legendre v s m nguyn t


nh l Legendre: S m ca s nguyn t p trong phn tch tiu chun ca n! c tnh theo
cng thc
Xn
vp (n) =
pi
i>1
Chng minh :

 
n
n
Ta c nhn xt n < p th m = 0, m > i. Trong phn tch chun n c ng
bi s ca
p
p
p.
Do
 
n
n
[
]
n! = p p
!A1 , trong (A1 , p) = 1
p
Tng t
"
# h i
n
 
[ np ]
p
n
p !A2 , trong (A2 , p) = 1
!=p
p
p
  h i
[x]
x
M theo tnh cht
=
ta c
n
n
h i
n

 
h i 
n
n
p = n n! = p[ p ]+ p2 . n !A2
p
p2
p2
i


 
n
n
L lun tng t trn vi 2 ! v tip tc cho ti khi k < p.
p
p
Cui cng ta thu c s m vp (n) ca p trong phn tch chun n! l :
   
 
n
n
n
vp (n) =
+ 2 + ... + k
p
p
p

107
trong s k tha mn pk 6 n < pk+1 ( PCM).
Vy nh l c chng minh.
y l mt nh l c ng dng kh rng trong cc bi ton chng minh chia ht v tnh s
m nguyn t trong phn tch chun ca n!. Ta s m u bng bi ton trong IMO 1972 :
V d 1: Cho cc s t nhin m, n, chng minh rng
(2m)!(2n)!
N
m!n!(m + n)!
HD: Ta s tm s m ca p bt k trong phn tch chun ca (2m)!(2n)! v n!m!(m + n)! , sau
chng minh vp ((2m)!.(2n)!) > vp (n!m!(m + n)!).
Gii :
Ta c
 X



X
2n
2m
+
vp ((2m)!(2n)!) =
pi
pj
j=1
i=1
 X
 X




X
m
n
m+n
vp (n!m!(m + n)!) =
+
+
pi
pj
pk
i=1
j=1
k=1
Ta cn chng minh


 
     

2m
2m
m
n
m+n
+
> k + k +
pk
pk
p
p
pk

Theo tnh cht 12 ta thy BT trn ng => PCM.


(2m)!(2n)!
Vy
N . (pcm)
m!n!(m + n)!
Cc bi tp tng t :
Bi ton 1 : CMR vi m, n N ta lun c
(m + n)!
N
m!n!
HD: p dng BT [x + y] > [x] + [y]
Bi ton 2 :( USMO 1975) CMR vi n N ta c
(5m)!(5n)!
m!n!(3m + n)!(3n + m)!
l s t nhin
HD: Chng mnh v p dng BT [5x] + [5y] > [3x + y] + [x + 3y]
Bi ton 3 : CMR vi n N ta c
12(5n)!
n!(n + 1)!(n + 2)!(n + 3)!(n + 4)!
l s t nhin.
Tip theo ta s i qua mt dng mi l chng minh chia ht, kt hp vi h m c s.
.
V d 2 : Cho s nguyn t p. Tm n N sao cho n! .. pn1 .
Gii :

108
.
>p = 2 ta cn chng minh n! .. 2n1 .
Vi n l tc n = 2k + 1 ta c :

v2 ((2k + 1)!) = v2 ((2k)!) =



X
2k
2i

i=1

<

X
2k
i=1

2i

2k
= 2k = n 1
21

Suy ra s m ca 2 trong phn tch n! l nh hn n-1, do n! khng chia ht cho 2n1 .


Vi n = 2k (2m + 1) ta c :
k

v2 (n!) = v2 ((2 (2m + 1))!) =


 k
X
2 (2m + 1)
2i

i=1

k1
X

2 (2m + 1) +

X
2m + 1

t=0

2t

t=0

< (2m + 1)(2k 1) + 2m = 2k (2m + 1) 1 = n 1


Suy ra n! khng chia ht cho 2n1
Vi n = 2k ta c
k

v2 (n!) = v2 ((2 )!) =

 k
X
2
i=1

2i

k1
X
 i
=
2 = 2k 1 = n 1
i=1

Suy ra n! chia ht cho 2n1 .


Vy n = 2k l gi tr cn tm.
>p = 3 lm tng t ta thu c khng c gi tr n no tha mn.
>p > 3 ta cng c kt qu l khng c gi tr n no tha mn.
Vy ch c gi tr n = 2k , k N ( ng vi p=2) tha .
Nhn xt : T y ta c nhn xt l nu n = 2k , k N th tt c cc c nguyn t ca n! u
ln hn n-1.
Sau y l 1 s bi ton s dng h m c s trong cc bi ton phn nguyn.
V d 3: Gi s m! = 2l (2k + 1), m, k, p N, chng minh rng tn ti v hn m sao cho:
2m l = 11 + 22 + ... + 20142014
Gii:
Gi s trong h m c s 2 , m c biu din di dng:
m = an an1 ...a0 =

n
X

ak 2k

k=0

Khi
l = vp (m!) =

h
X
mi
i=0

2i

n h
X
mi
i=0

2i

" n
n
X
X
i=0

k=0

#
ak 2ki =

n
X
k=0

"
ak

n
X
i=0

#
2ki

109


n
P
Mc khc ta c ai
2ki = 0 chnh v th ta c :
i=k+1

l=

n
X

"
ak

k=0
"
n
X
k=0
n 
X

k
X

#
ki

i=0
k
X
1
k
ak 2
2i
i=0

1 
2k+1
=
ak .2
1 12
k=0


n 
n
X
X
1
k
ak .2 . 2 k
= 2m
ak
=
2
k=0
k=0
k1

Suy ra
2m l =

n
X

ak

k=0

Cng vic cui cng l phi chng minh tn ti v hn b s (ak )ni=0 sao cho
n
X

ak = 11 + 22 + ... + 20142014

k=0

M iu ny l hin nhin. Vy bi ton c chng minh.


Ta cng xt cc v d tng t sau:
Bi ton 1: Gi s m! = 3l (3k + 1), m, k, p N, chng minh rng tn ti v hn m sao cho:
m 2l = 1! + 2! + ... + 30!
HD : Lm tng t bi trn, gi s m = an an1 ...a0 =
n
X

n
X

ak 3k , sau bin i ta c m 2l =

k=0

ak .

k=0

Hm c cha phn nguyn


Trong t hp, cc dng bi ton tnh tng rt a dng v nhiu cch gii . Mt trong nhng
vn hay gp trong cc bi ton tnh tng l tng cc hm c cha phn nguyn.. Sau y
ti xin c gii thiu 1 dng ton kh p v phn ny vi 2 bi ton m u.
V d 1: Cho p l 1 s nguyn t l, q l 1 s nguyn khng chia ht cho p, f : N
R , tha
1) f (x) khng chia ht cho p , vi mi x N
2) f (x) + f (p x) chia ht cho p vi mi x N
Chng minh rng :

p1
p1 
X
q
qX
p1
f (k). =
f (k)
p
p k=1
2
k=1

110
Gii :
qf (k)
qf (p k)
qf (k) qf (p k)
+
Z, m

/ Z nn

/ Z vi mi k = 1, p 1.
Ta c
p
p
p
p
Do vy t tnh cht phn l ta c
 


qf (p k)
qf (k)
+
<2
06
p
p
Suy ra


qf (k)
p


+

qf (p k)
p


=1

Ly tng cc gi tr t 1 n p-1 ta c :

p1 
X
qf (k)
p

k=1


p1 
X
qf (p k)
p

k=1

=p1

Suy ra
2


p1 
X
qf (k)
p

k=1


p1 
X
qf (k)

=p1

k=1

p1
2

T ta c
p1 
X
k=1

 X
 X
p1
p1 
p1
X
q
q
q
p1
q
f (k)
f (k) =
f (k)
f (k) =
p
p
p
p
2
k=1
k=1
k=1

Suy ra pcm.

p1
p1 
X
p1
qX
q
f (k)
Vy
.
f (k) =
p
p k=1
2
k=1
Vic p dng bi ton trn kh d v ch cn thayf (x) bng 1 hm hay 1 gi tr no .
Bi ton p dng:
Bi ton 1: Cho p,q l 2 s nguyn dng v nguyn t cng nhau, chng minh rng :

p1 
X
kq
k=1

(p 1)(q 1)
2

HD gii : Chn f (k) = k , theo bi ton m u ta c



p1 
X
kq
k=1

p1
X
k=1

f (k) =


p1 
X
f (k)q
p

k=1

p1

qX
p1
=
f (k)
p k=1
2

p(p 1)
nn d dng c
2

p1 
X
kq
k=1

(p 1)(q 1)
2

Bi ton 2 : Cho p l s nguyn t l, tnh


p1  3 
X
k
k=1

111
HD: Chn f (k) = k 3 , khi
p1
X


f (k) =

k=1

p(p 1)
2

2

.
Bi ton 4: Cho p l s nguyn t l, tnh
"
#
p1
X
(1)k+1 k 3
S=
p
k=1
p dng gii phng trnh S = 0.
HD : Chn f (k) = (1)k+1 k 3 , khi
n
X

(n + 1)2 (2n 1)
f (k) =
4
k=1

khi n chn v

n
X

f (k) =

k=1

n2 (2n + 3)
4

khi n l.
Ta nhn thy trong cc bi trn, tt c deg f (k) u l ( v khi mi p ng c iu kin
2 ca . Cn nu deg f (k) chn th sao? Trng hp ny ta s xt cc v d sau:
Bi ton 5 : Vi p l s nguyn t c dng 4k + 1, tnh
S=

p1  2 
X
i
i=1

Nhn xt : Ta th lm ging cc bi trn. Chn f (k) = k 2 . Khi d thy f(k) khng chia ht
cho p v f (k) + f (p k) = k 2 + (p k)2 = 2k 2 2pk + p2 cng khng chia ht cho p. iu ny
mu thun vi 2 iu kin cho. Vy ta khng th lm nh cc bi trn c.
Gii :
Trc ht chng minh b .
p1
B : Vi p l s nguyn t tha p 1(mod4) th mi s t nhin a vi 1 6 a 6
s
2
p+1
lun tn ti duy nht s t nhin b tha mn
6 b 6 p 1 v a2 + b2 0( mod p).
2
Chng minh : Theo nh l Wilson ta c : (p 1)! 1( mod p).
p1
Vi mi k = 1,
th p k k( mod p) k(p k) k 2 ( mod p).
2
p1
Kt hp vi gi thit p 1( mod 4)
0( mod 2), ta c:
2
 2 
 2

p1
p

1
p

1
1 (p 1)! (1) 2
! =
! (modp)
2
2


p1
t =
! 2 1(modp)
2
p1
p+1
ta chn
6 b 6 p 1 tha mn b2 a2 2 (modp), d thy b tn
Vi mi 1 6 a 6
2
2

112
ti v duy nht.
Khi
a2 + b2 a2 (p2 + 1) 0(modp)
Suy ra PCM.
Vy b c chng minh.
p+1
p1
v
6 j 6 p 1 ta thy :
p dng vi 1 6 i 6
2
2
S=

p1  2 
X
i
i=1

p1

2  2 
X
i

i=1

i2 j 2
+
p
p



=

p1 2
X
i
i=1

p1
(p 1)(p 2)
=
2
3

.
Vy
S=

(p 1)(p 2)
3

Tip tc l bi 5 trong thi Chn i tuyn IMO Vit Nam nm 2005 (Vietnam TST 2005).
Bi ton 6 (Vietnam TST 2005) : Cho p l s nguyn t (p > 3). Tnh:
p1

 2 
2 
X
2k 2
k
2
nu p 1(mod4).
a) S =
p
p
k=1
p1
2  2
X
k
nu p 1(mod8).
b) P =
p
k=1
Gii:
Trc ht ta chng minh 2 b :

p1
B 1 : Vi p l s nguyn t tha p 1(mod4) th mi s t nhin a vi 1 6 a 6
s
2
p+1
lun tn ti duy nht s t nhin b tha mn
6 b 6 p 1 v a2 + b2 0(modp).
2
1
B 2 : Vi x l s thc bt k th [2x] 2 [x] bng 1 nu
6 {x} < 1 v bng 0 nu
2
1
06x< .
2
Chng mnh :
B 1 : Xem b ca bi 5.
B 2 : Ta c x = [x] + {x}. Suy ra
[2x] 2 [x] = [2 [x] + 2 {x}] 2 [[x] + {x}] = [2 {x}] 2 [{x}] = [2 {x}]
Khi :
1
+) nu 6 {x} < 1 th [2 {x}] = 1 => [2x] 2 [x] = 1
2
1
+) nu 0 6 x < th [2 {x}] = 0 [2x] 2 [x] = 0
2
Vy b c chng minh.
Quay tr li bi ton :
p1
s hng.
2
Theo b 2 th tt c s hng trn u c gi tr l 0 hoc 1.

a) Ta thy S c tt c

113
Theo b 1 th vi mi s t nhin a vi 1 6 a 6
b tha mn

p1
s lun tn ti duy nht s t nhin
2

p+1
6 b 6 p 1 v a2 + b2 0(modp)
2

Suy ra a2 + (p b)2 0( mod


 p)

p1
0
Do tn ti duy nht a 1;
tha a2 + a0 2 0(modp).
2

Gi x,y ln lt l s cc s d khi chia k 2 cho p 1 6 k 6 p1
c gi tr ln hn v nh hn
2
p

1
p1
p1
.
Theo
nhn
xt
trn
th
x
=
y,
hn
na
x
+
y
=

x
=
y
=
.
2
2
4
T S = x.1 + y.0 = p1
.
4
p1
Vy S =
.
4
b) Do p 1( mod 8) nn tn ti a sao cho a2 2( mod p) v p 1( mod 4).
Ta c :
p1

P =

p1

2  2
X
k

k=1

p1


2 
X
2k 2
k=1

k2

p




p1


2 
X
2k 2
p

k=1

Ta cn tnh


=


2 
X
2k 2
k=1

k2

p



S

p1


2 
X
2k 2
p

k=1

Ta c :

k2
2
p


k2

p




p1


2 
X
2k 2
k=1

k2

p




p1

p1

2 
X
2k 2

k2

p
p

k=1
p1
2

p1
2

X k2
k=1


2 
X
2k 2

k=1

X 
k=1

2k 2
p

k2
p

k2
p





Theo nhn xt trn th tp hp cc s d khi chia k 2 , 1 6 k 6


p1
cc s d khi chia 2k 2 ,1 6 k 6
cho p tc l
2

p1
cho p trng vi tp hp
2

p1


2 
X
2k 2
p

k=1

k2
p


=0

Suy ra
p1

p1


2 
X
2k 2
k=1

k2

p



=

2  2
X
k

k=1

p2 1
24

p2 1 p 1
(p 1)(p 5)

=
.
24
2
24
T 2 nhn xt trn ta thy b 1 kh quan trng, v c s dng thng xuyn trong cc
Vy P =

114
bi ton tnh tng dng ny. Sau y l cc bi ton tng t :
Bi ton 7: Cho p 1( mod 3), m khng chia ht cho p, tnh

 2 
p1 
X
3i
3mi2
m
p
p
i=1
(m 1)(p 1)
.
2
Bi ton 8: ( THTT s 430) Cho p l s nguyn t c dng 4k + 1 , tnh tng

 2 
p1 
X
2k 2
k
2
p
p
k=1
HD : Lm tng t ta c kt qu

p1
.
2
Bi ton 9: Cho p l s nguyn t c dng 4k + 1 , tnh tng

 3 
p1 
X
2k 3
k
2
p
p
k=1
HD: Gii tng t nh cu a bi ton 6 v cho kt qu l

p1
.
2
Nhn xt : Bi ton 8 v 9 cho ta nhn xt v bi ton tng qut l tnh tng S =

 n 
p1 
X
2k n
k
p1
, vi n=1 th cng lm nh trng
2
vi n = 2 v n = 3 th S =
p
p
2
k=1
HD : Bi ny ta s dng kt qu ca bi ton m u 1 , v cng cho kt qu l

hp n=3 ta cng c kt qu tng t. Vy n Nthi kt qu trn c ng khng? p n l


ng, v phn chng minh xin dnh cho bn c.
V d 2: Chng minh rng vi p l s nguyn t tha p 1(mod4), vi hai hm s f (x) v
g(x) tha mn
(
(f (x), p)) = 1
x
(g(x), p)) = 1
Khi chng minh rng
p1 
X
i=1




g(x)f (x)i2
f (x)i2
(g(x) 1)(p 1)
g(x)
=
p
p
2

Chng minh: Bi ny d dng chng minh bng bi ton m u 1 v theo cch ca Bi ton
6 cu a.
p dng bi ton trn kh hay khi ch cn chn 2 hm tha mn l c th s dng. V d nh
bi 8 : Chn f (x) = 1, g(x) = 2, khi
 2 
p1  2 
X
2i
i
(2 1)(p 1)
p1
2
=
=
p
p
2
2
i=1
Bi tp tng t:
Bi ton 10: Cho p l s nguyn t c dng 4k + 1, chng minh rng

 2 
p1 
X
(p + 1)i2
i
2
(p + 1)
0( mod 2)
p
p
i=1

115
Bi ton 11: Cho p l s nguyn t c dng 4k + 1, chng minh rng

p1 
X
(2p2 + 2p + 1)i2
p

i=1

 2 
i
(2p + 2p + 1)
p
2

khng l s nguyn t vi mi p.
Bi ton 12: Cho p l s nguyn t c dng 4k + 1. Tm p
S=


p1 
X
(2p + 3).i2
p

i=1

 2 
i
(2p + 3)
25
p

l s chnh phng.

Hm phn nguyn trong vic tnh tng cc ch s


nh ngha
Gi s n l 1 s t nhin. Ta nh ngha S(n) l tng ca cc ch s ca n khi biu din trong
h thp phn.

Tnh cht
1. Vi n l s nguyn dng ta c
h
X
n i
S(n) = n 9
10k
k=1

CM: Trong h thp phn ta biu din


n = am am1 ...a0 = 10m am + 10m1 am1 + ... + a1 .10 + a0
Khi
n9

h


X
n i
=
a
a
...a
|10

9
a
a
...a
|10
+
a
a
...a
|10
+
...a
m m1
0
m m1
1
m m1
2
m
10k
k=1

Suy ra n 9


P
k=1

n
10k

m
P

ai (10i 9(10i1 + 10i2 + ... + 1) =

i=0

T c pcm.
2. S(n) n(mod9)
h
h
X
X
n i
n i..
l
s
nguyn
nn
9
.9
CM: Ta nhn thy
10k
10k
k=1
k=1
Suy ra pcm.

m
P
i=1

ai = S(n).

116
3. 0 6 S(n) 6 n
CM : D thy S(n) > 0, v t
h
X
n i
S(n) = n 9
10k
k=1

Suy ra S(n) 6 n ( du = xy ra khi 0 6 n 6 9)


4. S(m + n) 6 S(m) + S(n)

!
h

X
m i Xh n i
CM : Ta c : S(m) + S(n) = m + n 9
+
.
k
k
10
10
k=1
k=1
p dng BT [a] + [b] 6 [a + b] ta c
S(m) + S(n) > m + n 9



X
m+n
k=1

10k

= S(m + n) (pcm)

Du bng xy ra khi v ch khi php cng m + n khng c nh.


5. S(m.n) 6 S(m).S(n)
Bn c c th chng minh thng qua biu din ca m, n v mn trong h thp phn.
T ta c tng qut :
X
X
6. S(
ai ) 6
S(ai )
i=1

7. S(

i=1

ai ) 6

i=1

S(ai )

i=1

Bi tp v d

V d 1: Tm n sao cho
S(n) = n2 2014n + 5
Gii :
Ta c 0 6 S(n) 6 n 0 6 n2 2014n + 5 6 n.
Gii h bt phng trnh, kt hp n N ta c n = 2014.
Vy n = 2014.
V d 2: Tm n sao cho: n + S(n) + S(S(n)) = 2001.
Gii :
Ta c n 6 2000 S(n) 6 S(1999) = 28 S(S(n)) 6 10 n > 1972.
M
3n n + S(n) + S(S(n)) 2001 3(mod9) n 1(mod9)
n {1963, 1966, 1969, 1972, 1975, 1978, 1981}

117
Bng cch th trc tip ta thy cc gi tr n cn tm l n {1969, 1972, 1975}.
V d 3: Tm n tha n + S(S(n)) = 2014.
Bi ton ny xin dnh cho bn c.
V d 4 : (PTNK 2008) Vi mi s nguyn dng n, gi S(n) l tng cc ch s ca n.
a) Chng minh rng cc s n = 999 v n = 2999 khng th biu din c di dng a + b vi
S(a) = S(b).
b) Chng minh rng mi s n, 999 < n < 2999 u biu din c di dng a + b vi
S(a) = S(b).
Gii :
a) Gi s c th biu din c.
>n = 999 : Ta c : a + b = 999, m php cng trn khng c nh nn S(a) + S(b) = S(a + b) =
S(999) = 27
M S(a) = S(b) nn 2S(a) = 27 (v l)
>n = 2999 :
Tng t v php cng a+b=2999 khng c nh nn ta c PCM.
b) Trc ht ta chng minh rng nu 999 < n < 2999 th tn ti s t nhin k sao cho
S(k) + S(nk) l mt s chn. Tht vy, nu S(n) l s chn th ta chn k = 0. Nu S(n) l,
gi s n = ba2 a1 a0 , trong b {1; 2}. Do 999 < n < 2999 v n 6= 1999 (do S(n) l) nn tn
ti i sao choai < 9. Chn i ln nht tho mn iu kin ny. Khi chn k = 10i.(ai + 1) th
S(k) = ai + 1 cn S(n k) = S(n)ai1 + 9 (php tr c nh to ra s 9 v tr ai v gim i
1 n v v tr trc ). T suy ra S(k) + S(n k) = ai + 1 + S(n)ai1 + 9 = S(n) + 9
chn do S(n) l. By gi gi s ta tm c k sao cho S(k) + S(n k) l s chn. Khi
nu t
k = a3 a2 a1 a0 v n k = b3 b2 b1 b0
Do S(k) + S(n k) chn nn s cc ch s i sao cho ai + bi l l chn. Vi 1 cp ch s (i, j) sao
cho ai + bi = 2ji + 1, aj + bj = 2ki + 1 l, ta i ai a0i = ki + 1, b0i = ki , a0j = kj , b0j = kj + 1.
Vi cc ch s i sao cho a1 + b1 = 2ki , ta i ai a0i = b0i = ki . Khi d dng nhn thy rng
a03 a02 a01 a00 + b03 b02 b01 b00 = a3 a2 a1 a0 + b3 b2 b1 b0 = n
v


S a03 a02 a01 a00 = S b03 b02 b01 b00
T ta c iu phi chng minh.
V d 5: (IMO 1975) t A = S(44444444 ) v B = S(A). Tm S(B).
Gii :
t N = 44444444 . Do N < 100004444 nn N khng qu 4444.4 < 20000 s .
T
A < 9.20000 = 180000 B 6 S(9999) = 45 S(B) 6 39 = 12 (1)

118
Mt khc
4444 (2)(mod9) N 24444 = 81431 .2 (2)(mod9)
Do S(B) 7( mod 9) (2) T (1) v (2) ta c S(B) = 7.
Bi tp tng t:
V d 6: t A = S(30.42012 ), v B = S(A). Tnh S(B).
V d 7: (VMO 20004) Tm gi tr nh nht ca S(n) khi n chy trn tp cc bi ca 2003.
HD : S dng 3 b :

1001 l s nguyn dng nh nht trong s cc s nguyn dng mm 10m 1(mod


2003).
Khng tn ti bi dng ca 2003 c dng 10k + 1 vi k N.
Tn ti bi dng ca 2003 c dng 10k + 10h + 1, vi k, h N.
V s dng thm cc tnh cht ca S(n) ta c p n min S(n) = 3 .
Bi tp t luyn :
Bi ton 1 : Cho s t nhin N tha S(N ) = 100, S(5N ) = 50. Chng minh rng N chn.
Hng dn: t M = 5N th S(M ) = 50 v S(2M ) = S(10N ) = S(N ) = 100. Suy ra php
cng M + M = 2M l php cng khng nh.
Bi ton 2: Tm n nh nht sao cho trong n s t nhin lin tip ty lun chn c mt
s N m S(N ) vdots 13.
p n : 79.
1999

Bi ton 3: t a = S((29 )
p n : c=8.

); b = S(a); c = S(b). Tm c.

Bi ton 4: CMR vi n l s t nhin bt k ta lun c


S(8n)
1
>
S(n)
8
Bi ton 5: Cho a l s chn nhng khng chia ht cho 5. Chng minh rng
lim S(an ) = +

n+

Bi tp tng hp

119
Bi ton 1 : Cho n l s nguyn dng tha mn n! c ng 2002 ch s 0 tn cng. Chng
minh rng n 6 8024 .
Bi ton 2 : Vi n l s nguyn , chng minh rng
 
 


n+4
n1
n+2
+
+
=n
4
4
2
Bi ton 3 : ( i Loan 1998) Chng minh rng vi mi s nguyn dng m v n, ta lun c:
(m, n) = 2

m1
X
k=0


kn
+ m + n mn
m

Bi ton 4 : Tm min{xn |1 6 n 6 M } vi M = 20142014 v

x 1 = M
 M 
(xn ) :
x+[ x ]
n

n > 1
xn+1 =
2
Bi ton 5 : Tm s nguyn dng n ln nht sao cho 2014! chia ht cho 7n .
Bi ton 6: (Canada 1998) Tim s cc s a tha mn
hai hai hai
+
+
=a
2
3
5
n h i
P

Bi ton 7: ( Hn Quc 1997)Tnh tng


k theo n v a = n.
k=1

Bi ton 8 : Cho (m, n) = 1 vi m chn. Tnh tng


n1

X
km
1
(1)[ n ]
S=
+
2n k=1

km
n

Bi ton 9 : (Balkan 1998 ) Tnh cc s hng khc nhau trong dy


 2 

k
: k = 1, 2, ..., 1997
1998
Bi ton 10 : Chng minh rng tch ca n s nguyn lin tip lun chia ht cho n!.
Bi ton 11 : (APMO 2001) Tm s nguyn N ln nht sao cho s cc s thuc tp hp
{1, 2, . . . , N } v chia ht cho 3 bng s cc s thuc tp v chia ht cho 5 hoc 7.
Bi ton 12 : Chng minh rng vi mi s nguyn dng n > 3 th
.
(2n 1)(2n 2)(2n4). . . (2n 2n 1) .. n!
Bi ton 13 : Chng minh rng
Cnk 1 (

mod 2) k = 0, n n 1 (

mod 2)

120
Bi ton 14 : Cho n > 2 l s nguyn dng. Chng minh rng:
n  2
X
n
k=2

.
Bi ton 15 : Tnh

lim

n
+

1X
n k=1

n2  2 
X
n
=
k
k=n+1



!

h n i
2n
2
k
k

Bi ton 16 : (nh l LTE) Gi vp (n) l s m ca p trong khai trin ra tha s nguyn t


ca n. Cho p l s nguyn t l, x, y l cc s nguyn sao cho x, y khng chia ht cho p nhng
x y chia ht cho p, n l s nguyn dng. Chng minh
vp (xn y n ) = vp (x y) + vp (n)

Ti liu tham kho


1. Phn nguyn- Bi tp v ng dng : Hong Xun Thanh.
2. V p phn nguyn t tnh cht c bn : Nhm hc sinh chuyn ton trng KHTN H
Ni.
3. Hm phn nguyn v ng dng: Lun vn thc s ton hc ca T Duy Phng.
4. S hc qua cc nh l v bi ton : Trn Nam Dng.
5. Cc hm s hc v ng dng : Lun vn thc s Ton hc, Cao Sn.
6. Cc din n : mathscope.org , diendantoanhoc.net/forum, artofproblemsolving.com/Forum.
Ngoi ra tc gi cn nhn c s ng h v gip sc ca Tin s Trn Nam Dng, xin trn
thnh cm n s gip ca thy. Chc cc bn s hiu su hn, chc hn v Phn nguyn
sau khi c bi vit ny.

You might also like